Текст
                    н. А. ШАПОШНИКОВ и Н. К. ВАЛЬЦОВ
СБОРНИК
АЛГЕБРАИЧЕСКИХ
ЗАДАЧ
ДЛЯ СРЕДНЕЙ ШКОЛЫ
ИЗДАНИЕ 13-е, ПЕРЕРАБОТАННОЕ
ЧАСТЬ ВТОРАЯ
8—9 ГОДЬ£ ОБУЧЕНИЯ
I
9
УТВЕРЖДЕН^,КОЛЛЕГИЕЙ НКП РСФСР
Ь'ПШ
щ
ГОСУДАРСТВЕННОЕ
УЧЕБНО-ПЕДАГОГИЧЕСКОЕ ИЗДАТЕЛЬСТВО
МОСКВА 1933


i #•»**** • *« »» * 1 » lUtPT Отв. редактбр В. Молодишь. Техн. редактор М. Ткачуна5. Сдано в набор 29/IV—33 г. Подписано к печати 25/V—33 г. Формат бумаги 62X94 */»• Тираж 100 ООО экз. Издат. листов 61/.. Бум. .шстов З1/.- 1060.'О аы. в бум. диете. У—21. Учпедгиз № 4812. Заказ К 6740. Уполн. Главлта Ь—30849. Третья ф бр^ка книги Огиза РСФСР треста «Полигртфкннга» «Красный пролетарий». Москва. Краснопролетарская, 16. Г
ГЛАВА ГХ. ИРРАЦИОНАЛЬНЫЕ ВЫРАЖЕНИЯ. § 1. Общие сведения о корнях. Извлечение корня из одночлена. Определение. Корнем п-й степени из а называется такое коли¬ чество х, которое, будучи возвышено в п-ую степень, дает а. Выражают это количество х знаком Уа. В равенства 1/а = х а называется под¬ коренным количеством, л—-показателем корня, а х или равное ему У а — корнем л-й степени из а. Действие отыскания х по данным я и л называется извлечением корня. Правило знаков. Корень четной степени из положительного коли¬ чества имеет два знака', положительный и отрицательный; так 2f/ -j- а = = ±2^я. Корень четной степени из отрицательного количества есть мнимая величина; таков корень 2j/— а, если само а есть абсолютное число. Корень нечетной степени из всякого колцчес/пва, положительного или отрицательного, имеет тот же подкоренное количество; так: *’ 2п4-1/— . 2п+1/— 2я+1/ — 2я+1/— + а = -f |/ я, у — а = — у я. Теорема 1. Корень из произведения равен произведению корней из каждого множителя; так, \/ab = {/ я • ?ъ. . Теорема 2. Корень из дроби равен корню из числителя, разделен¬ ному на корень из знаменателя; так: Теорема 3. Корень из степени получается через деление показателя степени на показатель корня; так, |/ят” = ят. Извлечение корня из одночленов. Чтобы извлечь корень из одночлена, нужно поставить знак по правилу знаков, затем извлечь требуемый корень из каждого множителя и делителя и расположить результаты множителями или делителями, соответственно тому, как располагались множители и делители данного одночлена. При этом корни из числовых коэфициечгов извлекаются непосредственно, а к буквенным выражениям применяется третья теорема. Например, имеем: 3 / 2_ Зп2Ъ У 64c3ndis — 4сва*' 1»
4 Глава IX Показатель кормя может быть отрицательным количеством. Всякий корень с отрицательным показателем равен единице, разде¬ ленной на подобный же корень с положительным показателем. Так: К корням с отрицательными показателями применяются указанные выше: правило знаков, все три теоремы и правило извлечения корня из одно¬ члена. Извтечь корень из одночленов: 1. 3. 5. У 32.10-’. з/ 55 ‘ к 64' 9. j, 1rv&“. II. “^32. 13. — 3 . 1. у'2*5. 3. УУ”- 5. У 8 • З3. 7. (/£. 9. 1. ~^/27. 3. jj/a-6. V=l- «»■ У^- 5. 7. ^16п-4й12, 8 К/Н5“”Г‘- 9. 1/б4-н6с1,п. 4/ 1C 20. |/ ga8"M6. 21. ft 0,027a6”-3bl*c~e. 22. 23. ]/| 4-<я*6-в 25- У “2/д 4/i“6/^HhF ■ 9/ 29. 2. 3/^~в. 4. я+^'йз«+в. 6. ^16-81. 8- У -£• 2. У —1010. 4. 3+yyi5+5n> 6. ^125-ЮОбГ 8. х |/ feu- 10. 1/-27аЩ\ .2. У|. 10. У-32п5^». 12. У|. 14. У-п-2«. >в- УУЕ. 14. У — а-11. 16- 17. у 64а-12&*. 18-Jra_8^4- 10» 19. |/1 |Л го. уу.“Я 21. У0,0625:г1п^&21с-1а. 22. У — 64а3л-6&14 24 V o-fii у 26. УI 28. 2abn- Y %а3Ьс* УШЛ¥*. 1000ри<г6/зп 27а-3 ,Mi9 * 30. 3w'-'b ;-«i ь-1я л/21 К64й 15fo3tt£6S»j9
Иррациональные выражения б § 2. Вывод множителя из-под радикала и введение множителя под радикал. Если подкоренное выражение разлагается на два множителя, из которых один представляет полную степень, а другой — неполную, то можно извлечь корень из первого множителя и полученное рациональное выражение умно¬ жить на иррациональный корень из второго множителя. Такое преобразо¬ вание называется выводом множителя из-под радикала. 31. / 8. 31. 1/18. 32. /75. 32. /28. 33. / 81. 33. /500. 34. / — 108. 34. / —72. 35. /48. 35. /162. 36. /1250.1 36. /П2. 37. /486. 37. /96. 38. / —224. 38. / —1215. 39. 2/405. 39. 3/Т92. 40. f/243. 40. 4/128. 41. 1/'а3с*. 41. /ви£*. 42. Уа'ЧК 42. /а5**. 43. /я*_у3. 43. /* 1ву. 44. /Ж 44. / а1067. 45. |/4 а*Ь. 45. /25 аЧ. 46. / 64xe_y1. 46. / 27xs_v3. 47. 43 о 00 со . 47. 2/ 75Л14. 48. */?• 48. з/1- 49. *Га3 У 49. Vi- 50. л/-'- у ЫЯ’ 50. Vi’ 51. . /0,542 й V dW . 51. *V=&- 52. J/ —0,729т У в® 52. /8,64т У ~а*~ш 1 /(°2 — 2яЬ -j- b2)y 53" / 502 25 К а2 + 2ай + 63* 54. Vi~ 1 6* 64. I3/-3-1 / а» д * 55. -*Ду*_д8)» V 8(*+.У/ 55. ,V(*2-J'2)e У 32{у-хУ 56. V/ ы аУ S- 6* ' аР‘ 56. 2а У а 9 57. У ‘2п'л1ат'тЬт V пр+1 57. т + j*/ д2т 1 и* 58. yz^/x^ {■1уйг t'-l z\ 59. 1—Зп Если при корне находится рациональный множитель, то можно ввести его под радикал, возведя его для этого в степень, указываемую показа¬ телем корня, и умножив результат на подкоренное выражение. Такое пре¬ образование называется, в в едением множителя под радикал.
6 Глава IX 60. 21/3. 60. 3|/ 2. 61. 6^ 5. 61. 4у/ 3. 62. ЪУ~2. 62. 2$"3. . 63. 5|ГЗ. 63. 71/~2. 64. 2 у 5\ 64. 3^4. 65. а|/5. 65. 5^/ а. 66. хУ 2. 66. У Г5. 67. Ъ\Га. 67. а^ 5. 68. з — ту п. 68. пУ т1. 69. rC-у а. 69. тУ а. 70. За j/ ах. 70. а3 У^аЪ. 71. тъ Утп. 71. 2п Уmbi. 72. 72. 2 з a*"** 73. 73. У */■* X У у ' 74. а 3 / 61 Т У ~~~а?' 74. -W- а2 Ь3‘ 75. 5/ " 1 тУ1-^’ 75. — У т3 — 1 т 76. (т+п) |/- 1 /и2 — -и2' 76. —-— У т% т — пг — п 3 77. 2ас3 l/8abc1 1 77. 4а J'/ 276» 36 У 16а«* 78. ЗапЬ-у’ЗаЧ. 78. 2аЬт-пУд,апЬ\ 79. За2с3 р 2ап1 С*. 79. 2 а”Ь~* УЪа »ь3 § 3. Сокращение показателей корней и приведение радикалов к общему показателю. Величина корня не изменится, если умножим или разделим показатель корня и показатель подкоренного выражения на одно и то же число. Из етой теоремы выводятся два следствия: 1. Если показатель корня и показатель подкоренного выражения содер¬ жат общий множитель, то на этот множитель их можно сократить. 2. Если несколько корней имеют различные показатели, то, умножая показатели корней и показатели подкоренных выражений соответственно на одинаковые числа, можно привести корни к одинаковому показателю. Умножить показатель подкоренного выражения — значит то же, что воз¬ вести это выражение в соответствующую множителю степень. Разделить показатель подкоренного выражения — значит то же, что извлечь из этого выражения соответствующий делителю корень. Сократить показатели корней: 80. У а6. 80. у а. 82. 3Уа1пЬ*п. 82. S’/a1V1’1 84. V 9п46®. 84 |/4а8йа. 86. 'УШИь ". 86. v 81а1664” 16л1&12 81с • 81. 83. 85. У27а3тЬ6. 89. -£Ла~*й™<г*. 81. 83. 7й2™й3». 85. У 64 а9Ь3т. 87 ^п/27сгШ^ .у 89. У 9 я*й'8с‘. 8с«
Иррациональные выражения 1 Привести к общему показателю корни: 90. fya* и р'а3. 91. У 2а2 и \/аЬя. 90. {/а1 и £ а"’. 91. /ЗБ и (/2?. 92. У1к№ и \/2hb. 92. р'2а*Ь и ^3а3Ь. 95. \/аЬ*, У а1 и Л\/Ья. 96. *\/£?Ъ, 1y'asb* и "(/'а10#2®. 96. 1р/а7Ь3 и 1j/'a,e^ss- § 4. Приведение корней к нормальному виду. Всякий корень может быть приведен к простейшей или нормальной форме. Для этого нужно произвести последовательно следующие действия. Преобразовать подкоренное выражение в одночлен, если такое преобра¬ зование не сделано и возможно. Сократить показатель корня, если последний имеет общий множитель с показателями всех множителей и делителей подкоренного выражения. Выделить из-под радикала ту часть подкоренного выражения, которая допускает извлечение корня. Уничтожить иррац тональность знаменателя. Последнее преобразование состоит в том, что умножают числитель и знаменатель подкоренного выражения на одно и то же выражение, выбирая множитель так, чтобы знаменатель сделался полной степенью, и затем из¬ влекают из знаменателя корень. Привести к простейшей форме следующие корни:
8 Глава IX § 5. Подобие корней. Когда корень приведен к простейшей форме, то рациональный множи¬ тель корня называется его коэфициентом. Корни называются подобными, если они различаются только коэфи- циентами, но имеют одинаковые показатели корней и одинаковые подко¬ ренные выражения. Чтобы судить о том, подобны ли данные корни или нет, нужно привести их к простейшей форме. Доказать подобие корней. Для сложения и вычитания корней соединяют их посредством знаков этих действий. Затем приводят корни к нормальному виду и, если между корнями окажугся подобные, делают приведение. Это приведение со¬ стоит в том, что коэфициенты подобных членов, взятые со знаками соот¬ ветствующих членов, заключают в скобки, а общий корень выводят за скобки множителем. Затем полученный общий коэфициент упрощают. 09. / 3 и >/12. 11. /$4 и 2 f/T. И ^ / ~ 2\.y~tfb и 122. УЪ,027ху* и у 0,064у. 28. у|^("-Dy^1, JL ухи (2х— у) Ух5~*у. § 6. Сложение и вычитание корней. 129. (5/2 — 4 /з) + (3 \/2 + 6 /з). 129. (7/4 — 2/5)-(5/4 — 4/5). 130. (10/Г+^3)-(5^3#+ 2^7).
Иррациональные выражения 9 30. (2/ll-8/7) + (7/7—/ll). 31. (а/b — ft/с) — (За/ft— 5£/с). ЗГ. (з/а + б/с)—(2/а + Зб/с). 32. (а /fc* — 2с p'd) — ( — 5с {Td + За /£*). 32. (2/а* — /а*б) + (— /а^+б’/а^)- 33. /2 + 3/32 + |-/Т28 — 6 /18. 33. [/75 -/147 +1/48 —1/300. 34. 20/245 —1/5 + 1/125 —2-i-j/TsO. 34. /275—10/11 — 2/99+ 1/396. 35. /5-2^-/40+10/135-/320. 35. 3/2 — /64+10/486 —6-i-/2. 36. /20-5 ^1-1 ,/Ж-|/|. 36. 2^/4-|/60-^ + |/4+|/i. 37. 3-i-/24-^+2^-li-/44+3/2. 37. /54 + |/1-/250—|у/|+^/б|. 38. 6|/8-8j/4-/4 + 6/4-H-t.i4i. 38. ЗУ32 + ^74-^Т®-16^/^+4^. 39. /а5 + b /а — /9а. 139. /а® + 40. /270* —3/8а+ 3/125а7. 140'. /а*6 — /32^® + За //. 41. 3 /125а36а + Ь /20а5 — /500а*6*. 41. 2 /я®* — За3 /646 + 2а* /1256*. 42. ^ЗЛ^-^1/Т2Л13-Л-|/|?. 42. 4ас2 /а5*7 + Ь3 /Л¥ — /8а2613Л 43. 5/^у + 4у + |/T^V-6^17|-|.vyjy3Z.
10 Глава IX 143. 3/ху + 6*У\/~~ 4*У \/-xlys+Yy •У-*У- 144. /wz3 —/ягя— V{m + n) {m- — nl)—/tnri* — na. 145. j/1 — ~ — 3/4^2* — ]/ 16 — 8x +8-|/4~f' 146. (a* - 2bl) ]/£±| - (a2 + b*) i/(a + bf(a-b) + j/frt&*-fce. 147. ~У(1 + 2x + x2) (x + 1) (x* -1) - /^(I-jT1) + i jt3 . 148. /8*9 — 8-*У + Уx*y* — x6 + У1 — x3^-3 + /x~sy3 — х“6У- § 7. Умножение и деление корней. Произведение корней с одинаковыми показателями (одноименных корней) равняется корню той же степени из произведения подкоренных выражений. Частное от деления одноименных корней равно корню той же степени из частного от деления подкор?нных выражений. Если показатели корней различны, то их сначала приводят к общему- показателю, а затем производят умножение или деление по предыдущим правилам. Когда корни имеют коэфициенты, то последние перемножают или делят отдельно и результат пишут перед полученным общим корнем. Произвести указанные действия над корнями: 149. /3 -/27. 149. /5- /20. 150. У 2 4 16. 150. /3-/18. 151. з/Ts-l/^. 151. 2,3 16-1/-5. 152. ~3У27- J/243. 152. i/32-1/128. 153. /^Т08. 50 • /40. 153. /73- /—112 - /l4. 154. 2/32^'2Тб-3/б0. 154. / 1024 -2/6561 -/1620. 155. (Vs + ftH2—У55) • 8/32. 155. (2j/ Т35 — 5/5,— Ю/ГВ) • 1-т 156. (У 9 — 71/ 72+ 6 /~ТТ25) ■ Vi- 156. (2]/f-8l/f+зк1 • 157. (3/1-5/30-2,/f ;)V1 " •
Иррациональные выражения It is?. (ey'f-s^s-nyAfV I j'/i. 158. (2/6-3 /5) (/3 +2/2). 158. (^9 —2 ^4) -(4^3 + »/2). 159. (^16-2^2 + 4^54)-(5^-3|/-i). 159. (|^25 + 1^Ш-^75)-(2^5-5^Г5). 160. + 16°. (5 “ 317-^ + 417 ^ ). (б |У| - ;736 - ,772). 161. /dJb •161. i/tiFb-i/ab*. 162. a2 V5x'\ V^x- 162- ~~ »/4?-a3 ?8x. 163. 2 ^25?-3^15?- 163. 5^12?.2/i&z\ 164. 3|/|.2 j/g 164. JI/'I-I VH »*• f v7? з« yH-_ >«5-5 УШ- «• “6. !®7SS-vW-^ ••'-^(SSr-^-Sr-w 168. . 168. (^6+j7g-/i6«) j/g. 169. (/« + |/4)-(/“J-1^4)- 169. (<! + ^ /ai). (a —2 j/|) . 170. (%/агЬ+ |/a&2)-(|/a— j/й). 171. /3.^2. 171. s/5-/2. 172. y'|.y/|. 172. |/| ■ J,7i 173. £ 54■ /6-^2. 173. ^2 . y/i.. jJ/|..
42 * Глава IX 74‘ ' Vi ■ V*' '№■ 174. 1/12 • }/~ ■ fti ■ /з. 75. (3 /10 — 2 /4 + 6 f/ 25) • /2. 75. (2/6 + 3 /15 — /Г6) • /12. 76. (2 /ТО + 3 /2 — 4 /5). /ТО. 76. (2/2 + 3/2 — 4 /2)-3/2. 77. (3 /2 + 4 /з)-(/2-2 /з). 77. (5 /3 — 2 /2)-(/2 —/3). 78. (6 /2 - /32) • /2 - 2 . 78. (/4-2 /б) • (2/2— 3 j/y). 79. /а3* • 179. ’/^ • /554 БО. Зд^/ЗАс-б^/гЛ. 180. 8а2£ /ЗдсГ2ас2 /2й*с. 81. а*\/~с№-Ь ^Уу /^-ай/^. 81. а /а4£3-аЬ8 /а&2• /а&‘• а • 82. (, "а2 - 2 /Г2 - о /ft®) ■ а2 /«Г 83. (/а2 —/а4+ а/а3)-(—2а/а2). 84. (/а — /а2 + /а4) • (/а — ’/а*). 85. /28:/7. 185. /45: /5. 186. р=. 186. 87-Г8:П- 187-/т:/Г 88. |г/Гб:з/|-. 188- 2/J;i/S- I. (5/4 — 6 /То +15 /Тб): 3 п- К (3 /6 + 2 /Т8 — 4 /Т2) : 2 /I- 90. /5а:/а. 190. /За2:/^. 191. /Г?:/2а2. 191. /2а2:/2а. 92. = 9S- /1 : \/Ч- 153- /I: VS- J4. (а^2 /а—х /Т>) : /блГ. 194. (2а6 /а* — а /й) : lf bx.
Иррациональные выражения 13 195. (ид3*3 — х\/а3 — 4 а у/ах1): у/ах3. 196. (2 3 + ^/1) : ^ J/??. 197. (^Д* - ^: (^д + s/b). 198. (¥^Ь — 2 ]/2W- +Ь\/4): (?/а - \Щ. 199. {У&^3 - Ь yWtf): (tf2a - УЩ. 200. (да ра + Ь* \/Щ: (,/^ + ^2^)- 201. (** j/*3 4- *у ^*V +У* j^/"):(**/* + ^*V +.У Уу)- 202. У 9: \/3. 202. \/8:^2. 203- /т: г)/3 204. (^6-2 i/З + |/б): i-i/6. 2 1 204. (31/2 — 12 $/12 + 10 f/2): -J ^2. О 205. l/д: ^я*. 205. У а3'- У а*. 206. j/402 : в,/2а1 206. ]у2а1: У 2а*. 207. l/fo® : р27^. 207. у/~ : ^40^. 208. 10а l/д : j/д2. 208. За : {/а*. 209. 6а2 УЗаГЧ>: 2а3 ^аГ1. 209. 2a3£ */<rV: 6а£! j/a^'7. 210. ох*у: У2оху1. 210. 2хгу3: У8х3у2. 911 24Д5/2-,5/^ If! -,3/5W 9,, 2а2Ь 3/а5р _ 4^2 8/^2 ги‘ tP У & ' Ь У cd>- и- с У М ’ сЗ у bW •212. (а2Ь + а**)■ |/~^3: а* |/^2. 213. (*+3')4^~3'i. 214. 215. {У 8a6b9 — ab У&№ + аЬ2 У^ь) : У2Ь . 216. (^8р — 31/3): (У2х — /з). 217. (2а У ах2 — а У ах3 — ах)т. (J/a2x — \fax). 218. {х~ У27ху3 + 2ху У2ху): (У3х3у +\/2 ху). § 8. Возведение корней в степень и извлечение из них корня. Для возведения корня в степень нужно возвести в эту степень йодко- ренное выражение. Предыдущее правило можно выразить так: при возведении корня в сте¬ пень показатель корня остается без изменения, а показатели подкоренного выражения умножаются на показатель степени.
14 Глава IX Если данный корень имеет коэфициент, то последний возводится в сте¬ пень отдельно и результат пишется коэфициенгом при самом корне. Возведение многочленных выражений делается по общим правилам воз¬ вышения многочлена в степень. 219. {Уа% 219. (/а4)7. 220. (/а2)8. 220. (/а8)5 221. (/2^)*. 221. (/Уё)а. 222. (-а УсРЬ'У. 222. (-а /о3!)'. 223. (а*х УЗа'х}. 223. (ах* /2а*5)*. 224. (- 2a j7i)‘. 224. (_| /§)’. 225. (/(^309)‘. 225. C/fc+yT)7'. 226 226. \ а~Ч* ) ’ V 227. (аг'Ь-* /4апЬ~‘)~2. 227. (аЧ^У^Щг3. 228. (У(х*+у*)т)”Р. 228. (/'(**—У/* ),пр. 229. (/3—/2)3. 229. (/5 + 2>. 230. (1+2/2 )8. 230. (2 |/3 -1)*. 231. (/4+] 2)9. 231. (/2-]/3)3. 232. (/3 — 2 /2)3. 232. (/2 + 3/З)3. 233. (/2 — /3 -f- V 6>. 233. (/2 + /5 — /То)3. 234. (3 /2 — 2 /5 — /10)3. 234. (5 /б + 3 /2 — 2 /з)3. 235. (|/1+/5 +|/з^1/5)г. 235. (|/7+ 2/6 +1/7-2/б)®. 236. (|/и+б/2—|/ll —б/2)8. 236. (|/l3 + 5/4,2+j/l3-5/4^ 237. (j/l 1 + 4 /7 - |/ll—4/7'у. 237. (j//7 - /3 -[//7 + /3~) *. 238.({^_^у. 238. (f 239. (а /а + а /2а)3. 239. (а [ 'В—2а \/Щ\ При извлечении корня из корня показатели корней перемножаются, а подкоренное выражение остается без изменения. Если данный корень имеет коэфициент, то обыкновенно прежде извле¬ чения из данного корня нового корня вводят этот коэфициент под знак радикала данного корня. •
Иррациональные выражения I 15 Извлечь корень: 240. У240. (/pp. 241. )/УТ1. 241. 242. УуТ25. 242. 243. 243. ||/^512?». 244. У а У а3. 244. ^ а \/а. 245. У a* ft?'. 245. ^ а3 ^. 246. |/¥а10ЬЧ\ ' 246. J/^alWs. 247. |/247. ^]/|^. 248.. j/л3 ^л: 248. j/*2 j/* ]/*. 249. у, j/f y/J- 249. у fj/jyf . 250. у/2х У 2хгу-3у1/3^. 250. }/ 2Y |/ Ъху-2ууШу. 251. ^20736 . 252. J/59049 . 253. ^4096 . 254. ^262144. § 9. Уничтожение иррациональности в знаменателе дроби. Для уничтожения иррациональности в знаменателе дроби нужно поды¬ скать простейшее из выражений, которое в произведении со знаменателем дает рациональное выражение, и умножить на подысканный множитель чи¬ слитель и знаменатель данной дроби. В более сложных случаях уничтожают иррациональность не сразу, а в несколько приемов, последовательно вводя множители в чл'ены дроби. Например, если в знаменателе дроби стоит дву¬ член \/a =t Vb, то надо числитель и знаменатель дробГг множить на коли¬ чество, сопряженное знаменателю, т. е. сумму надо множить на соот¬ ветствующую разность и наоборот. Уничтожить иррациональность в знаменателе дроби: 257. 259. 261. 264. а 1 а 255. —. Vb 256. m V m3 256. n Vn3' а 257. r^=. 'V ccl 258. m -\- n 258. m —n Vm—n Vm -\-n 4 Vi' 259. Д=. VS 260. 6 V8* 260. 6 VI2' V49 261. Vl2 262. a? — b* 233. a — b Vii' Ya — b" V a3 — bJ Va — Vb °64 ^a 265. a 265. a a-\-b *. ^D^r. . -• Va— Vb 1 — Va V a + 1"
16 Глава 1Х| 266. 268. .269. 271. 273. Уз - 266. 2+КЗ ' п Va-Vb ' 12 3-+-V2 — Уз‘ 1-\-ЗУ2 — 2УЗ У2 + УЗ + У6 * У 2 3 — У2 УеР~Ы-\-Уа1+ V У а? — № — Уа3-\- Ь* 287. 268. 270. 272. 274. 1 —а /l —Уа п Vfl+V* ’ 2 + Узй УЗ + Уб — УТ’ п / У2+У 3 * 1 Кг+Уз' 267. I — а l/l-H а 275. 47 2J/3—V3 § 10. Квадратный корень из двучлена вида Л — У В. Квадратный корень из двучлена Az*z\/B может быть упрощен в том случае, когда двучлен А* —В есть полный квадрат. Формула упрощения такова: / ’ i" .А Ул±)/е = }/- + 1?а -±/ По этой формуле решить следующие примеры: А - У А3— В 276. У 2 + /3. 276. /4-/7- 277. У 6 + 4]/2. 277. У 7 + 2/10. 278. У 5 —/2Т. 278. |/ 8-/15. 279. ]/ 7+4/3. 279. У 11+4/7. 280. ]/4/2 + 2/б. 280. |/ 5/5 — 2/30. 281. / 4|/5 — 2р'15. 281. |/з/7+ 2/14. 282. у'/м + б/! 282. У У124 — 32J15. 283. -|/17 + 6у/4 |/ 9 + 4J/2. 284. У (д + £) —2/яй. 285. У 2гг® + 2/ а4 — 286. 287. / 0,38 + 3j/0,0091. 288. У За + 2а/2 +1/ За -Г 2а]/2.
//Гб № Иррациональные выражения 17 § 11. Задачи на все действия над радикалами, з 1 — 1 . I 289. 290. 290. 291. 29'2. 5— У 5 3+ VT 5 4 2 4-^lT vn—Vl 3+ VT 9 . 22 1 5— V~7 7 + Vb V7+V5 1 1 289. 5+1'5 3-1/5* 7 + 41/ 3 7 — 4 V 3 Vl2 (1+ V 2)(V 6— V 3)' 3—V!> 3+^5 a i /«/-r; (Уз+Уьу (Vs-Vbf i ?94‘ 5 + v'ls + 7 — I7 29* 5.1 6 291. 292. 293. 294. 1 У 5 — 1 , Iх 5 + 1 l/5 + l V5~ l‘ 1 2+1/3 4 + 2 + ]/3. 3 31/5 — 5 7 К 5’ 16 4— V7 7 + Vl3‘ 4—Til 3+1/7 1/7 — 2 l/ a V x V a — V x V aV x 14 3 296. l/2—l 1/2 + 1/ 1/2 + 1 Кгуч" v&99- (^+^НтгЬ+1) 400 a */<z2 — ■** a— l/a2 — *3 a — l''я2 — & a + V(fi — jc2 301. Ц\/4^/63- 10 ^/3/7— 31/6|/28. , _ ^ . - 302. |/A-]/ jcj/x — 2]/+ 3^/дГ5^ — 4,t2v/лг|/~ 303. д + 2+ l/g2 — 4 , a + 2— l/g2 — 4 g + 2— ^2 — 4 *" с + 2 + Va% •—4* 304‘ [wi-5^:(v/l"r,,^),]-V/'T“,l/^ 3»5- (1+1/Л1):(1-/Ш)- 306. 5u|/aj/aj/g— 2|/а3^+ 3^T«^ —4a*jp/a j/^. 3 Сборник алгебраич. задач, ч. II,
18 Глава IX 307. ( - 4af/a-у о*)* + ( - 1 0а\/ x\/'^j ~[5( \/ a\^ ] 308. (j^I^'V*)6} зоэ Г -- /(l — a)s/i+a . , V W П-1 . ,3/ За Va >• []/■■ ]У=|7 j 310. (р^==+1/Г=1^:(^Т^Р+1> Я11 / g^ + "8 -./" 2/гх ^ . i*/~ -2 \]/d2«JC—алз I * У'ял:—ri-- Vax) V an~ ° »«■ y^M^V^+iwi- 313. 2+^ ■ 2-,/3 V2+/2 + V 3 т'г —/2 — Vх з* Определить частные значения выражений: 14-х . I—х 1/3 314. , - Ч . при x=f 1 + VI + х 1— V\ — * 2’ Jf+1 , дг—1 2 315. > —:=Ч 7=5=- ПРИ ^ = -7=- Х + l^ + JC х— Vx3—X V о з17* при *=4v/t-1* § 12. Степени и корни с дробными показателями. Количество с дробным показателем представляет коречь. показатель ко¬ торого равен знаменателю дроби, из того же количества в степени," указываемой числителем дроби. 3 __ Я» Так, д*=|/а3; вообще ап = Уат. Корень с дробным показателем равен степени, показатель которой об- ратен показателю корня. 3 2 ” _ — Так, 'у/~а — а3\ вообще \/а = ат. Действия со степенями и корнями, имеющими дробные показатели, производятся по тем же правилам, какие выведены для степеней и корней с целыми показателями. »
Иррациональные выражения Заменить радикалы выражениями с дробными показателями: 318. У&. 318. 319. fir*. 319. ^г5. 329. ^г*А*. 320. УаЯг*. 321. /аГ*. 321. 322. 322. \/!?~ЪК 323. 323. 324. 325. mp+yap+t. 326. j/^. 327. Г/ 1 у>* 0'‘1* |/ ^н* 328. ^ в^й. 328. ^ х1/у. 329. ^ а4 + л:а— 2ojc. 329. у/а1 + 2ай + й3. Заменить выражения с дробными показателями радикалами: 5 2 3 з 330. а6. 330. а3. 331. с 4. 331. <Г?. 2 5 332. (а + £)3. 332. (а-£)3'. L 3 _ 1 333. За2(а —£)3. 333. 4а *(а + 6) 3. г *—1 334. (4а) *. 334. а*. 335. (а + й)0’25. 335. (а — 336. яГ*\ 336. (^)~0Л”> 337. хг«". 337. 1 338. е“-. 338. е *. 339. п2,5. 339. Вычислить: (а — *« ,—0,6* 11 3 5 340. 4*. 340. 273. 341. 814. 341. 164. 5 _4 2 4 342. 16 4. 342. 32 *. 343. ( — 8)*. 343. (—27)3. 344. (I)"’. 344. (f) 3. 345. (-4)*. 345. (-lg) 61\ 3 346. (0.64)0,5. 346. (0,027)3. 347. 81~0,75. 347. Ю24“0,в. 2 1J_ 133 348. 83 — 164 + 92 . 348. 253—273 + 814. 349. 16°’5 + (^)~°,75-(-j)~e* 349. 9-°'*-8''1»+(0125)“^
20 Глава IX Произвести указанные действия: 2332 5223 350. а3Ь3- аАЬ3. 350. А А А 3 -1 А 3 5 351. аЛ2Ьб :а3Ь4. 351. аЩ'3ш.а3Ьб. 352. (а2 - Ь2): (а2 - Ь2). 352. (а 2 + Ь2): (а2" +Ь2). 353. (а2 + аАЬА + Ь2) : (а2 — аАЬ* + Ь2). (Зя Зж\ /я _ я \ а2 + Ь~ 2) : U2 + Ь 2). (А А 2 AN / А 1 А 2 \ а3 + аэЬ3 + 16й3) :\а3 + 2а3Ь3 +4 Ь3). 356. {а2 - Ь2 - с2 + 2 Ь* с*): {а4 + Ь4 - с*). С з ± j_ з\з / г_ ^ 1 А\г 357. U2— авЬ* + Ь*). 357. [а*+а*Ьу—Ь*) . { А A A L\3 / А А 3 _2\3 358. {а2Ь3—2а3Ь4). 358. \а3Ь2 —За*Ь*) . 359.[(я-ии-н.и-4* 359. [(оЧГ1) .(а^Ь-1)2 .(ь*)"*]. 3 3 360. ~ Ь-л—-д3д~^2 . 361. Уа2Ь 2-6в'*6 3 + 9й3. 2 .2 a — b °Л/<2-268 >/2a6fc-3 з/ 4_ А 362. J/ Т-. 363. J/а3+я-2а3. (|/с-®6з)15 § 13. Мнимые числа. Корни четных степеней из отрицательных чисел, рациональных и ирра* циональных, называются мнимыми. В противоположность им числа по¬ ложительные и отрицательные называются действительными, или ве* щ е с т в е н н ы м и. Простейшее из мнимых чисел есть У — 1. Принято обозначать его буквой i, так что ]/ — 1 =i. В алгебре показывается, что г — —1. Отсюда j возведя i в последовательные степени, получаем: i1 —/; г* = — 1; х® = —*; i*= + I.
Иррациональные выражения 21 При дальнейшем увеличении показателя те же четыре результата по¬ вторяются периодически. Вообще оказывается, что всякая степень от i с целым и положительным показателем равна степени, показатель которой представляет остаток от деления данного показателя на 4. Так: j»6 _ £3 _ J. £33 _ £3 _J £ Всякое мнимое число вида |/ — а может быть представлено в виде произведения действительного количества на /, именно: ]/— a = i\/а. Подобное выражение мнимого числа называется нормальной его формой. Для производства действий с мнимыми числами нужно привести их сначала в нормальную форму. Выражение вида а + bi, где а и b суть действительные числа, называется комплексным числом. Оно становится действительным в случае Ь = 0. Два комплексных числа вида а + bi и а — bi, т. е. такие, которые от¬ личаются только знаками при мнимой части, называются сопряжен¬ ными. В теории действий с комплексными числами часто встречается число |/я*-(-й*. Оно называется модулем комплексного числа а -}- Ы и обозначается обыкновенно через М. При производстве всяких действий с комплексными числами нужно приводить предварительно мнимые части их к нормальному виду. При сложении и вычитании комплексных чисел отдельно складываются или вычитаются их действительные части и отдельно мнимые части. Так: a + bi± (at + Ь$ = (а± a,) + (bdz bt)i. Умножение комплексных чисел совершается по общим правилам, причем только принимается во внимание, что к* = — 1. Поэтому: (а + bi) (ах + bxt) = aat + atbi + abj — bbt = (aax — bb^) -}- (avb + abx)i. Деление комплексных чисел выполняется '"посредством умножения дели¬ мого и делителя на выражение, сопряженное с делителем. От этого новый делитель становится действительным, именно обращается в квадрат модуля прежнего делителя. Таким образом: . (а + Ы)-(а \ ЬЬ-=1а + ЬГ) {ах ~ h0 - ggl + bhi I 'аф ~П-< i Возведение в квадрат и в куб комплексных чисел делается по формулам для действительных чисел. Применяя эти формулы, полезно сначала только обозначать степени мнимого I, а потом уже заменять их простейшими выражениями. Таким образом: (а + bif = а* -\- 3cPbi -f- ЗаЬ^Р + 63/3 = а? — 3ab* + (Зя®й — 63} г. Извлечение квадратного корня делается по формулам: где М обозначает модуль подкоренного комплексного числа. Полученному корню можно приписать или те знаки его действительной или мнимой
22 Глава tX частей, какими они являются по этой формуле, или знаки противопо? ложные. 365. (/^Т)6. 366. (/^Т)31. 367. (i ^Т)7. 368. (/^Т)56. 369. ilu. 370. i37. 371. i13. 372. iin'\ 373. iln 3. 373. i*»-*. 374. i8eh*. Упростить мнимые выражения: 375. /"—"4. 376. / — 81. 377. j/^3. 378. 379. 380. [/--• 381. /^a. 382. / —9лг. 383. /— а2 — fi2. 383. у' — (a — й)а. 384. / — л2—_у2+2лу. Произвести показанные действия: 385. /—25 + /—49 — / — 64 + /^Т. 385. /—144 —/ — 81 /~Т + /— 9. 386. 3/^4 + 5/ —27~—З/^ТбГ—5/^3. 386. 10/ —25 — 5/^8 +/^49"—2/^2. 387. 3 + 21 + (4 - 31) - [(8 - 51) - (5 + 131)]. 388. a -f Ы — (2а—Зй1) + [(а — 4й1) + (5а — 2£*')]. 389. /—16 -/^9. 389. /^8-/^2. 390. /—"а - /^й. 390. /—/и*/^я. 391. 1/ —л2. 391. —г/— у2. 3Q2. /а — 6 ./ТП^Г. 393. (2 - 51) (8 - 31). 394. (5 + 2/ — 7)• (6-5 /^7). 395. (/а-/^й) (/а + 3/^Т). 396. (3 /^5 - 2 /^7).(2 /^7 + 3 У^5). 397. а:/ —а. 397. а!:/ —а. 398. /—ал:/—л. 399. 399. 400. а — bi* a + W* ' л—у!" 400. 401- %=' 402. х+У* 1+У-З 3+5гГ8 ,по 36-У=2 лпо 5 — 29г V5 лпя 2— V^7 403. -=. 403. 404. !=• 2 + 3iV2 7 — 3 У—5 3+ У—21 405. (а + й1)г. 405. (a — bif. 406. (З —/^2)а. 407. (-+ 2~3)%- 408- (3/—5 + 2/—Т)3- 409. (2-3 /^2>г.
функции и их графики 23 410. (—--+2- -)*. 411. (я-й/)3. * 411. (n + fti)3. 412. (3 + ]/^2)3. 413. (]/--3 — 2j/^"l)s. 414. (~--+2 415. 173 + 4 |/—Т. 416. |/ — 3 — 4/. 417. /l + 4 ]/^з. 418. У2 — 3^/^5. 419. ^20 —4]/^ТГ. 420. ]/б+ ]/^ЛЗ. 421. *V~1. 421. V'—if—i. 422. ГЛАВА X. ФУНКЦИИ И ИХ ГРАФИКИ. 1. Построить точки с координатами: (5. 8); (-9,3); (-6,-10); (12,-7). 2. Построить точки с координатами: 04- 4М-18- 4М-4- -4);(2'7- -эд- 3. Какие особенности есть в расположении точек: (12, 0); (—7, 0); (0, 0); (0, 9); (0,-17); (0,—8,6). 4. На какой линии лежат точки, координаты которых равны между со¬ бою и по величине и по знаку? Как расположена эта линия? Какие углы она образует с осями координат? 5. На какой линии лежат точки, координаты которых равны между собою по абсолютной величине, но противоположны по знаку? Какие углы она образует с осями координат? 6. Построить прямолинейный отрезок, соединяющий две точки: (10, 13) и (-14, —3); (-5, 7) и (3, —9). 7. Построить треугольник по координатам его вершин: (6, 12); (9, 1); (-6, 5). 8. Построить четырехугольник по координатам его вершин: (-4, 9); (12, 7); (3, -3); (-10, -6). 9. Построить графики: у = 3х- 2лг = 5у; Зл:—2у = 0; 4х — 7у = 0. 10. Построить графики: х+у = 2; х—_у=1. 11. Построить прямую у= 2х -J- 5 и найти точку пересечения ее с осью ординат.
21 Глава X 12. Построить прямую у = 3х — 7 в найти точку пересечения ее с осью абсцисс. 13. Построить прямую _у= —лг + 3 и найти точки пересечения ее с осями координат. 14. Построить прямую у — х— 2 и найти точки пересечения ее с осями координат. 15. Построить графики: лгу=8; лгу=1. 16. Построить графики: ху=—5; ху =—9. 17. Построить графики для перевода дюймов в сантиметры, зная, что 1 дюйм = 2,54 см. 18. Построить график зависимости длины окружности от радиуса (при¬ нять т; = 3,14). 19. Средняя скорость движения поезда равна 35 км в час. Составить таблицу числовых значений пройденного поездом пути за различные про¬ межутки времени и построить график этой зависимости (указание: принять единицу масштаба по оси У равной единицы масштаба по оси X). 20. Построить график уравнения равномерного^ движения s = sB-{- vt, принимая s0 = 2 см и v = 3 см/сек. 21. Изобразить графически формулу: 1=606,5 + 0,3051, выражающую зависимость количества тепла водяного пара ). от темпе¬ ратуры t. 22. По закону Фарадея количество Q вещества, отложившегося при прохождении тока, прямо пропорционально электрохимическому эквиваленту а, силе тока 7 и времени t. Построить график этой зависимости для меди [а. = 0,328) при постоянной силе тока 7=24. 23. Опытным путем получена следующая таблица для скорости звука ■о м/сек в сухом воздухе при различных температурах С: fc — 30 -17 — 5 0 8 12 20 30 v м/сек 313 321 329 332 337 339 344 349 Составить приближенную линейную формулу зависимости v от t. 24. При испытании электровоза получена следующая таблица зависи¬ мости между силой тока 7 (амп.) и тягой Р (кг): 7 амп. 65 86 106 116 137 150 Р кг 160 360 560 660 850 980 Начертить график зависимости Р от 7; заменить этот график прибли¬ женно прямой линией; составить приближенную линейную формулу зависи¬ мости Р от 7, найдя угловой коэфициент и начальную ординату по изме¬ рению на чертеже. *
функции и их грпфики V 25. По закону Ома где / (амп.) — сила тока, V (вэльт) — на¬ пряжение тока, R (сек/см) — сопротивление проводника. Считая R постоян¬ ным, построить график зависимости I от V; считая V постоянным, пост* роить график зависимости / от R. 26. При температуре в 14°С и давлении в 5 атмосфер газ занимает объем в 6 л. На основании закона Бойля-Мариотга построить график изменения объема массы этого газа в зависимости от изменения давления. 27. Решить графически систему уравнений: 30. Пешеход вышел из города в дерев то в 12 час. дня и шел равно¬ мерно со скоростью 3 км в час. В 14 час. того же дня из города отпра¬ вился по той же дороге другой пешеход, проходя по 4‘/2 км в час. Когда и на каком расстоянии от города второй пешеход нагонит первого? (Ре¬ шить графически.) 31. Себестоимость К в копейках 1 м сварочного шва при толщине d миллиметров листа можно приближенно вычислить по формулам: Начиная с какой толщины листа сварка водяным газом будет стоить де¬ шевле, чем кислородно-ацетиленовая? 32. Велосипедист выехал из города в 9 час. утра и ехал со скоростью 12 км в час; в 10 ч. 30 м. выехал по тому же пути автомобиль со ско¬ ростью 48 км в час. Через 15 мин. после отъезда автомобиль остановился на 15 мин., после чего пошел со скоростью 30 км в час. Определить гра¬ фически место и время, когда автомобиль нагонит велосипедиста. (Принять масштаб по оси У равным ^ единицы масштаба по оси А’.) 33. Из Москвы в Курск, расстояние между которыми 530 км, в 6 час. утра отправился автомобиль, который шел безостановочно все расстояние со средней скоростью 50 км в час. Другой автомобиль вышел по тому же пути в 4 ч. 30 ы. утра и шел со скоростью 65 км в час. Пройдя 162 км, этот последний автомобиль имел остановку до 8 ч. 30 м., после чего пошел со скоростью 40 км в час до Орла (380 км от Москвы). В Орле автомо¬ биль пробыл 1‘/2 часа и затем пошел со скоростью 90 км в час. Начертить ( х +у = 7; 1 х—у=3. 28. Решить графически систему уравнений: 29. Решить графически систему уравнений: |>= — Зх — 9; К— 1,5 + 0,95if — при кислородно-ацетиленовой сварке, ЛГ=4 + 0,6й — при сварке водяным газом.
2а Глава XI график движения обоих автомобилей. Где и когда автомобили сошлись? Который автомобиль прибыл ранее в Курск и на сколько времени? (Принять масштаб по оси Y равным ^ единицы масштаба по оси X.) ГЛАВА XI. КВАДРАТНЫЕ УРАВНЕНИЯ. § 1. Решение буквенных квадратных уравнений. Приведение буквенных квадратных уравнений к простейшему виду и решение их выполняется теми же приемами и по тем же формулам, какие были указаны для квадратных уравнений с числовыми коэфициентами (см. 1-ю часть задачника). Решить неполные квадратные уравнения: 1. ах* + Ьх = 0. 1. — — £ = 0. а о 2. (л:-а)2 + (л:-5)а = а2 + йа. 3. и2*2 + тх = п&у? + пх. 3. и*х2 — т?х = /и*х2 — «2х. А *+«., х — а а(3х + 2а) 4. х + Ь х—ft 4х® х — а х +а “ л'2 —а* • х — а х + а х2 — а2 ’ 5. х2 — 9а-Ь*. 5. х2—16а‘с2 = 0. 6 а 6. 2х -f а а а х — а а 2х + а" - x-f« а—х x + ft х — b ' 8. 1 + 2x-f х2=, П[1+Г?- + п^ 7. )• х—2а x-f 2а ft —X =х+Ь' QaX — У* а +1 a -f-1 ах 9. ах — 3 а _ а + 6 ах + 3 * 10. ах2— bs = а3 — Ьх\ 10. , а2х2 + 5*=а4 + й2х2. II 2« + ft + х jc — 2<г-f-& |0 х2 2ах х 1 ’ х + 2а — Ъ~2а + Ь — х‘ х2_аз~ + (х + а)2 — ах~ 1Г^а ' Решить полные квадратные уравнения: 13. ха— 4ах + 3а*=0. 13. ха + 8ах + 15а2 = 0. 14. д:а + 2а3х — 35J6 == 0. 14. д:2+ 6о2лг — 27а4 = 0. 15. Xs — 2ах-fa2— йа = 0. 15. л:® — 2Ьх — а2 + йа = 0. 16. х2 +2йх— a* + 8ab — 15йа=0. 17. 2л:2 — Зал: — 2д2 = 0. 17. 4х2— 20ах + 9а2 = 0. 18. 6х2 + 5ах-f а2 = 0. 18. 8х2+2ах—3а2 = 0. 19. Зй2х2 + \Qabx + За2 = 0. 19. бЛс2 — ЬаЬх — 6а2 = 0.
Квадратные уравнения 27 20. 20ft2.*2 — 9abx — 20а* = 0. 20. 24йалг2 + 14аЪх— 3я2 = 0. 21. (/ял: +л)(л* — лг) = 0. 21. (л — тх) (пх + т) = 0. 22. яй(л;2 + 1) — (a2 + fc2)л: = 0. 22. ax(bx — a) — c(a — bx) = 0. 23. Ьх* — а = (а — Ь)х. 23. (л — ft) *2 + 2й = (л + й) д:. 24. (я2— ft2)*2-fflft=(fl2 + ft2)jt. 24. (л2 - fta) д:а - яЬ = (а2 + ft2) л:. 25. *-! = £-!. 25. * + ± = £=* + £+* х о а х а-\-Ь ' а — Ъ па ^ I ^ -**— ^ пд ^ х 7 26. —+ —-т^- 26. ' 27. 1. 27. *-±*£=£=1. „О Ч.-Г-.1/ с ■» -то Оо Л + 66 | а — 6ft 6ft ^•..^9», 9/,—„• ^ jc + 36 + 3ft^“ я • 29- i+iTi+»-T2;=0- "■ ОЛ Х t 91 1 1 с 1 I ^ йи’ дг + л^д:— л-4(л«—в*)’ а+Ь+х~ в + ft + * * о0 а(х — 2) , л Л Ь*\_Ъ{х-2) Ь "TftA* а*)~ а 33. (а + Ь)(а — Ь)х^ — аЬ(2ах — аЬ). о„ „ сх 2 с2 п 0_ 2a + ft — х а х + Ь 34. *-^-^+^ = 0. 35. 2&qra—^—т F+g~ од 4я + 3ft— х 2я + ft 2a + 3ft + * 4ft + Зл — х — 2ft + л ’ 2ft + 3л-fx ' о- x + a x + b_a ft оо 1 , 1 , 1 „ 6Ъ 7=Ъ + 7=Ъ-Т + а * 38. + + чп л + ft— х_а — с + х лп (a~x)(a — b)-\-(x — bp _49 а-Ь-х~а — с — х' Чи* (л — *)2 + (2л:-л — ft)(jc— b)~ 19* а + с(а+х) L« + *__ а а + х Ь х lO- х + л дг + ft il. х—а л:— ft fi + 4ft •О *3* 1 4ft дг-|-2& д: —2ft а ' -Ч- 1 + 1 а + с (а —х) х а — 2едг ’ 42. £±f+ £±* + £±£=3.' 43. ^-U*(l+C-). д: —л 1 л:—& 1 л: — с ап v 1 1 44. х2 + алх = яг"д: + я*п. 45. jc2 j/л = л |/2л + (л — л]/ 2)дг. § 2. Свойства корней квадратного уравнения. Корни приведенного квадратного уравнения л:2 + рх + q = 0 бывают дей¬ ствительными и различными при условии действительными и рав¬ ными при условии /?2 = 4у и мнимыми при условии p*<^4q. Подобным же образом корни общего уравнения яле2 -\^.Ьх + с = 0 дей¬ ствительны и различны при условии ft2 4<зе, действительны и равны при условии й2=4яс и мнимы при условии 6а<^4яс. Не решая следующих ква¬
28 Глава XI дратных уравнений, определить, какие из них имеют действительные различ¬ ные, действительные равные или мнимые корни: 46. х® + 6х + 5 = 0. 46. х2 — 6л:+ 8 = 0. 47.*х® — 1 Ох + 25 = 0. 47. л:®— 14* + 49 = 0. 48. х* + 4х + 5 = 0. 48. л:® — 9* + 20 = 0. 49. х* + 8х + 25 = 0. 49. х* + 11л: + 130 = 0. 50. х® + 2х — 120 = 0. 50. х® + 3х—180 = 0. 51. х* + 24х+144 = 0. 51. л:® + 30л: + 225 = 0. 52. 12л:® + 7л:—12 = 0. 52. 9л:®— 12х + 4 = 0. 53. 4х®—4х+13 = 0. 53. Зх® + 12х + 13 = 0. 54. 25х® + 30л: + 9 = 0. 54. 9х® —42х + 49 = 0. 55. 2л:®—18л:+ 65 = 0. 55. 36х® + 48л: + 61 = 0. В приведенном уравнении сумма корней равна коэфициенту р, взятому с обратным знаком, а произведение корней равно коэфициенту q. В общем уравнении сумма корней равна отношению коэфициентов ~ , взятому с обратным знаком, а произведение корней равно отношению с коэфициентов —. Пользуясь этими замечаниями, можно определить знаки действительных корней. Не решая следующих уравнений, определить знаки корней их. если последние действительны: 56. х® — 8х+15 = 0. 56. х® + 9х+14 = 0. 57. х® + 4х— 3 = 0. 57. х®— 2х—15 = 0. 58. х®—17х —60 = 0. 58. х® + х —42 = 0. 59. х® —5х+ 130 = 0. 59. х® +7л:+200=0. 60. х® — 26х+ 169 = 0. 60. х® —34х + 289 = 0. 61. х® —Зх — 460 = 0. 61. х® —Зх — 340 = 0. 62. 2х® + 5х + 2 = 0. 62. Зх® —7x+2 = 0l 63. 6х® —5х —6 = 0. 63. 9х2 — 24х — 20 = 0. 64. 4х®+2х+ 1=0. 64. 9х® + 3х+1 = 0. 65. 8х® + 4х—1=0. 65. 26х® —ЗОх—1 = 0. Квадратный трехчлен вида х® + рх + q всегда разлагается в произведение (х — xt) (х — х2), где xt и х2 суть корни трехчлена. Трехчлен вида ах* + Ьх + с разлагается в произведение а (х—xt) (х —х4), отличающееся от предыдущего множителем а. Разложить трехчлены в произведения: 66. х® —7х+12. 66. х® —9х+18. 67. х* + 3х—108. „ 67. х* + 5х—204.
гКв’адратные' уравнения 29 68. 6дга + 5лг— 6. 68. 15л:9 + 34л: + 15. 69. 30л:9 + 37л:+10. 69. 21л:2 + 22л: —8. 70. л:9 —6л: + 11. 70. л:2 —9л:+21. .71. ла+15^ + 44. 71. л:2—10л; + 22. 72. Xй —ах—6я9. 72. л:9 + ял:— 2я2. 73. аЪх* — 2ял: + я9 — й9. 73. (я2 + й2) л;9 — 2й9л: + й9 — я2. 74. л:9 — ах—а\Ь — й. 74. л:9 + |/йл:— я9 + я ^й. 75. яйле9 — 2я f/ab-x + я9 — й9. 75. я2й9л;9 — 2яй9 /й • л: + й3 — я3. Пользуясь связью между коэфициентами и корнями квадратного уравне¬ ния, можно составлять уравнения по данным корням их. При этом уравнение составляется в приведенной форме. Если же коэфициенты полученного уравнения оказываются дробными, то, уничтожая знаменатель, получаем уравнение в общей форме. Составить квадратные уравнения по данным корням их: 76. 2 и 3. 76. 7 и —5. 77. —4 и 6. 77. —8 и —5. 78. —5 и 0. 78. 8 и 0. 79. 3 и —3. 79. —7 и 7. on 1 1 on 2 1 oi 2 3 01 3 7 80. -g- и — 80. g- и у. 81. — -д и — -g-. 81. у и у. 82. Уб и — >/3. 82. |/2 и— >/б. 83. 4=ty,3. 84. —3 =t У—13. 85. lrhy''— ю. 86. За, —2b. 87. 2я — й, я — 2й. . 88. —у, -J. 89. а±Ь. 90. у, ~. qi я ^ 1 оо _ ~а + Ь’ * 1— а’ 1 — Ь' 93. я=Ь 94. y'ai |/^й. 95. Составить уравнение, корни которого были бы обратны корням уравнения x* + px-\-q — 0. 95. То же самое для уравнения вида ялг9 + йл: + с — 0. 96. Составить уразнение, корни которого были бы в т раз более кор¬ ней уравнений ах“ + йл: + с = 0. 96. То же самое для уравнения вида л:9 + рх + q — 0. 97. Составить уравнение, корни которого были бы на g больше корней уравнения х* + рхq = 0. 98. Составить уравнение, корни которого были бы сумма и произведе¬ ние корней уравнения ял:9 + йл: + с = 0. 99. Выразить сумму кубов корней уравнения ял;2 + йл: + с = 0. 100. Выразить неполный квадрат суммы из корней уравнения ■*а + рл; + <7=0.
so Глава XI 101. При каком значении Ъ уравнение + Ъх + 25 = 0 имеет равные корни? 102. Показать, что если дискриминант квадратного уравнения ах1 zt 6 + с = 0 равен нулю, то левая часть этого уравнения есть полный квадрат. 102. То же самое для уравнения вида xtdzpx + q = 0. КЗ. При каких положительных значениях с корни уравнения Зх*— — 18лг + с = 0 действительны и при каких—мнимы? 104. Решить уравнение ах* + Ьх = 0 по формуле полного квадратного уравнения. 105. Решить уравнение ах*-\-с = 0 по формуле полного квадратного уравнения. 106. В уравнении х* — 6лг + ^ = 0 определить то значение q, при кото¬ ром корни уравнения а и р удовлетворяют зависимости За+ 2^ = 20. 107. В уравнении д;2— 5х + ^ = 0 определить то значение q, при ко¬ тором корни уравнения аир удовлетворяют зависимости За+ 5,3 =17. 108. Какая зависимость должна существовать в уравнении х‘ +рх + q= О между коэфипиентами р и q, чтобы один корень этого уравнения был в т раз более другого? 109. В формуле решения квадратного уравнения х* -\-рх -\-q = Q уни¬ чтожить иррациональность в числителе дроби. § 3. Составление буквенных квадратных уравнений. 110. Найти два числа, произведение которых р, а частное q. 111. При делении одного числа на другое в частном получается а и в остатке Ь; произведение этих двух чисел равно с. Найти эти числа. 112. Разложить число а на два множителя, разность между которыми равна Ь. , 113. Найти два числа, сумма квадратов которых S, а отношение p:q. 114. Сумма правильной дроби с обратной равна а. Найти величину дроби; определить при каких значениях а задача имеет решение. 115. Одна часть суммы в а руб. приносит ежегодно b руб., а другая с руб. прибыли. По скольку процентов прибыли дает каждая часть, если со второй получается одним процентом больше, чем с первой? 116. Отрезок длиною а разделен на две части, из коих одна есть средняя пропорциональная между всем отрезком и второй частью. Опреде¬ лить длины частей отрезка. 117. Разделить прямолинейный отрезок длиною а на такие две части, чтобы удвоенная площадь квадрата, построенного на одной части, равнялась площади прямоугольника, измерениями которого служат другая часть и сам данный отрезок. 118. Периметр прямоугольника равен 2р, площадь равна S. Определить стороны прямоугольника; исследовать решение и выяснить, при каком со¬ отношении между р и S прямоугольник обращается в квадрат. 119. В равнобедренном треугольнике боковая сторона равна п, а вы¬ сота менее основания на т. Найти основание и высоту равнобедренного треугольника. 120. В прямоугольном треугольнике из вершины прямого угла опущен на гипотенузу перпендикуляр. Определить части, на которые этот перпен¬ дикуляр делит гипотенузу, если .длина перпендикуляра h, а гипотенузы — с.
Квадратные уравнения 31 121. Если радиус круга увеличить на а сантиметров, то площадь круга увеличится в п раз. Определить радиус круга 122. По сторонам прямого угла от его вершины в одно и то же время начинают двигаться два тела. Скорость одного Vt сантиметров в секунду, скорость другого Vt сантиметров в секунду. Через сколько времени рас¬ стояние между этими телами будет равно d сантиметрам? 123. Равнодействующая двух сил, приложенных к точке под углом в 120°, равна Р\ отношение этих сил /к:п. Определить составляющие силы. 124. Электрическая лампа в т свечей висела над столом. Когда она перегорела, ее заменяли лампой в п свечей и, чтобы степень освещения не изменилась, опустили на h сантиметров. На какой высоте над столом висит теперь лампа? 125. Два магнита А и В находятся на расстоянии d сантиметров друг от друга. Магнит В в п раз сильнее магнита А. На каком расстоянии от магнита А находится точка, в которой притяжение обоих магнитов одина¬ ково, если известно, что сила притяжения магнита обратно пропорциональна квадрату его расстояния от притягиваемого тела? 126. Две водопроводные трубы с диаметрами а и Ь надо заменить одной трубой с той же пропускной способностью. Каков должен быть диаметр этой трубы? 127. Камень падает в колодец, и звук его падения наблюдатель услышал через п секунд после начала падения. Найти глубину колодца, если ско¬ рость звука равна 330 ж в секунду и ^—9,8м в секунду. 128. Из прямоугольного куска жести, измерения которого а и Ъ, тре¬ буется сделать открытую коробку так: 1) чтобы площадь стенок равнялась площади дна, 2) чтобы площадь дна имела заданный размер 5. Выяснить условия годности корней. 129. Железнодорожный путь имеет длину а километров. Если увеличить скорость поезда на b километров в час, то поезд будет затрачивать на прохождение пути на с часов меньше, чем теперь. Сколько времени затра¬ чивает поезд на прохождение всего пути и какова его скорость? 130. Два автомобиля выезжают одновременно из места А в место В. Один из них проходит в час на т километров больше, чем другой, и по¬ этому приходит в место В на п часов раньше другого. Расстояние между А к В равно Р километрам. Сколько километров проходит в час каждый автомобиль? 131. Два прокатных стана могут, работая одновременно, прокатать а килограммов железа в Т часов. Если будет работать только первый из станов, то на прокатку указанного количества железа потребуется на t часов больше времени, чем при работе одного второго стана. Во сколько времени может прокатать а килограммов железа каждый из станов, работая отдельно? § 4. График квадратной функции. Графическое решение квадратного уравнения. График уравнения у = ахга есть парабола, касающаяся оси X в точке 0 и симметрично расположенная относительно оси У. Если я ]> 0, то пара¬ бола у~аха проходит по ту сторону от оси X, по какую лежат положительные
Глава X! значения ординат; если д О то, наоборот, парабола проходит по ту сто-J рону оси X, по какую лежат отрицательные значения ординат. х2—х—6=0. у хК у=х + 6. у Черт. I. Черт. 2. I. у=х2; y—x-j-2. II. у=х2; у=х\ III. у=х2] у=х— 1. График уравнения у = ах* + Ъх + с есть парабола, ось которой парал¬ лельна оси У и ветви которой напра¬ влены в положительном или отрица¬ тельном направлении оси У, в зави¬ симости от того, будет ли а^> 0 или а<0. Можно двумя способами графически решать квадратное уравнение. Первый способ. Построив по точкам параболу у — ах? + Ьх + с, определяем корни уравнения ах? + Ьх + с= 0 как абсциссы точек пересечения параболы с осью X. Если парабола у — ахг Ьх + с пере¬ секает ось X в двух точках, то урав¬ нение ах* + Ьх + с = 0 имеет два раз¬ личных действительных корня. Если парабола касается оси X, то уравне¬ ние имеет два равных действитель¬ ных корня. Наконец, если парабола не пересекает оси X, то уравнение имеет два мнимых (сопряженных) корня (черт. 1).
Квадратные уравнения Второй способ. Перепишем уравнение ах3 + Ъх + с = 0 в виде ах'1— —Ъх—с. Полагая ах3—у, получим систему: у=ах3\ v = — Ъх — с. 33 Так как корни этих уравнений должны быть одни и те жэ, то они должны представлять собою координаты точек, лежащих о повременно на обоих графиках этих уравнений, т. е. должны быть координатами точех пересечения этих графиков. Построив на одном чертеже график уравнения у —ах3 и график уравнения у — —Ъх — с, найдем координаты точек их пересечения (черт.' 2). Если парабола у = ах3 имеет две общие точки с прямой у = —Ъх — с, го уравнение ах3 + Ъх + с = 0 имеет два действительных различных корня. Если они имеют одну общую точку, то квадратное уравнение имеет два равных действительных корня. Наконец, если они не имеют общих точек, | то квадратное уравнение имеет два мнимых (сопряженных) корня (черт. 3). 132. Построить графики функций: у — х3; у = ^х1; _у = 2х3. 133. Построить графики функций: у — —лг3; у= —У~ —®х2- 134. Построить графики функций: у — х3+ 1; у = х3 — 2. 135. Построить графики функций: у = х3-\-Ъх; у = х3 — 4х. I 136. Построить график функции: у=х3 — З.х+2. I 137. Построить график функции: у = х3-\-х— 12. I 138. Построить график функции: у = х'1 — 4х + 4. I 139. Построить график функции: у — 2х3— 2х + 3. I 140. Построить график, зависимости площади квадрата от его стороны. I 141. Построить график зависимости площади круга от радиуса. I 142. Решить графически уравнение: х1 — Злг = 0. у I 143. Решить графически уравнение: л3 + 4л: = 0. I 144. Решить графически уравнение: х3 — 4 = 0. I 145. Решить графически ургвнение: х3 — 2х — 3 = 0. I 146. Решить графически уравнение: х‘ — 5.v + 6 = 0. I 147. Объем V одного грамма воды при температуре t° С приближенно ■выражается формулой: Построить график зависимости V от t и определить, при каком значении получит наименьшее значение V. ■ерх с начальной скоростью 125 м/сек; пройденный им путь S (в метрах), •-ли пренебречь сопротивлением воздуха, выразится формулой: 5=125/— 4,905/3, Це t—время, протекшее от начала движения. Построить график зависимости 07 определить, какова будет наибольшая высота подъема снаряда и какой момент она будет достигнута. а Сборник адгебраич. задач, ч, 11. V=1 +8,38.10-® (/ —4)3. 148. Снаряд зенитного орудия вылетает из дула орудия вертикально
34 Глава ХЦ] ГЛАВА XII. УРАВНЕНИЯ ВЫСШИХ СТЕПЕНЕЙ. Уравнением высшей степени называется всякое уравнение степени выше второй. Всякое алгебраическое уравнение п-й степени имеет п корней, кого- рые могут быть не все различны, и левая часть уравнения разлагаете# на п линейных множителей (теорема Гаусса). Общий вид уравнения 3-й степени есть ах3 + Ъхг2 + сх + d = 0. Если разделим обе части уравнения на а, то получим приведенное ура- внение, которое пишется в виде х3 + рх3 + qx -уг=0. Общий вид уравнения 4-й степени таков: ах1 + Ьх3ex*dx-у с=.О, а приведенное уравнение 4-й степени таково: х1 + рх3 + дхг2 -f- гх 4-5 = 0. § 1. Биквадратное уравнение. Биквадратным уравнением называется такое уравнение 4-й степени, в ко тором отсутствуют члены с нечетными степенями неизвестного, т. е. урав нение вида: ах1 + Ьх3-\-с=0. Это уравнение можно рассматривать как квадратное, но не относительно х а относительно х2; xi можно найти по формуле решения квадратного уран1 нения; извлекая из полученного результата квадратный корень, определяем х. Таким образом формула решения биквадратного уравнения такова: — Ъ ± У №— 4ас 2 а Эта формула дает четыре корня биквадратного уравнения, именно: Xl=+ l/-»+ Vb*-iac■ xt=- l/~b+ . 1 V 2 a a Г 2 a Хл= + -|/~ b- ^.-Jac. x^_y-b- V&-4ac . 1. Xх — 5x'2 + 4 = 0. 1. х*+12л:2 — 64 = 0. 2. + 12л:2 + 32 = 0. 2. xK + 9л;2 + 20 = 0. 3. 5л:* + jc2 — 4 = 0. 3. Злг* — лг2— 2 = 0. 4. 12v4 + at2 — 6 = 0. 4. 6л:1 — лг2— 15 = 0. 5. л1 — (а — b)x*— ab = 0. 6. х3 — -^- = 4 —а2. 7. а^лг4 — (а1 + Ь*) лг2 + а262 = 0. 8. Чему равна сумма корней биквадратного уравнения? 8. Чему равно произведение корней биквадратного уравнения? 9. Разложить на множители трехчлен 4х* — 17лг2 + 4. 10. Составить уравненцр, корни которого были бы ztl и :ii3.
Уравнения высших степеней 33 § 2. Двучленное уравнение. Приведенным двучленным уравнением называется уравнение вида: xn-*za = 0. Для решения таких уравнений принимают, вэ-первых, х = ^/а-г вследствие чего данные уравнения приводятся к более простым: г” —1=0 и а" + 1 = 0. Эти последние при нескольких небольших значениях п ре¬ шаются посредством разложения первых частей на множители, а затем най¬ денные значения г помножаются на \/а. Уравнения общего вида ax”z4f> = 0 преобразуются в приведенные посредством деления на коэфициент а и ре¬ шаются тем же способом. 11. х3 = — 1. 11. х3=1. 12. х3 = 8. 12. х3=—8. 13. х3 —а3 = 0. 13. х34-а3 = 0. 14. лг*— 16 = 0. 14. х‘ — 81=0. 15.x* 4-81=0. 15. х14-16 = 0. 16. х9 — 64 = 0. 16. .Vе—729= 0. 17. 125 д:3 4-8 = 0. 17. 125х3 — 27 = 0. 18. 81 лг44-4 = 0. 18. 16х‘ —25 = 0. 19. в3х8 —й3 = 0. 19. Ь*х*— я8 = 0. § 3. Трехчленное уравнение. Трехчленным уравнением называется уравнение вида: axa"4-ftx"4-c=0; и^2. Решение его приводится к решению двух двучленных уравнений подста¬ новкой х” = г, которая обращает данное уравнение в квадратное и позволяет найти два значения г. 20. х6 —Зх34-2 = 0. 20. х64-4х34-3 = 0. 21. Xе — 28х34- 27 = 0. 21. хв—19х3 —216 = 0. оо л х-в-1-3 ОО 8 — х-2 х~2 — (Зх -|- 5х2) £1. 4 — Д--3 • .г-4 — 5 6 в 24. х 5 4- 8 = 9 ^х3. 25. д4 — 7 = б^х3. 26- 9-^“^Sf3 = 0. 27. х10 10х5 4-9 = 0. § 4. Уравнения, левая часть которых разлагается на множители. Иногда левую часть уравнения высшей степени удается разложить на множители — на линейные или на нелинейные. Тогда уместно применить правило о том, что произведение нескольких сомножителей лишь тогда может быть нулем, когда один из них равен нулю. Приравнивая каждый из сомножителей нулю, разбиваем заданное уравнение на ряд уравнений, решая которые найдем корни начального уравнения. з*
36 Глава XII 28. х(2х— 7) = 0. 28. х(ах — Ь) = 0. 29. (5х —8)(4х+7)(х + 9) = 0. 29. (ах + 6)(or — d)(x — е) = 0. 30. х3 —Зх = 2. 30. х8 + 4 = Зх*. 31. х3 + 6 = 7х. 31. х3+ 12= 13х. 32. х3 + х2 = х+1. 32. х3 — х2 = х—1. 33. х3 — 5х3 = х— 5. 33. х3 + 2х2 = 4х + 8. 34. х3 + 2х2 —2х + 3 = 0. 34. х3—4х2 —4х —5 = 0. 35. х3 + 8х2+15х + 18=0. 36. х, + х3 = —2х + 4. 37. х* + 2х3—13х3—14х + 24 = 0. 38. х‘ — 2х3 — 8х2 + 19х—6=0. 39. лХ3■+ х + и + 1 = 0. 40. (а + х)3=(За — х)3. 41. х3 + (53-а2)х + а53 = 0. 42. g-+g+j=g. § 5. Возвратное или симметричное уравнение. Возвратным (или симметричным) уравнением называется такое уравне¬ ние любой степени, у которого коэфициенты членов, равноудаленных от начала и от конца уравнения, равны между собой. Полностью решаются в самом общем виде лишь уравнения 3-й, 4-й и 5-й степени. Кубическое уравнение решается простым разложением на множители левой части. Ле¬ вая часть уравнения 5-й степени также разлагается на два уравнения, из которых одно линейное, а другое — 4-й степени, опягь-таки возвратное. Возвратное уравнение 4-й степени первого рода таково: ах4 + bxs + сх9 + Ьх + а = 0. Разделив обе части этого уравнения на х, получают: а х2 + 5х + с + — + -^2 = 0. Сгруппировав члены с одинаковыми коэфициентами, получают: я(*а+}з)+# (*+ г)+с~°- Заменив х 4- ~ через новое неизвестное у, получают: и тем самым приводят возвратное уравнение к квадратному относительно^. Возвратное уравнение 4-й степени второго рода таково: ах4 + Ьх3 + сх2 — Ьх + а = 0. Решается оно так же, как и уравнение первого рода, с той лишь раз¬ ницей, что у будет заменить собою х ^, а выражение х2 -J- -~3 будет ви¬ ража гися через у*+ 2.
Уравнения высших степеней 37 Возвратное уравнение 5-й степени, как уже сказано, по выделении ли¬ нейного множителя сводится к возвратному уравнению 4-й степени. Всякое возвратное уравнение четной степени приводится к уравнению степени вдвое меньшей приемом, аналогичным решению уравнения 4-й степени. -Но это новое уравнение вдвое меньшей степени, вообще говоря, не будет возвратным. Например; ах* + Ьх3 4- сх1 + dx3 4- сх2 4- Ьх 4- а=0*, ах3 4- Ьх* 4- сх + d 4- j 4- ~ 4- ^ == 0; а С*3 х®) ^ х®) + с +3^) 4-d=0; x-i-j=y; ха4-^=У —2; х3 + ±=у3 — Зу; ' a (З'3— Щ + Ь (у* — 2) 4- су 4- d = 0; и окончательно: ау3 4-by* — (За — с)у — (2Ъ —d) = 0. Если это уравнение разрешимо, то и возвратное уравнение 6-й степени разрешимо. Наконец, возможны возвратные уравнения неполные; часто они разре¬ шимы разложением на множители левой части уравнения. 43. х3 — х24-х— 1 = 0. 43. ах3 -j-bx3 -\- Ьх-\-а = 0. 44. 2х* — 5х34-5х — 2 = 0. 44. Зх* — Юх3 4- 10х — 3 = 0. 45. 6х* — 5х3 — 5х — 6 = 0. 45. 12х* 4- 7х3 + 7х — 12 = 0. 46. ах* — bx3 + bx— а = 0. 46. ах1—Ьх3—Ьх + а = 0. 47. 6х*4-5х8 — 38х24-5х-{-6 = 0. 47. 6х*—35х34-62х2—35х 4-6 = 0. 48. 2х*4-х3 — 11х24-х4-2 = 0. 48. 2х* —Зх3 —х2 —Зх4-2 = 0. 49. 4х* — ЗЗх3 4* ЗЗх 4* 4 = 0. 49. 6х*4~73х3 — 73x4-6 = 0. 50. 6х* 4- 7х3 — 36х2 — 7х 4- 6 = 0. 51. 2хв4-5х*—13х3—13х24-5х4-2 = 0. 52. 15х!; 4- 34х* 4- 15х3 — 15х2 — 34х—15 = 0. 53. х® — 10х* 4- 27х* — 20х3 4- 27х2— 10x4-1 =0. 54. 2хв —х5 —8х‘4-8х24-х —2 = 0. 55. Доказать, что корни возвратного уравнения 4-й степени первого рода попарно обратны по величине друг другу. 55. Доказать, что корни квадратного уравнения 4-й степени второго Рода попарно обратны друг другу и по величине и по знаку. 56. Разложить на множители симметричный многочлен: 2х* —9х3 4-14х2 —9x4-2. .
58 Глава XIII 56. Разложить на множители симметричный многочлен: ЗОх* — 17х® — 228х- + 17лг + 30. 57- Составить уравнение с корнями: 2; -i-; 3 и 2 о 57. Составить уравнение с корнями: 2; J-; 3 и — . 2 О ГЛАВА XIII. ИРРАЦИОНАЛЬНЫЕ УРАВНЕНИЯ- Иррациональным уравнением называется такое уравнение, в котором неизвестное входит под знаком корня. Для решения такого уравнения нужно заменить его другим, не содержащим корней из неизвестных выражений.. Это достигается посредством возведения в степень, применяемого один раз или несколько раз последовательно. Так как возведение в степень вносит посторонние решения, то, решив иррациональное уравнение, нужно проверить каждый из корней подста¬ новкой его в то из уравнений, которое первоначально возводилось в степень. Если окажется, что испытуемый корень не удовлетворяет проверяемому уравнению, то он и не будет корнем данного уравнения, а должен при-| надлежать одному из дополнительных уравнений, которых всегда будет столько, сколько раз при решении производилось возведение в степень. 1. 5 Ч- |/б — х = 7. 1. х-цЛб + х^в. 2. |/ 5 +]/х — 4 = 3. 2. У17 — \Гх —8 = 4. 3. Vx + l + ]/2х+3 = 1. 3. \/~2х— 1 + |/х — Г = 1. 4. Зх + 4 + ]/лг + 2 = 8. 4. l/*+З + ^/Зх —3 = 10. 5. |/22 — х— ]/ 10—х =2. 5. ]/х + 20 — j/x— 1 = 3. 6. 2j/x + 18 + ]/4х — 3 = 15. 6. [/х^7 —]/х+Т= — 2. 7. l/2x+ 1 +Vx — 3 = 2 j/x. 7. j/x+З + j/x + 8 = 5^/х. 8. j/Зх — 3 + р/5х — 19 = \/ Зх + 4. 8. ]/2х+ 1 +]/ 7х— 27 = |/Зх + 4. 9. ]/1 +xj/xa+ 12= 1 +х. 9. ]/1 + х|/ха —24 = х —1. 10. х= — 2 + ]/ 4 +х|/36 + х2. 10. х=1 — У 1 — х|/16+х\ 11. | + 2 = /l+iv/64+“ 12. =
Иррациональные уравнения П ° Ч*. R ' X+V5 + X? х— Vb + x? ,4. 1= + j -13 x-\-V4 — & x—V4 — x? ^ 15,7+7^=4 2 ‘ 16- */5*~T^TT=l/3*+1- J7 V2x* + l + Vx — l_t? V ЗХ3 +1 — V2x-j- 1 2 ^2x2+ 1 — Ух— 1 ' ' К + K2x+T ~ 5 ' i9. / x+vx-Y x-^x=iyjYh' v + l — Vto + l_ ^2*+1+1 01 j , ; ,, . *+f+ySJ+l_ 2I- (*+2) +*'=4<*+2) 22. [5 + (*-4)*']*' = 3. 23. ^* + 2 ^**-3=0. 24. |/jc— 3 + 6 = 5^ — 3. 25. Ara + j/jca — 9 = 21. 26. Зл2 + 15x+ 2 j/.x2 + 5л: + 1 = 2. I 27. jc + j/2ax + xa = c. 1 27. 2a — ]/2ax + x*=x. 28. \Гх+\/а — x a. 28. j/a + Jcr + J/a— x — j/2a. 29. \/?>x + a + 2b — \/3x + a — 2b = 2]/x— a. 29. \/5x—3a + 4ft + |/5jc — 3a — 4£ = 2(/лг + я. 30. Уa — bx + j/c — dx — \/ a + c —(6 + d).>c. 31. |/a x + i/2a-f- x = -y ^ . 32. |/a + yf x— Yо, — у x= |/a. "a x L tfi V a?Jt3 v1' 34 V ax -f- —ж gj. V nx b + Vax V а к b — Vax Va + x — Va — x x ' 1 + Vax — b ~ 1 — Vax — b ЗД Va+x + Vx— a Va + к — Vx — a Vb + 2x + Vb — x ~ Vb + 2x — Vb^x' 37. Va—x+Vx—b Va — x — Vx—b Va — x Vx—b •к.
40 Глава XIV ГЛАВА XIV. СИСТЕМЫ УРАВНЕНИЙ СТЕПЕНИ ВЫШЕ ПЕРВОЙ. § 1. Решение систем уравнений. Для решения системы уравнений: Ах* + Вху + Су* + Dx + Еу + F— 0 и ахby —с, из которых одно 2-й, а другое 1 -й степени, выразим у через х из второго Решив последнее, найдем два значения Jtt и х„, а подставив их в вы¬ ражение у, получим соответствующие значения yt и _уа. В результате по¬ лучаются две системы решений. Для решения двух уравнений 2-й степени: исключаем из них сначала квадрат одного неизвестного, например у. Для этого умножаем первое уравнение на С,, второе на С и вычитаем одно из другого. Получим вспомогательное уравнение, которое представим для крат¬ кости в виде: Пользуясь тем, что полученное уравнение содержит только 1-ю степень у, выражаем из него у через х в рациональной форме: Полученное выражение у вставляем в одно из данных уравнений. Тогда составится уравнение относительно одного х 4-й степени. Если последнее будет решено, то будут найдены четыре значения х, а вставляя каждое из них в предыдущее выражение у через х, получим четыре соответствующих значения у. В случае, когда квадрат одного из неизвестных не входит в одно из уравнений, вычисление упрощается. Решить системы уравнений: Ах* + Вху + Су* + Dx + Еу + F= 0 и Л1х* + Вхху + Cty* + Dtx + Е,у + Fi = 0 ax* + bxy + dx + ey + /= 0. + dx + f bx-\-e
Системы уравнений степени выше первой 41 ! ха +-2ху — 4_у2 — 5х + 4 = 0; 4 / ха +-2*_у — ' \ -У—у— 2. I 2х* —ху + ЗУ" — 7х — 12_у + 1 = 0; 4* ( х+1=у. г l^=°'5; в fi = 5'' М7-‘ MS-* 14 /*+.у = а; I х2+У2 = 6ху. f ху=а\ 15- £=„. ^ У У х+у гху=99; ( *1г±5-10- *■{>+?-* [ х — 6 ==у + 6; | 7- ||-*-.v=.v О-i). 7‘ { 3 у 8.У+^,20: 8.JV'1: \ х* + У = 500. \ X —У = 0. /х*+У = 58,4; f*+J М»*-»=,0. 9.|*_ 2+У. 1 2 ' , 9и,. оч,. Л*+У+1)(*+У-11 ~ |о ^-3)-., т./ ,*+•; -3- '■■у — 3 12* 2у лгу' !х—у = п\ | 2х +_у=г5а; 89a2_(5y_*)y_ И- { х(к— у) + 3у2_ - (х+у)2 ■ I о(я —1) + 3 \(Ь+у)х = Ь*\ \2.{г 2 12- | д>Ъ - Ъ Ь -5: = — . КаЧ—у Ъ — х' \ у X а 1- ( — — - = 0- р+^З-1’ 1*2 J,2 p+4=i. }1 + 1=ш. ч а ' Ь ч * У (х+У _а . х-у Ь ’ 4 ху = с2. |£+-£ = 2- ««•ft" ' I ^ + ОУ т \ Ьх — ау п ■'
42 Глава XIV IK /**+3л^=18; \xy + 4y* = 7. (х+у-х*=><У, '* \3.у-х-У = 0. (х'“-ху+у* = 21-, ' \ 2ху —-у* =15. 4* —4у—ху = 0; 2ха *j* 2уа — 5ху — 0. 17. { I 6x* + xj/—У — Зх —4у = 15; \ 4ху~у‘* — 3х* +\Ьх — 7у = 18. ( 6х+21_у — 2л:2—27ху — 6У = 4; • ( 18. 19 20 9 ху + Зу ( 3ха + 2ху+уа = 43; \ х3 + 2ху + 3у* = 33. ( х1 + ху + 2у2 = 74; { 2x*-J-2x_y +У = 73. (^х*+{у*=19>7; 21 * I (* + 2)(x-2) _ 4 I (> + 7)(7— y) 3' /Зх4 —4х + У = 40; ( 2x* + Зх +У = 52. ( 2x + 3xy — 6 v , t I g 23. { I x — 2xy — 3y __ j I 3 ( 8(x+j/) — 7(xy+ 1) = 0; 24. < - ' \ 4(x—.y) — {xy~ 1) = 0. { xa +У + x +y — 68; *’ \ Xя—У + х — _y = 44. , t x3 + xy = a\ ''\y2 + xy=b. , (x(a—x)=y(a—y); ’ 1 xy = a\ i xa — xy = 2(a+ 1); ’ \ У—x^y — 2(1 — a). j (x — a) (y — b) — 0; 1 *(* — b)~y{*~У) _ | — 25. 26 27 28 23. 19. 2x3 + 6x—-6y = 4. ( 3xa—xy 4- 4y = 14; \ 2x* — xy+ 2y = 8. — 4x^ + 2y=17; jc3=16. /(x+l)(x-l) —78,3 01 | 78,3-0+ IjCv— 1> ( 3xs — 20. { a ly—• = 1: i x+iM8 \ V . f _У = 0. 22. y — x ( xy + x3=18; \ 2x3; — X3 = X + 6. f 6(x3-l)-^f3x-y-13; \ 4(x3— 1)—y(2x—У = 9. x3 — 4У — ху + 5_y = 1; Х3 + ЗУ — xy — 4y = — 1. ! + ху +У — 2 a\ ’• — xy + yi = 2b. ( 1 I x — — = a; -{ ■I* y I 11 I y~=-i ( a*x3 + !- V xy—a +У ,9 _ :2a*; [ (x+y + a)(x—y + a) = 0; 29 < 1 ' {x3 — +У + l)_x = a— y.
Cncietfbi уравнений степени выше первой 43 х2-(а + х)у-а2 = 0; (x* + f + x + y = *{a* + 1); ( х‘ — (а + х)у — с" = и; 30. < *2_2(х-у)+у2 _ 30- {.2 I 5^2 ~а- + ху = 0. Нередко возможно применить некоторые искусственные способы решения систем уравнений, соответствующие особым формам уравнений. Разъясним на примерах некоторые из этих способов. « Пример 1. Пусть даны уравнения х + у~ 8 и ху = 15. Форма этих уравнений показывает, что х и у можно рассматривать как корни одного квадратного уравнения: z2— 8г+15 = 0. Корни последнего сугь 3 и 5. Так как каждый из этих корней может быть принят за х и каждый за у, то- данная система уравнений имеет две системы решений: xt = 3, _yt = 5 и *а = 5, У2 = 3. Пример 2. Возьмем уравнения х+у — 7 и лг+у2 = 25. Возведя пер¬ вое из них в квадрат и вычтя затем второе, найдем произведение ху — 12. Зная же сумму и произведение неизвестных, можем определить неизвестные так, как показано в первом примере. Пример 3. Пусть даны уравнения Xя—у2 =24 и х~у—4. Разделив первое на второе, найдем уравнение 1-й степени х +у = 6, которое со вто¬ рым из данных определяет единственную систему х, = 5 и yt = 1. Пример 4. Даны уравнения х2 + у2 = 25 и ху — 12. Умножив второе уравнение на 2, сложим его с первым и затем вычтем из первого. Полу¬ чим: (х+у)2 = 49 и (х—у)2 = 1, откуда х +у =rt 7 и х—y = ztl. Поэтому решения данных уравнений получатся из следующих систем урав¬ нений 1-й степени: х+у = 7-, х+у=7; х + у = — 7\ х+у = — 7; х —‘у — 1; х —у = —-1 ] х у == 1: х —у — — 1. Эти решения суть: х, — 4, yt=3; xi=3, у2 = 4; х3 = — 3, у3=—4; xi~ 4, yi— 3. Те же уравнения можно было бы решить посредством особой подстановки, которую мы разъясним на следующем примере. Пример 5. Возьмем уравнения 2ху—у2 = 15 и х2 + ху = 36, первые части которых суть однородные выражения 2-й степени. Положим у = их. Получим: х*(2и—ы2) = 15 и х2(1+ а) = 36. Отсюда, определяя два выражения и сравнивая их, находим уравнение: . 15 , = т-^г-» или 12и2 — 19// + 5 = 0. 2 a—Ф 1 + и Корни этого уравнения сугь: н, = ~ и — По первому корню вы- 36 числим х* = —— = 16, т. е. х = zt: 4 и вследствие этого y = ux = t±l 5: 1 + « ' по второму корню найдем так же х2 = 27, т. е. х = zt 3 J. 3, в.ледствие чего у = z!z \/ 3. Всего получаем четыре системы решений.
44 \ Глава XIV Решить следующие системы х+_у = 12; *Ax+yZ' { ху = 35. Я2 / ^+^=13i I х'2-/ = 5. (х* + у* = 7 4; *** \ х + у=12. / / = 32; 34. \ . * I х—у = 4. I х + у _ з . 35. или обычными или искусственными приемами 31. ( = \ ху = 20. 36 37, J х-> 2* I. х.у = 80. f —+ —= 1; . { x У { x+y = 4. ( х3 + У = 2£ ^ xy = 12. / x^ — xy + y1 • 1 x-y = l. 3 = 43: = 10; 39. <{ F > К * 2’ [ x~y = 6. ( 40. 4 ( j/xy = 16. Г x2—_y=7; 41* { x2;y = 18. 42. { * ~yS==37; I x—y=l. 43. s i Xя—/ = 8. [£?+>:=18; 44. I У x I x+y = 12. 45.; I 4x'2 + 9/ = 45; / x2 + 2/ = 33; \ 2**-/ = 46. - {1+;Г4: { •I (J 1__ b. 3> x~ 6 ; X-.y=l. I x2 — _y ’ = 5; ’ \ xy=6. I * 34 35 36 37 x2—/=120; x -|-y = 20. x—у 3 _ x+y~T; xy= 10. 38 39. 40. 41. a + xy+./ = 67; x+y = 9. x+_y = 10. ( _У-У =2- j A ( /3^=15. ( x+/ = 11; \ x/ = 18. 42/ХЧУ = 65; I x + j<=5. I - 1. J >» x 2 * ’ I x2+/ = 45. 43 ^ xy - - - 3. (J* v3_2I. > + x — 2 » x— _y = 3. f 25x2 ' \ *У = -y-- 25x2 / : 16..
Системы уравнений степени выше первой 4S х2 +У2 5 . 46. { *у 2 ’ х2-у2 = 3. Xs—у3 =19; ,{ •{ 47 43. 49. х2у — ху2 = 6. х+у , х—у _ 5 . х + у 2 ’ (х+У х—j X—у ^X+j х2+уа = 20. v'S-?* — v = 5. х—у f х2+уа— ху=61; 5°. \ У V х+у— уху — 7. 58 6х , т/х+у _ 5 . х+у^г 6х 2* [ ху — х—у = 9. f , 20 59. { * + У х+у х+у’ | х2+уа=34. Г х + у = 444; I { I У х+ 10+ Уу+ 14=12. 60 46. | Л2—у2 ХУ X2 +у2 = 6 ’ = 13. 47. j х3+у3= 132; \ хау+ ху*= И 120. х+у *—у _ 1]). 48. •! х У Х+У 3 ' ( х2+у2 = 45. х+у— |/ху 51. х +у = ху = х2 + у 52. х —у = х2 —у2 = х8 —у3- ' 53 1Х+У = 5' \ х1 + у* = 97. ГА / *--У = 3: I хв —у3 = 33. Г X2 у2 X у _112. 55. { У^хз+у + х 9 1 х+у = 4. f + Z — 2®• 56. < У х 4 ’ I х-у=1. | ха —8 = 2х(2у —3). ; V X+V + Г 50 51 52 = 30; |х2+у2 + ху = 84; 1х+у— |/ху = 6. X — у = ху = X2 + у2. . х+у = х2+у2 = х3+у3. 54 55 56, 57. 58. 59. ( х у = 2; ’• \ x*+y* = i Г х + у = 2; I х5 +у5 = 242. I * л.^ д. * I Ji+rf+J I х —у = 2. ( + z=?Z. X 4 ’ ^ , у у2 х2 у х ^ х +у = 3. 11, = 4 ! 4 г + Зу f V 5у = 2: 60 4х + Зу у’ + 8 = 2у (х+2). (1/1МГ_1/1Е1=_2±; ^ К х—у 5х 10 ’ [ ху + х+у= 11. ху = 80. х—у = 2; х +44— у — 21 = 1. {]
4b Глава XIV xy = 12; 61. xz — 6; У+ z-= 20. ( xy = 48; 62. { >>2 = 54; I xz=72. f xy+yz = 28; 63. | xz y~ rrr 30* l x_y + xz— 10. f xj/ + x2+_y2 = 27; 64. <( x —у — 6; l .у — 2 = 3. j' x(x + .y + 2) = 70; 65. { y(x+y + 2) = 28; I z(x + j/ + 2) = 98. ( x+y + z — 20; 66. { xj/2=130; I x—2y-tz = 5. ( x+y+z= 12; 67. <( xz-\-yz — 35; l ха+.уа + га = 50. f x+.y + 2=7; 68. I х‘+у* + г* = 21; l уг = Х2. f ха+У = га; 69. { x+_y + 2=30; l xy = 60. ( xa + 2a—_ya = 1; 70. j х+.у + г = 3; У* - X2. / x+y + z= 13; 71. { АГ!+Уг + 2! = 61; I 2xz = xy + xz. ( x2, + У + 2'1 = 30; 72. _ya = 2x2 + 21'; I 2x = z. лгу = 54; 61. ^ yz = 36; xa —23 = 20. xy = 92; 62. | xz — 4y; yz= 16x. xa+_ya = 52; 63. { yj + 2*= 100; xa + 2a = 80. ( xa+y + 2a = 98; 64. «I x—y = 5; I x + г = 8. ( x{x—y + z)= 12; 65. <( y(x—y-l-z) = 9; I 2 (x—_y + 2) = 6. x—_y + 2=:8; 66. xa+y2 + 2a = 74; x—y + 32=22. x—_y + 2=3; 67. xz—yz = 2; x1 —_ya + 22 = 25. x+.y + 2 = 6; 68. -J xa+y + 2a=14; уг — 6. y3 + 2a = xa — 6; x+^ + 2=8; xz = 3. *»+.),» + «■ = 35; x— _у + г = 3; y3 = xz + 4. x—^+2=14; x2+y+ 2a = 244; 2г (x—_y) = x_y. xj/ + X2 —j;2 = 14; 21 ■ 2xj/ — 4; Зх - 2г. 69. 70. 71. 72.
Системы уравнений степени выше первой 47 ( 1 + 1 + 1=12; I х ' у 1 2 73 74. 75. 76. * 77. — + 1= 18; X у 3у+ 10z=3. (i+i-i= д: у z 1 . 1 с —|- — — 6j i X 1 z 9 i --- V V xz = 1. \ 78. { 3__ у xz x+y + z=6; xy + xz +yz = 11; I xyz = 6. [ x+y + z = 0; { xyz — 30; I X2 +У + z2 = ( jc + « = 5; I _у + 2 = 9; J _y2 + и = 28; ( x + za = 18. f jc + u —10; _y—2=1; = 38. I 79. i I 80. 81. 82. j _yz = 20; I у + a* = 74. [ xu—yz\ x + u= 13; | y + z= 11; ( >rs + ya + 23 + и2= 170. jc3 +.У3 + z3 + u3 = 252; x + y = 5; 2 + и = 7; . Xy=2U. ]/x — \/y=a\ ]/xy = й. ( j jc3 + л:уа = a; \ y + jc*y = ft. 73. f!_l + l = 6; j jc У z ’ I X у I 8x — 5г = 1. 74 —- = l; у X ’ 77. it . 1 — - = 0. \ xy z [ x -y + 2 = 0; 75. <{ xz xy yz = — 31; I xyz = 30. ( *+.V + z = 9; 76. { дс+г = 24; ( л*+У + 2:2 = 29. и—-x = 3; z "У — 5; «+>* = 12; za— x=44. f Ы—JC = 5; J лга + 2a = 52; I д;г = 24; ( y + «2=90. j xy — zu; 1 x.+:ziu j .£,— U—Z, (, лта+_уа—za — и2 = 21. ^ jc3 + y3 —л3+и8= 187; 80.)^ = 8: j 2 — н=1; ( xy=zu. [ ll±lz=:^_. I^jc— Vy~~b' { xy = (a2 — bi)‘*. ( ax’* + bxy = a; I Ay2 + 0*У = b. 78, 81. < 82
48 Глава X ( х2+_у2 = 579; ( х2—у* = 4яп; 83. { у2 + г2 = 1 За*; 83. { у» - г4 = /г2 - 2ап; I 2й — 8х2 = а2. I х2 + 22 = /г2+2ал + 2а2. ( f2=^l = £:. [ х2+y‘i-z* = a2; 84. < а b с'- 8М*_.у_* (. Ьсх2 + асу2 + абг2 з= /п9. ( т ~~ п ~ р ' <3 II а ( х2+у = а; 85. xz = b; 85. { х2 + гч — Ь; (у + г2 = л2. i У + 22 = С. ( x(y + z) = a; ( x+y=az; 86. <{ ,у(с+ г) = й; 86. х—y — bz; С II + 1 х2+У = сг. ( х* — (у — г? = а; ( х (у+ z) — 4a2; 87. <{ У — (х — г)2 = Ь; 87. { _y(x + z) = 3..2; К г2 — (х —yif- = с. 1 z(x +>) = 2a2. [ х°+у* = а; [ (x +y) (x + z) = a2; 88. { f + z* = b; 88. { (x+y)(y + z) = b2; 1 z2 + х2 = с. I (x + zj(y + z) = c*. ( х+у = а; ( x+y+z^a; 89. j х + г = Ь; 89. { yz = bx; { л«=у.+ г\ [ Х2=У + 22. 1 2х—3y + z = 5/z; ( x+y + z=2/n+n; S3, j х*-\-yl + zl = 3ml-\-2п2; 90. { х2+У + г2 = 2т3 + Ъп*; 1 ху xz у: — Зт2 — /г'3. I [x+y)z — 2mn. § 2. Составление систем уравнений. 91. Разложить число 209 на два множителя, сумма которых равна 30. 92. Разложить число 195 на два множителя, разность которых равна 2.| 93. Сумма двух чисел равна 13, сумма их квадратов равна 89. Найти эти числа. 94. Разность квадратов двух чисел равна 200. Если каждое из чисел уменьшить на 1, то разность их квадратов будет равна 180. Найти эти числа. 95. Если к произведению двух чисел придать меньшее из них, то полу¬ чится 5^-. Если к тому же произведению придать большее число, то полу¬ чится 55. Найти эти числа. 96. Произведение цифр двузначного числа в три раза меньше самого числа. Если к искомому числу прибавить 18, то получится число с теми же цифрами, но в обратном порядке. Найти число.
Системы уравнений степени выше первой 4Э 97. Произведение двух целых положительных чисел в три раза больше суммы их, а сумма квадратов тех же чисел равна 160. Найти эти числа. 98- Сумма двух чисел равна 22, а сумма кубов их равна 2926. Найти эти числа. 99. Разность двух чисел равна 3, а разность кубов их равна 657. Найти эти числа. 100. Найти такую дробь, чтобы сумма квадратов ее элементов равня- * 25 лась 25, а сумма этой дроби с обратной дробью равнялась бы —. 101. Сумма квадратов цифр двузначного числа равна 34; произведение искомого числа на обращенное равно 1855. Найти число. 102. Сумма трех чисел, составляющих непрерывную разностную пропор¬ цию, равна 54, а произведение их равно 5760. Найти эти числа. 103. Сумма трех чисел, составляющих непрерывную разностную пропор¬ цию, равна 12, а сумма квадратов их равна 66. Найти эти числа. 104. Сумма цифр трехзначного числа равна 11; сумма квадратов тех же цифр 45. Если от искомого числа отнять 198, то получится число обра¬ щенное. Найти это число. 105. Сумма трех чисел, составляющих непрерывную кратную пропор¬ цию, равна 19, а сумма квадратов их 133. Найти эти числа. 108. Найти трехзначное число по следующим условиям: частное от деле¬ ния искомого числа на сумму его цифр равно 48; частное от деления на ту 2 же сумму произведения цифр равно 10 цифра десятков есть среднее ариф¬ метическое остальных цифр. 107. Найти два числа, сумма квадратов которых превышает сумму их первых степеней на количество а, а разность квадратов превышает разность первых степеней на количество Ь. 103. Отношение суммы двух чисел к их разности равно отношению р к q, а произведение суммы тех же чисел на их разность равно а. Найти эти числа. 109. Площадь прямоугольника 112 см'1. Сумма площадей квадратов, построенных на смежных сторонах прямоугольника, 260 см1. Найти сто¬ роны прямоугольника. 110. Отношение сторон прямоугольника равно 6. Сумма площадей квадра¬ тов, построенных на этих сторонах, 592 сж2. Найти стороны прямоугольника. 111. Высота трапеции равна 18 с.и. Площадь ее равновелика площади прямоугольника, построенного на основаниях трапеции; тройное верхнее основание, сложенное с нижним, в 4 раза больше высоты. Определить осно¬ вания трапеции. 112. Гипотенуза прямоугольного треугольника больше катета на 1 см, сумма же гипотенузы с этим катетом в 5 раз больше другого катета. Опре¬ делить стороны этого треугольника. 113. Периметр прямоугольного треугольника равен 24 см, площадь 24 с.и2. Найти стороны треугольника. • 114. Периметр прямоугольного треугольника равен 30 с.и, площадь 30 с.и2. Найти стороны треугольника. 115. Сумма диагоналей ромба на 6 с.и меньше его периметра. Вычис¬ лить длину стороны ромба и его диагоналей, если площадь ромба равна 24 с.и2. 4 Сборник ялгебранч. задач к. Ц.
50 Глава XIV 116. Сумма площадей двух кругов, касающихся внешним образом, равна 90тгсла. Вычислить диаметры кругов, если расстояние между их центрами равно 12 см. 117. Определить катеты прямоугольного треугольника, гипотенуза кото¬ рого равна а, а площадь s. 118. Определить стороны прямоугольного треугольника по данной пло¬ щади его s и периметру 2 р. 119. Найти стороны прямоугольного треугольника по периметру 2 р и разности катетов d. 120. Найти диагонали ромба по стороне его а и площади s. 121. Тело, находившееся в покое, начинает двигаться равномерно уско¬ ренно. Пройдя путь в 1440 м, оно приобрело скорость 24 ж в секунду. Сколько времени и с каким ускорением двигалось тело? 122. Равнодействующая двух сил, пересекающихся под прямым углом, равна 15 кг. Если одну из этих сил уменьшить на 4 кг, а другую увеличить на 6 кг, то равнодействующая их будет равна 17 кг. Найти величины дей¬ ствующих сил. 123. Между двумя городами, находящимися на расстоянии 450 км, ходят пассажирский и товарный поезда. Пассажирский проходит весь путь на 8 час. скорее, чем товарный. После увеличения скорости обоих поездов на 5 км в час пассажирский поезд стал проводить все расстояние только на 6 час. скорее товарного. Найти скорость обоих поездов. 124. Два велосипедиста выезжают одновременно навстречу друг другу из городов А и В, расстояние между которыми 28 км. Через 1 час езды они встретились и, не останавливаясь, продолжали путь. Первый прибыл в го¬ род В на 35 минут скорее, чем второй прибыл в город А. Найти скорость каждого, а также расстояние, которое проехал каждый до встречи. 125. Двт тела начинают двигаться равномерно по двум прямым линиям, пересекающимся под прямым углом, к точке их пересечения. В начале дви¬ жения расстояние между телами 169 см, скорость первого тела 1 см/сек, второго—4 см/сек. Определить расстояния тел до точки пересечения их путей, если известно, что через 2 секунды после начала их движения рас¬ стояние между телами будет равно 25 см. 126. В бак проведены две трубы; через первую вода вытекает, через вторую вливается. При совместном действии обеих труб бак наполняется в 24 часа. Если бы увеличить площади поперечных разрезов труб тЗк, чтобы первая труба двумя часами скорее опоражнивала бак, а вторая также двумя часами скорее наполняла его, то при совместном действии обеих труб бак наполнился бы в 12 час. Во сколько часов первая труба о: оражнивает бак и во сколько часов вторая его наполняет? 127. На протяжении 18 м переднее колесо эки'пажа делает на 10 обо¬ ротов больше заднего. Если бы окружность переднего колеса увеличить на 6 дм, а окружность заднего уменьшить на 6 дм, то на том же протя¬ жении переднее колесо сделало бы на 4 оборота больше заднего. Найти окружность обоих колес. 128. На протяжении 27 м заднее колесо экипажа делает на 5 оборотов меньше переднего. Если бы окружность заднего колеса увеличить на 3 дм, а окружность переднего уменьшить на 3 дм, то на том же протяжении заднее колесо сделало бы на 9 оборотов меньше переднего. Найти окруж¬ ность обоих колес.
Прогрессии 51 129. Тело взвесили на неправильных весах, т. е. с неравными плечами; при установке гирь на одной чашке весов вес оказался р граммов, на другой—д граммов. Найги истинный вес тела. 130. Три проводника, соединенные последовательно, имеют сопротивле¬ ние в 11 омов. При параллельном соединении их сопротивление равно 1 ому. Вычислить сопротивление каждого из проводников, если сопротивление одного из них на 1 ом меньше суммы сопротивления двух других. 131. Определить удельный вес двух тел, если смесь, содержащая а ки¬ лограммов первого тела и b килограммов второго, имеет удельный вес р, а смесь, содержащая с килограммов первого и d килограммов второго, имеет удельный вес д. 132. Из двух точек М и N, находящихся на расстоянии d сантиметров, одновременно начинают двигаться два тела, которые встречаются, когда первое тело прошло а сантиметров. Определить скорости тел, зная, что число сантиметров, выражающее разность между скоростями первого и вто¬ рого тел, равно числу секунд, протекших от начала движения до встречи. 133. По окружности круга движутся два тела; первое пробегает окруж¬ ность d секундами скорее, чем второе, и если они движутся по одному направлению, то сходятся через каждые т секунд. Какую часть окружности пробегает каждое тело в секунду? 134. Три тела движутся по прямой линии от точки М к точке N. Второе тело начало двигаться а секундами, а третье b секундами позже первого. Скорость первого тела менее скорости второго на с сантиметров, скорость третьего равна v сантиметрам в секунду. Найти расстояние MN и скорость первого тела, если известно, что все тела достигают точки М в один и тот же момент. § 3. Графическое решение систем уравнений с двумя неизвестными 2-й степени. Начертив графики каждого из уравнений системы, определяют коорди¬ наты точек пересечения этих графиков; значения координат и дают искомые корни системы уравнений. Решить графически системы уравнений: 135. х+у—5; 136. х — j/=2; 137. лг3+У^25; 138. ха+уя = 41; 4 ху — 2. ху = 3. у = — х. х—у= 1. 139. х8+уа = 61; 140. *+_у* = 19; ху = 30. ^ + **=13. « ГЛАВА XV. ПРОГРЕССИИ. § 1. Арифметическая, или разностная, прогрессия. Арифметической, или разностной, прогрессией называется такой ряд чи- сел, в которых каждое последующее, начиная со второго, получается из предыдущего путем прибавления к нему одного и того же постоянного числа, называемого разностью аоифметической прогрессии (обозначаемого 4*
{& Глава XV буквой d). Любой из членов прогрессии может быть принят за первый член, обозначается он я, (или просто о), а следующие ая,а3,аг..ая (или просто о). Если разность арифметической прогрессии d есть величина положитель¬ ная, то прогрессия называется возрастающей, если разность отрицательна,— убывающей. Сумма некоторого числа членов арифметической прогрессии, начиная ет некоторого числа а1г принимаемого за первый, до некоторого а„, при¬ нимаемого за последний, обозначается через sn (или просто s). Формулы арифметической прогрессии: ак=аг + а(п—1)> или u = a + d(n—1); С\ -f- О -I- U su= 2 ~ , или s~—Y~'n' Полезно знать еще одну формулу суммы членов арифметической про¬ грессии: 2fl + rf(«-i) _ s = 2 п- 1. Найти 15-й член и сумму 15 членов прогрессии: 2, 5, 8, 11... 1. Найти 20-й член и сумму 20 членов прогрессии: 3, 7, 11, 15... 2. Найти 18-й член и сумму 18 членов прогрессии: — 3, —5, —7, — 9... 2. Найти 13-й член и сумму 13 членов прогрессии:—2, — 6, — 10, — 14... 3. Найти сумму всех двузначных чисел от 21 до 50 включительно. • 3. Найти сумму всех двузначных чисел. 4. Найти сумму ряда последовательных целых чисел от 1 до п. 4. Найти сумму ряда последовательных целых чисел от 1 до 2п. 5. Найти сумму натуральных чисел от 1 до 100. 5. Найти сумму натуральных чисел от 1 до 1000. 6. Найти сумму всех четных чисел до 200 включительно. 6. Найти сумму всех нечетных чисел до 175 включительно. 7. Найти л-е нечетное число и сумму л нечетных чисел. 7. Найти л-е четное число и сумму л четных чисел. 8. Найти сумму л членов прогрессии о, 2а — Ь, За—2Ъ... 8. Найти сумму п членов прогрессии Ь, 2Ь— а, 3Ъ — 2о... 9. Между числами 3 и 24 вставить 6 средних арифметических, т. е. так, чтобы искомые числа вместе с данными составили арифметическую про¬ грессию. 9. Между числами 17 и 82 вставить 12 средних арифметических. 10. Между 27 и — 28 вставить 10 средних арифметических. 10. Между 17 и—19 вставить 17 средних арифметических. 11. а — 7; d = 4; п— 13. Найти я и а. 11. я = 2; d = 2; п = 40. Найти а и а. 12. я = 56; d=—3; я=11. Найти о и s. 12. о = 63; d=—5; п= 8. Найти и и а. 13. о=149; d = 7; п = 22. Найти о и а. 13. о=65; 5; и=12.в Найти а и а. 14. о=—22; d = — 2; п = 40. Найти а я s.
ИрбГресСЙЯ 63 14. о =13; d— — 3; л = 58. Найти о и 5. 15. а —2; о = 87; 5 = 801. Найти d и л. 15. а =—13; о = 27; 5=77. Найти d ч п. 16. о=10; а = —9; 5=10. Найти d и л. 16. а=160; о=17; 5=1062. Найти dun. 17. а — 3; о = 63; л=16. Найти d и 5. 17. а=1; ц = 81; л=17. Найти das. 18. а = 36; ц = 8; л=15. Найти d и 5. 18. а=169; о = 8; я = 24. Найти d к s. 19. о=10; л=14; 5=1050. Найти о и d. 19. о = — 40; л = 20; 5 = —40. Найти о и «Г. 20. о = — 45; я = 31; 5 = 0. Найти и a d. 20. а=16; л = 9; 5=0. Найти and. 21- о = 21; л = 7; 5=105. Найти and. 21. о = 92; л=11; 5=517. Найти а и Л 22. и=105; л=16; 5=840. Найти а и d. 22. о = —143; л = 33; 5 = — 2079. Найти and. 23. о = 4; of = 5; ц = 49. Найти л и 5. 23. о=1; d=3; а —22. Найти л и 5. 24. 0=14,5; rf= 0,7; о = 32. Найти л и 5. 24. о = —28; d=7; u= 28. Найти л и 5. 25. d= J5; л=10; 5 = 340. Найти а и и. 25. л = 50; 5 = 425. Найти о и о. 26- d = y; л = 25; s = — 75. Найти о и о. 26. d =— л = 33; 5 = —33. Найти а и и. 27. о = 2; d—5; 5=245. Найти л и о. 27. о = 40; d = —4; 5=180. Найти л и о. 28. а —41; d= 2; 5 = 4784. Найти лил. 28. о=18; rf = 6; 5=1782. Найти и и о. 29. rf=3; п = 29; 5=155. Найти л и о. 29. d — 5; и — 77; 5=623. Найти л и а. 30. rf=4; о = 88; 5=1008. Найти л и о. 30. rf=l,5; о = 45; 5 = 682,5. Найти л и о. 31. Третий член прогрессии 25, десятый — 3. Найти первый член и разность. 31. Пятый член прогрессии 13, девятый 19. Найти первый член « разность. 32. Четвертый член прогрессии 10, седьмой 19. Найти сумму дасити членов. 32. Пятый член прогрессии — 8, семнадцатый 28. Найти сумму пятнад¬ цати членов. 33. Четвертый член прогрессии 9, девятый — 6. Сколько нужно взять членов, чтобы сумма их равнялась 54? 33,- Десятый член прогрессии 4, девятнадцатый — 32. Сколько нужно взять членов, чтобы сумма их равнялась 180? 34. Сумма третьего и седьмого членов прогрессии равна 4, а сумма второго и четырнадцатого — 8. Найти прогрессию.
$4 Глава XV 34. Сумма четвертого и десятого членов прогрессии равна 44, а сумма второго и пятнадцатого 53. Найти прогрессию. 35. Найти разность прогрессии, первый член которой равен 100, а сумма шести первых членов в 5 раз больше суммы следующих шести членов. 35. Найти первый член прогрессии, разность которой равна 4, а сумма первых пяти членов в 3 раза меньше суммы следующих пяти членов. 36. Составит^ такую прогрессию от 1 до 21, чтобы сумма всех ее членов относилась к сумме членов между 1 и 21 как 11:9. 36. Составить такую прогрессию от 1 до 29, чтобы сумма всех ее членов относилась к сумме членов между 1 и 29 как 4:3. 37. Первый член прогрессии 1; сумма т первых членов ее относится к сумме я первых же членов как /л2: я2. Найти прогрессию. 37. Первый член прогрессии 2; сумма т первых членов ее относится к сумме л первых же членов как /л (/я+1):я (я+1). Найти про¬ грессию. 38. Найти сумму /я + я членов прогрессии, в которой /я-й член равен я, а я-й член равен /я. 38. Найти сумму т — я членов прогрессии, в которой сумма т членов равна л, а сумма л членов равна т. 39. Найти прогрессию, зная, что сумма перзого и второго членов ее равна 7, а произведение их равно 10. 39. Найти прогрессию, зная, что разность между вторым и первым ее а2 членами равна а, а произведение их равно jg. 40. Найти сумму 100 членов ряда: 1 — 3 + 5 — 7+--- 41. Найти прогрессию, зная, что сумма трех первых членов ее равна 15, а произведение их 80. 41. Найти прогрессию, зная, что сумма трех первых членов ее равна нулю, а сумма их квадратов 50. 42. Найти прогрессию, зная, что сумма второго и четвертого членов ее равна 16, а произведение первого члена на пятый равно 28. 42. Найти прогрессию, зная, что сумма первого и пятого членов ее равна 12, а произведение второго члена на четвертый равно 32. 43. Числа градусов, содержащихся в последовательных внутренних углах многоугольника, составляют прогрессию, разность которой 10°; наи¬ меньший угол этого многоугольника 160°. Сколько в многоугольнике сторон? 44. Числа градусов, содержащихся в последовательных внутренних углах многоугольника, составляют прогрессию, разность которой 5°; наи¬ меньший угол этого многоугольника 120°. Сколько в многоугольнике сторон? 45. Свободно падающее тело в безвоздушном пространстве проходит в первую секунду 4,9 м, а в каждую следующую на 9,8 м более, чем в предыдущую. Какое расстояние оно пройдет в 21-ю секунду и за все 21 секунду? 46. Сколько секунд будет падать тело с высоты 4410 л? 47. Тело, брошенное вверх, теряет каждую секунду 9,8 м своей скоро¬ сти. Сколько времени будет лететь вверх тело, брошенное вертикально со скоростью 656,6 м в секуйду?
Прогрессии 55 48. Через сколько времени тело, брошенное вертикально вверх со скоростью 784 л в секунду, упадет на землю? 49. С какою скоростью было брошено вертикально вверх тело, если оно упало через 1,5 минуты обратно на землю? 50. До какой высоты поднялось вертикально брошенное тело и с какой скоростью оно было брошено, если через минуту оно упало обратно на землю? 51. Между сторонами угла проведены на одинаковом расстоянии друг от друга 15 параллельных отрезков, из которых каждый следующий на 3 см длиннее предыдущего. Найти длину первого и последнего, если сумма всех отрезков 405 см. 52. В горных местностях температура воздуха летом при подъеме на каждые 100 ж в среднем опускается на 0,7°С. В 11 часов на горе тер¬ мометр показывал 14,8°; как высоко находится наблюдатель, если у под¬ ножья в это время температура была 26°С? 53. Через сколько секунд упадет в колодезь глубиною в 44,1 м камень (сопротивление воздуха во внимание не принимается)? 54. Ступенчатый шкив состоит из шести ступеней. Диаметры их со¬ ставляют арифметическую прогрессию. Наи¬ больший диаметр равен 240 мм, наименьший 80 мм. Найти остальные диаметры (черт. 4). 55. Турист, восходя на гору, поднялся в первый день на высоту 900 м, а в каждый о следующий день продолжал подниматься на •'* высоту на 50 м меньшую, чем в предыдущий. Во сколько дней он добрался до вершины горы, если высота ее 5250 ж? 56. Паровоз прибыл с пассажирского вок¬ зала ша товарную станцию, расстояние ме¬ жду которыми 3 км. При этом в первую минуту он прошел 480 м, а в ка¬ ждую следующую проходил на 40 м меньше, чем в предыдущую. За сколько минут совершил он весь переезд? 57. По шоссе едут экипаж и автомобиль по одному и тому же направлению. Экипаж находится впереди автомобиля на 84 м и едет равномерно со скоростью 3 м в секунду. Автомобиль же проезжает в первую секунду 8 м, а в каждую следующую на 0,1 ж меньше, чем в пре¬ дыдущую. Сколько времени будет ехать автомобиль, пока поравняется с экипажем? 58. Два автомобиля движутся друг другу навстречу из двух мест, рас¬ стояние между которыми равно 240 м. Первый автомобиль выехал на 3 се¬ кунды раньше второго и движется равномерно со скоростью 10 .и в се¬ кунду. Второй же проходит в первую секунду своего движения 2 ж, а в каждую следующую на 1 ж больше, чем в предыдущую. Через сколько секунд после выезда первого автомобиля произойдет их встреча? 59. Предполагают, что при углублении на каждые 30,5 ж внутренняя температура земли возрастает на 1°С. Если на поверхности земли температура равна-10°С, то на какой глубине она достигает: точки кипения воды (100°), плавления свинца (334°), плавления железа (1200°)? Полагая, что указанный закон не изменяется до самого центра земли, найти температуру в центре вемли. Средний радиус земли равен 6370 км.
65 Глава XV, 60. Школьный кооператив приобрел учебные пособия на сумму 800 рубД с условием уплачивать эту сумму в течение нескольких месяцев так, чтобы в первый месяц внести 120 руб., а в каждый следующий платить на 20 py6J больше, чем в предшествующий месяц. Через сколько месяцев кооператив выплатит всю указанную сумму? S 61. Камень, пущенный в колодезь, достиг поверхности воды через 4 се-1' кунды. Определить глубину колодца до поверхности воды. j 62. Авиатор, выпрыгнув из корзины привязного аэростата, пролетает^ вниз 100 м прежде чем раскроется парашют. Сколько времени проходит] до раскрытия парашюта? 63. На стрелковые состязания назначено несколько призов; наибольший из них 150 руб., а ценность каждого следующего приза уменьшается на одну и ту же сумму до самого меньшего в 30 руб. Общая ценность при¬ зов 360 руб. Сколько всего назначено призов? 64. Периметр многоугольника равен 158 см, причем длины сторон его составляют арифметическую прогрессию, разность которой равна 3 см. Наи¬ большая сторона многоугольника равна 44 см. Сколько сторон имеет много¬ угольник? 65. Поезд, отходя от станции, равномерно увеличивает свою скорость| и через 26 минут достигает скорости 60 км в час. Каково ускорение по¬ езда в минуту? § 2. Геометрическая, или кратная, прогрессия. Геометрической, или кратной, прогрессией называется такой ряд чисел, в котором каждое последующее, начиная со второго, получается из предыду¬ щего пугем умножения его на одно и то же постоянное число, называемое знаменателем геометрической прогрессии (обозначаемое буквой q). Если знаменатель геометрической прогрессии q больше 1, то прогрес¬ сия называется возрастающей; если меньше 1, то прогрессия называется убывающей; если он отрицателен, прогрессия представляет собою ряд знако¬ переменный. Формулы геометрической прогрессии: ап — аlq”~i, или u — aqя-1; а„ я—-я, ия—я я—ия или s = -L—r, или s=——-. я q—1 q—1 > \—q 66. Найти сумму 10 членов прогрессии: 10, 20, 40 66. Н'йти сумму 8 членов прогрессии: 5, 15, 45 67. Найти сумму 7 членов прогрессии: —4, 16, —64... 67. Найти демму 10 членов прогрессии: 3, 6, 12... 68. Найти сумму 8 членов прогрессии: 3, —1, i... 68. Найти сумму 11 членов прогрессии: 2, 1, . 69. Найти сумму 5 членов прогрессии: /§■<■ VI- /5 Л t ^ 1, | j...
Прогрессии б7 2 13 70. Найги сумму я членов прогрессии: 3 , 2 , g 3 1 1 70. Найти сумму я членов прогрессии: ^о 71. Найти сумму я членов прогрессии: 3 ~\Г% 3 j/lT. 1 I 71. Найти сумму л членов прогрессии: 2, J/J, 1... 72. Между числами 47 и 1269 вставить 2 средних пропорциональных количества. 72. Между' числами 31 и 496 вставить 3 средних пропорциональных количества. а Ъ _ 73. Между числами ^ и — вставить 5 средних пропорциональных ко¬ личеств. Ь2 я2 73.-Между числами — и вставить 9 средних пропорциональных ко¬ личеств. 74. Найти сумму 6 членов геометрической прогрессии, любой член ко¬ торой выражается в виде 3 • 2K_l, где л есть порядок (номер) члена. 74. Найти сумму 5 членов прогрессии, любой член которой выра¬ жается в виде 2 -5m_1. 75. Найти сумму я членов прогрессии, любой член которой выра¬ жается в виде (—1)”а"-1 Ьт~п+*. 75. Найти сумму я членов прогрессии, любой член которой выражается в виде (—Ь”~*. 76. я = 6; д = 3; я = 8. Найти и и л. 76. о =—5; 7 = 2; л = 9. Найти и и s. 77. о = 5; 7 =—я = 6. Найти и и л. 77. Д=^; 7=§^ я =10. Найти и и л. 78. и = 128; q = 2; я = 7. Найти о и л. 78. ц = 78 125; 7 = 5; я = 8. Найти о и л. 2 2 79. к = 27? 7 — — з’>я = 5- Найти о и л. 79. и = —243; q=-r^‘, я = 6. Найти о и л. 80. о = 3; ц= 12 288; я = 5. Найти q и л. 80. о = 8, ц=10 368; л = 5. Найти q и л. 81. о=81; и= — я=6. Найти q и л. 81. а = ^; и =—л = 6. Найти q и л. 82. 7 = 2; я = 7; л = 635. Найти о и и. 82. 7=—2; я = 8; л = 85. Найти о и к. 83. 7= —л = 8; л = = . Найти о и и. ’ 2 ’ 16 • 83. 7 = 1; я = 6; л = ^. Найти о и и. 84. а = 3; 7 = 2; и = 96. Найти лил. 84. а = 5; 7 = 3; и =405. Найти лил.
58 Глава Х\ ^ 2 32 85. а— 9; g=v; ц = —. Найти я и s. о ZI 85. а = 9 =—4; « = 96. Найти лил 86. а = 2; «=1458; $ = 2186. Найти q и я. 86. а=1; «=2401; $ = 2801. Найти q и я. 87. а = 3; « = 96; s=189. Найти q и п. 87. а = 2; «=1458; $=1514. Найти q и л. 88. а =7; 9=3; $ = 847. Найти «ил. 88. а = 8; 9 = 2; s = 4088. Найти и к п. 89. а = 2; 9=—3; $ =—364. Найти «ил. 89. а = 3; 9=—2; $=33. Найти и и л. 90. «=—216; 9=—6; 5=—186. Найти аил. 90. « = 250; 9=5; s=312. Найти аил. 91. « = 32 768; 9 = 4; $=43 690. Найти а и п. 91. «=1215; 9=—3; $ = 915. Найти аил. 92. а =15; и = 4; $ = 225. Найти q и «. ' 92. а = 12; л = 4; $=480. Найти q и «. 93. а=12; я = 3; $=372. Найти 9 и «. 93. а=15; я = 3; $=105. Найти q и «. п. 32 - 133 „ .. 94. « = —д-; я = 6; $ = —. Найти 9 и а. 94. « =—л = 6; $=—66,5. Найти qua. 95. «=135; я = 3; s = 195. Найти qua. 95. « = 8; л= 3; $ = 14. Найти 9 и а. 96. Первый член прогрессии равен 1; сумма третьего и пятого членов 90. Найти прогрессию. 96. Первый член прогрессии равен 3, разность между седьмым и чет¬ вертым членами 168. Найти прогрессию. 97. Сумма первого и третьего членов прогрессии равна 15, а сумма второго и четвертого 30. Найти сумму десяти членов. 97. Разность между третьим и первым членами прогрессии равна 24, а раз¬ ность между пятым и первым 624. Найти сумму шести членов. 98. Найти четыре числа, составляющие геометрическую прогрессию, зная, что первое больше второго на 36, а третье больше четвертого на 4. 98. Найти четыре числа, составляющие геометрическую прогрессию, зная, что сумма крайних членов равна 27, а сумма средних 18. 99. Три числа, сумма которых равняется 114, можно рассматривать как три последовательных члена геометрической прогрессии или как 1-й, 4-й и 25-й члены арифметической прогрессии. Найти эти числа. 99. Три числа, сумма которых 124, являются тремя последовательными членами геометрической прогрессии и одновременно 3-м, 13-м и 15-м чле¬ нами арифметической прогрессии. Найти эти числа. 100. Найти прогрессию из шести членов, зная, что сумма трех первых равна 112, а трех последних 14. 100. Найти прогрессию из шести членов, зная, что сумма членов, сто¬ ящих на нечетных местах, равна 4-55, а сумма членов, стоящих на четных местах, равна 1365. 101. Три числа, составляющие геометрическую прогрессию, дают в сум¬ ме 26; если к этим^ числам прибавить соответственно 1. 6 и 3, то полу¬
(fporpecciw 59 чатся Три числа, составляющие арифметическую прогрессию. Найти эти числа. 101. Три числа, составляющие арифметическую прогрессию, дают в сумме 15; если к этим числам прибавить соответственно 1, 4 и 19, то получатся три числа, составляющие геометрическую прогрессию. Найти эти числа. 102. Если из четырех неизвестных чисел, составляющих арифметическую прогргссию, вычесть соответственно 2, 7, 9 и 5, то получатся числа, со¬ ставляющие геометрическую прогрессию. Найти члены арифметической про¬ грессии. 102. Если из четырех неизвестных чисел, составляющих геометрическую прогрессию, вычесть соответственно 5, 6, 9 и 15, то получатся числа, составляющие арифметическую прогрессию. Найти члены геометрической прогрессии. 103. Найти /я-й и я-й члены прогрессии, в которой (яг + л)-й член равен к, а (яг — я)-й равен /. , 103. Найти я-й и (яг 4- р)-й члены прогрессии, в которой яг-й член равен k, а р-й равен I. 104. Доказать, что во всякой геометрической прогрессии произведение членов, равноудаленных от начала и от конца, есть величина постоянная, равная произведению крайних членов. 104. Вывести формулу произведения л членов геометрической про¬ грессии. 105. Вычислить произведение пяти членов геометрической прогрессии, первый член которой ]/2, а знаменатель ^. 105. Вычислить произведение л членов геометрической прогрессии, первый член которой ]/ а, а знаменатель ^. 106. Допустимое число оборотов шпинделя токарного станка колеблется между Я(=11 и я(1 = 350 в минуту. Кроме этих чисел оборотов возможны еще девять других: я2, я3,..., л10, составляющих с данными геометриче¬ скую прогрессию. Найти эти числа оборотов. 107. В ремне, охватывающем шкив, натяжения возрастают в геометри¬ ческой прогрессии при увеличении угла обхвата на одну и ту же величину. Для углов обхвата в 15° и 90° натяжения соответственно равны 9,56 кг и 17,41 кг. Рассчитать натяжение через каждые 15° и определить наимень¬ шее натяжение при угле обхвата, равном 0°. 108. Числа, выражающие длину, ширину и вышину прямоугольного параллелепипеда, образуют геометрическую прогрессию; площадь основания параллелепипеда равна 108 ж2, а полная поверхность 888 м1. Определить измерения параллелепипеда. 109. Числа, выражающие длину, ширину и вышину прямоугольного параллелепипеда, образуют геометрическую прогрессию; объем паралле¬ лепипеда равен 216 ж3, а диагональ j/364 м. Определить измерения па¬ раллелепипеда. 110. После каждого качания поршня под колоколом воздушного насоса остается 0,83 бывшего раньше количества воздуха. Определить, как велико будет давление воздуха под колоколом после 15 качаний, если первоначаль¬ ное давление было равно 760 мм.
fiO Глава XV 111. В сосуде имеется 53 л 80-процентного спирта. Сколько литров чистого спирта будет еще в сосуде, если из него 20 раз отливать по 1 4 жидкости и каждый раз добавлять по 1 л воды? 112. В сосуде было 12-50 л 80-процентного алкоголя. Из него 3 раза брали некоторое количество жидкости и добавляли таким же количеством воды. После этого в сосуде оказалось 125 л чистого алкоголя. Какое ко¬ личество жидкости брали из сосуда каждый раз? Если абсолютная величина знаменателя прогрессии меньше единицы, то можно рассматривать в ней неограниченную последовательность членов. Из алгебры известно, что у бесконечно убывающей прогрессии пред. ип =0, Я-+СО а вследствие этого из формулы s„ = пРи п—*со* получается фор¬ мула Для предела суммы членов бесконечно убывающей про¬ грессии. Определить пределы суммы следующих бесконечно убывающих гео метрических прогрессий: 113. l-^ + ^ + g-l--.. 114. 1 — 3+ 9~~27+ • • • 45- |/1+|/§+s(/jH "‘•ifcb-zW+b- 113. 1 + 3~f"9~f"27-b ‘' 114. 1-Л + 1-Л+... 115. |/5+ j/5 2+ V3 116. 2— V3 1 -+ 2— КЗ 2 + V3 117. Составить такую бесконечно убывающую прогрессию, в которой каждый член в К раз больше суммы всех следующих за ним членов. 117. Составить такую бесконечно убывающую прогрессию, в которой каждый член в К раз меньше суммы всех следующих за ним членов. 118. Прямолинейный отрезок АВ делится в точке С пополам, далее АС делится пополам в точке D, затем CD пополам в точке Е, DE попо¬ лам в точке F, ЕЕ пополам в точке G и т. д. до бесконечности. Опреде¬ лить предельное расстояние точки деления от точки А. 118. Прямолинейный отрезок АВ делится в точке С пополам, далее ВС делится в D пополам, затем CD в Е пополам, DE в F пополам, ЕЕ в G пополам и т. д. до бесконечности. Определить предельное расстояние точки деления от А '. 119. В квадрат, сторона которого а, вписан через деление сторон по¬ полам другой квадрат, в этот квадрат вписан точно так же новый квадрат и т. д. до бесконечности. Определить пределы, к которым стремятся суммы сторон и площадей всех квадратов. 119. В правильный треугольник, сторона которого а, вписан через де¬ ление сторон пополам другой правильный треугольник. В этот треугольник вписан точно так же новый треугольник и т. д. до бесконечности. Определить пределы, к которым стремятся суммы сторон и площадей всех треугольников.
Логарифмы 61 120. Дан прав шьный треугольник, сторона которого а; из трех высот его строится новый правильный треугольник; из высот этого второго треуголь¬ ника— еще новый правильный треугольник и т. д. Определить пределы тес алгебраических сумм, из которых в одной периметры, а в другой площади треугольников поочередно являются слагаемыми и вычитаемыми. 120. Дан квадрат с диагональю а; сторона этого квадрата принимается за диагональ второго квадрата; сторона второго — за диагональ нового квад¬ рата и т. д. Определить пределы тех алгебраических сумм, из которых в одной периметры, а в другой площади квадратов поочередно являются сла¬ гаемыми и вычитаемыми. 121. В круг радиуса R вписан квадрат, в квадрат вписан круг, в этот круг — второй квадрат и т. д. Определить предельные значения сумм площа¬ дей всех кругов и всех квадратов. 121. В круг радиуса R вписан правильный треугольник, в этот тре¬ угольник вписан круг, в круг — правильный треугольник и т. д. Определить предельные значения сумм площадей всех кругов и всех квадратов. 122. На стороне угла в 45° взята точка на расстоянии а от вершины. Из этой точки опущен перпендикуляр на вторую сторону, из основания этого перпендикуляра — новый перпендикуляр на первую сторону и т. д. до бесконечности. Найти предел суммы длин этих перпендикуляров. 122. На стороне угла в 60° взята точка на расстоянии а от вершины. Из этой точки опущен перпендикуляр на другую сторону, из основания этого перпендикуляра—новый перпендикуляр на первую сторону и т. д. до бесконечности. Найти предел суммы длин этих перпендикуляров. ГЛАВА XVI. ЛОГАРИФМЫ. § 1. Общие свойства логарифмов. Логарифмом числа N при основании а называется показатель степени, в которую надо возвести основание а, чтобы получить число N. Это соотношение обозначается так: x = logaN. Исходя из определения логарифма, решить следующие задачи: 1. Какое число имеет логарифм 3 при основании 2? 1. Какое число имеет логарифм 2 при основании 3? 2. Какое число имеет логарифм -i- при основании 8? 3. При каком основании число 32 имеет логарифм 5? 3. При каком основании число 81 имеет логарифм 4? 4. При каком основании число 4 имеет логарифм -i-? 4. При каком основании число 9 имеет логарифм-^ ? 5. Чему равен логарифм числа 16, когда основание равно 2? 5. Чему равен логарифм числа 27, когда основание равно 3? 6. Чему равен логарифм числа 3, когда основание равно 81? 6. Чему равен логарифм числа 7, когда основание равно 49?
62 Глава XVI | 7. При каком основании log 16 равен 2? 7. При каком основании log 81 равен 2? 8. Найти х, зная, что log4xr = 3. 8. Найти х, зная, что log5xr = 3. 9. Какое число имеет при основании 5 логарифм — 2? 9. Какое число имеет при основании 3 логарифм — 3? 10. Найти логарифм при основании 2. 10. Найти логарифм —• при основании 3. о1 11. Найти логарифмы числа 1024, принимая за основание числа 2, 1 и 32. 11. Найти логарифмы числа 729, принимая за основание числа 3, 9 и 27. 12. Найти логарифмы числа 81, принимая за основания числа 12. Найти логарифмы числа 256, принимая за основание числа ~, ~ 13. Какое число имеет логарифм — 3 при основании 8? 13. Какое число имеет логарифм — 4 при основании 6? 14. При каком основании логарифм ^ равен — 5? 14. При каком основании логарифм равен — 3? 15. Найти логарифмы дроби ^ , принимая за основания числа 2, 4 и 8. 15. Найти логарифмы дроби ^, принимая за основания числа 3, 9 и 27 16. Найти логарифмы дроби ^, принимая за основания числа -i-, 9 ’ 27' 16. Найти логарифмы дроби > принимая за основания числа _L J_ 4’ 8 ' з 17. Основание равно —; найти числа, логарифмы которых суть 0, 1, -1, 2, -2, 3, -3. 17. Основание равно 1-^-; найти числа, логарифмы которых суть 0, 1, -1, 1, 3, -3, 4, -4. 1 2 18 18. Основание равно 2 найти логарифмы чисел, 6-^-, 1, 3 5 7 27 18. Основание равно; найти логарифмы чисел2-jr-, 1, 5 ЗУ 1/Ь 19. При каких основаниях число 125 имеет логарифмы 3, 1, —3, —1? 19. При каких основаниях число 343 имеет логарифмы 3, —3, 1, —1? 20. Если основание логарифмов равно 0,5, то чему равны логарифмы чисел 1, 4, 2, 8, i? 20. Если основание логарифмов равно 0,2, то чему равны логарифмы чисел 1, 25, 5, 0,04, 125, 0,008?
.Логарифмы 63 21. Какое число имеет логарифм у при основании 3? 2 21. Какое число имеет логарифм у при основании 2? .22. Между какими целыми числами заключаются логарифмы чисел 3, 5, 10, 25, 100 и 500 при основании 2? 22. Между какими целыми числами заключаются логарифмы чисел 5, 12, 862, 1613 и 11111 при основании 10? 23. Между какими отрицательными целыми числами заключаются лога¬ рифмы дробей 0,02, 0,034, 0,005, 0,000675, 0,00009 при основании 10? 23. Между какими отрицательными целыми числами заключаются лога¬ рифмы числа 597 при основании 0,1? 24. При каком основании число 5 имеет логарифм 2? 24. При каком основании число 3 имеет логарифм 2? 3 25. Найти число, логарифм которого при основании 8 равен ——. 2 25. Найти число, логарифм которого при основании 25 равен——. О 26. При каком основании число 7 имеет логарифм— 1у? з 26. При каком основании число 5 имеет логарифм—^? 27. Основание логарифмов — 8; найти числа, логарифмы которых суть: 1 а 9— L Л у g | 2 • 27. Основание логарифмов — 81; найти числа, логарифмы которых суть: 2 1 2— L z> х' 4 ’ Y’ 28. Найти логарифмы чисел — ^, у , 5 при основании, равном — 28. Найти логарифмы чисел — ^, — 2, — 32,64 при основании, рав- I НОМ у. 29. Чему равен логарифм ф/9 при основании 3, 81, 29. Чему равен логарифм \/49 при основании 7, у, 49, ;щ? 30. При каком основании [/8 имеет логарифмы--, —3, —1, --? 4 о з/— 2 3 30. При каком основании f/25 имеет логарифмы у , — —1, —2? 31. Чему равен log,, 1? 32. Чему равен Igieoo? 32. Чему равен lge>1 оо? 33. Чему равен lg10 0? 33. Чему равен lgoa 0? 34. Что больше: logu 15 или logjS 14? Если какое-либо число составлено из других чисел путем их перемно¬ жения, деления, возвышения в степень или извлечения корня, то логарифмы такого числа можно вычислить по логарифмам чисел, из которых оно со¬
Глаза XVI ставлено; напрчмер, логарифмы чисел ab и ~ можно вычислить по лога¬ рифмам чисел а и Ь. Процесс этот называется логарифмированием данного количества. Действие логарифмирования производится на основании следующих четырех теорем: 1) loga (MN) = loga М 4- loga N; 2) ]°g0 (]?) =lQSa М—1°ёа 3) loga (Л*) = k log„ N; 4) l°gaVN = jlQgaW. T- e.: 1) логарифм произведения равен сумме логарифмов сомножителей; 2) логарифм дроби равен разности между логарифмом числителя и ло¬ гарифмом знаменателя; 3) логарифм степени равен показателю степени, помноженному на лога¬ рифм основания степени; 4) логарифм корня равен логарифму подкоренного количества, делен¬ ному на показатель корня. 35. Выразить log 6 через log 2 и log 3. 35. Выразить log 21 через log 3 и log 7. 2 36. Выразить log 1^- через log 5 и log 3. д 36. Выразить log 2^- через log 13 и log 5. 37. Выразить log через log 5. 37. Выразить log 81 через log 3. 38. Выразить log |/11 через log 11. 38. Выразить log ^2 через log 2. * 39. Если основание логарифмов равно 3, то log 81 =4 и log 243 = 5. 81 Чему равны log (81 -243) и log ^ при том же основании? 39. Если основание логарифмов равно 2, то log 64 = 6 и log 1024 = 10. Чему равны log (1024-64) и log при том же основании? 40. Каких первоначальных чисел нужно знать логарифмы, чтобы найти 125 3/ 7 логарифмы при том же основании чисел: 24, у/38, "j/gg? 40. Каких первоначальных чисел нужно знать логарифмы, чтобы найти логарифмы при том же основании чисел: 18; 27; 36; 40; 50? 41. Зная, что lg 2 = 0,30103, lg 3 = 0,47712 и lg 5 = 0,69897, найти lg 6, lg 15, lg 30, lg 10, lg 1000. 41. Зная, что lg 2 = 0,30103, lg 5 = 0,69897 и lg 7 = 0,84510, найти lg 14, lg 35, lg 50, lg 100, lg 10 000. 1 2 2 42. При данных предыдущей задачи найти lg 2—, lg 1 lg ^, lg 0,6, lg 0,016.
Логарифмы 65 4 О с 42. При данных предыдущей задачи найти lg 2 у, lgy, lg ~, ]g 0,07, lg 0,0014. 43. При данных задачи 41 найти lg 20, lg 200, а также lg 15, lg 150, lg 1500. 43. При данных задачи 41 найти lg 70, lg 700, а также lg 35, lg 350, lg 3500. 44. При данных задачи 41 найти lg 0,3, lg 0,003, lg 0,06, lg 0,0006. 44. При данных задачи 41 найти lg 0,2, lg 0,002, lg 0,14, lg 0,0014. Прологарифмировать следующие выражения: 45. дс ;=2 аЬ. 45. х = 3 Ьс. 46. ab х — — . с 46. х=~. Ьс % 47. х = а3#*. 47. х = cPbc3. 48. а2 x~W>m 48. esfce х= с* 49. х = 2 (a-\-b). 49. jc = 5 (а — Ь). 50 3 V 50. а2— &2 V W* А а2 — Ь2 л 7 . 51. (а — Vfc X~\a+b)d' 51. Y_a(b + c) (b—c)2d‘ 52. Х — 5а*Ь /с". 52. х — 2 Ь / ас. 53. , /ш х=у-рг• 53. }/~аг х—У 2Ъ2с’ 54. х = 5а У а* (а — Ь). 54. х=8а8 У аф-\-с)\ 55. 2аЬ» х = ~. 55. х-^1. cVd cVd 56. 56. \ к 1000/ 57 I у 57. 1 V ■ UI • Л — г— • а” /Ь •Л> ““ ' ж— • a’yF 58. —ii/J- 58. x=Vi-Vi- 59. 2 2 х — а*Ья. 59. х = а~*Ь*. 60. 60. 61. x=V2V 6/15. 61. ■* = ]/ 3^21 /6- 62. J/~&b х у VF»' 62. б Сборник алгебраических задач ч. 11.
Глава XV III. 2Igx—1 = 0; = 1; ** = 10; x — j/lO (здесь нельзя брать радика; со знаком минус, так как при десятичном основании Отрицательное число логарифма не имеет). ' IV. 10-2* — 2а*=16; 2а*—10-2*+16 = 0; 2* = 5it[/25 — 16 =8 и 2; = 8, откуда хх = 3; 2*» = 2, откуда x8 = 1. 175. 10-*= 10 000. 176. 1Уах= |/ aSx¥3. 177. 16* =4-. 4 178. l-y& 179. (4И1Г- 180. У ax~l У alx~x У а*~3х— 1. 181. UL) т т- 182. а(1-*){*-2) = 1 # a* 183. у 256 = 4*. 184. 2х — 2*~a = 3. 185. 2*. 3х =.144. 186. 5xrl -f- 5* = 750. 187. 10<з-*)(4-*)_1оо. 188. =a+y/cF^ 189. 4УХ+i = 64-2*/x+1. 190. g(ac4-®—2)(3—x) _ 191. 2*W,7a+16,5_-8 j/~2m 192. ]/ 9i(*-i)-| _ у~зм 193. 92^ * = 778 688. 194. \ “~y 4096 = 2 У 32768. 195. 5**+^3125*rf =*+jj// 15625*+a. 196. ga*+4_33i.2.>-,-s> 197. qx + g*+1 = 2* + 2*+I + 2Xxa. 198. 51"* = 7X_1. 199. (4rV?=T<^-*- 200. 5a*_1 + 22x ■ 521 + 2,THa = 0. 201. Xх —X. 202. *1^ = (/л)х. 203. 3-2x=4pr9. 204. л->«*=10. 205. x'sx=:100x. 206. yx!g*-i = 100. 207. 10*= /3. 208. 5*= 17. 209. 10* = 200. 210. ©'=*• 211. 23*= 100. 212. 10*= J/ 2. 213. 52* = 0,1. 214. У 1,3713 = "/10. 215. 3* — 5x,a = 3*+* — 5*Л. 216. 7*_i + 7Z~- + 7*"8 = 5zl + 5*-a + 5*- 3 217. 218. / 0,35* = 0,00007882. 219. lgx=l —lg 3. 220. Ig* = lg24 — lg8. 221. lgx=31g 18 — 41g 12. Л 222. •g-г _-n 1 — lg2 A
Логарифмы 69 122. 3,52163-0,0272 0,21765 * 123. у у “/54321: 124. 0,0875 9,8304 125 126. 127. 128. 129. 1/. 78 К 0,0j7615' 1 / 2^118098 Г ЗУ1и_ ' 0,170586»-У 0,065613-Уо,1 51вУ 0,01059137 12,631 8,36 У 0,0067254 0,96578V 0,000035746 87.2853- У 75,846 У — 3,055 У136 00034 * /003425 1ои. у 0i0003 131. г^58 • 0,3599545-s. (~^322625 )~5 132. 133, (V 10732874)-»* 109 716 V 95п 404 ‘ 134. (9,8^ ]/1,6325)3. 135. О.ОООЭ0-0009. 136. 0,0376°-0376. 137. 7,V 0,4275. 138. 0,513Ь,/о-6Ш7. 122. 40,122.|,01133 0,98763 ' . I */23468. уЯ 124. 123 ,иОУо43' 125. 126. 127. 128. 129. ]/т- 0,0379 2,4548 |/ 0,261255 0,9-в.5120°-4 У2293Тб 10,599521°. г 00 678510 2,79У 0,0029745 0.79438 У0,000054237 29,3482- 1/ 93,594 V —2,743 >30. ущ 0,26758 У 0,4 006422 131. у 0,06084925 • 0,63_1°. I 132. (0,03029067- У4834,719)» 4 , “«■SyS- 3^ 134. (1,6224 3/4т:)5. 135. O.OOOZ0-0007. 136. 0.02890,0289. 137. 3'27// 0,2837. 138. 0,29342 7,/М12б. 139. Плошадь прямоугольного треугольника равна 282,14 см2\ один ка¬ тет его втрое больше другого. Найти катеты. 140. Объем прямоугольного параллелепипеда равен 385 м9. Измерения параллелепипеда относятся друг к другу как 3:5:8. Найти измерений па¬ раллелепипеда. 141. Объем правильной треугольной пирамиды равен 187,2 см9. Сторона его основания вдвое меньше ее высоты. Определить сторону основания. 142. Найти ребро куба, имеющего объем, в два раза больший объема куба с ребром 2,378 м. 143. Найти объем шара, поверхность которого равна 0,038 м®. 144. Найти радиус R основания конуса, имеющего объем V —36,785 dj£ и высоту H—R.
63 Глава XVI 93. Преобразовать в искусственную форму логарифмы: — 2,69537, — 4,21293, —0,54225, —1,68307, —3,53820, —5,89990. 93. Преобразовать в искусственную форму логарифмы: — 3,21729, — 1,73273, —5,42936, —0,51395, —2,43780, —4,22990. 94. Найти истинные значения логарифмов: 1,33278, 3,52798, 2,95426, 4,23725, 1,39420, 5,67990. 94. Найти истинные значения логарифмов: 2,45438, 1,73977, 3,91243, 5,12912, 2,83770, 4,28990. * Вычислить при помощи логарифмов следующие выражения: 95. 311-25,6. 96. 758-0,53. 97. 6603:213. 98. 3,264:0,078. 99. 23,5а. 00. 0,028а. 95. 4,51-215. 96. 0,037-269. 97. 8132:338. 98. 23,65:0,94. 99. 11,8*. 100. 0,0067*. 01. J/T2A 101. 1/23,2. 102. i/o,052. 102. ^0,61. 03 438-6-2,138 1 03 47,54-3,642 1ЛЛ 0,045-7-513 14,5-0.0178 -2,071-0.864 • * - 25,58 ’ 145,4 ‘ 2,071-0.864 ' 0,83-3,105 05. 'i/347567. 105. ^71,238з; 106. ^0,0643^. 106. ^О^РС 07. 5V/3TT866. 107. 27/277892: 108. /| * 108‘ SV^295* 09. 1,04,0°. 109. 2,083°. ПО. 1'][/Тб0. 110. *у/Ь0. 11. у/ОТО98756*7 111. ^0^8437*. 112.112. |/(^3)‘ 13. (8,53 7ю)з. , . /З8’\0-07/51\ам \277 (43) ' 15. У 0,006 ,/ 0,17624. 16. -|/ 0,4293 j/g. 17 - 0,73453-0,1643‘ 18. У 0,0541/0,0003617. 19. (f^ КТ) 20 21. 0,0045-7,5132 2,0719.0,864 * 13. (2,38 у/1 о) » . /25\°>03 /39\п.07 14Чт) У ■ 15. ]/ 0,893943/0ТШ1. 16. ]/^ »/0“,3798. 17 L 0,21272-0,921S- 18. У 0,0007 / 0,09342. ... уц. 20. 2739977”*, 11.51-0,017085 21. 0,78-34057
Логарифмы 69 122. 3.521 б3-0,0272 0,21785 ' 123. ^ у V 54321. 124. 125. 126. 127. 128. 0,0875 9,8304 78 ,Oj7615" ЗУ Jo 0,170586* -Уад 0,06561». VlU ’ 51° У 0,01059137 129. 12,631 8,36 V' 0,0067254 0,965783/0,000035746 87,2853 -1?/ 75,846 V—3,055 /0,034 * У о,( ,03425 /136 130. ,/ *‘,00034 131. ^58.0,3599545-". (~/322625 )"* 132. 133. (V 10732874)-*’ 109 V 95п 404' 716 г 134. (9,8л5/ 1,6325)3. 135. 0,0009°-°009. 136. 0,0376°*Ш76. 137. 7,06У 0,4275. 138. 0,513Ь,/о*69837. 122. 40,122-0,0113з 0.98763 ' 123. |/ i У /23468. 124. 0,0379 1 /ПИ V 0.U09 *43' 2,4548 125. у/ 0,261255 Vt 0,9-«-5120а4 126. 127. 128. V2293/6 10Л99521®. V 0$ 67851,3 2,79 V 0,0029745 0,79438 /0,000054237 ’ 129 29,34^. У 93,594 ■2,743 ion ,V 0,26758 Vu.4 ' [/ 0.006422 * 131. /0,06084925-0,63"10. I 132. (0,03029067- /4834,719)5 £ 134. (1,6224/4л)5. 135. 0,0007°-Ш7. 136. 0,02890*0289. 137. 3 27/ 0,2837. 138. 0,29342 7/°.«2б. 139. Площадь прямоугольного треугольника равна 282,14 см*; один ка¬ тет его втрое больше другого. Найти катеты. 140. Объем прямоугольного параллелепипеда равен 385 м*. Измерения параллелепипеда относятся друг к другу как 3:5:8. Найти измерения па¬ раллелепипеда. 141. Объем правильной треугольной пирамиды равен 187,2 см3. Сторона его основания вдвое меньше ее высоты. Определить сторону основания. 142. Найти ребро куба, имеющего объем, в два раза больший объема куба с ребром 2,378 м. 143. Найти объем шара, поверхность которого равна 0,038 м*. 144. Найти радиус R основания конуса, нмекицего объем 36,785 И высоту H=R.
«в Глава XVI 93. Преобразовать в искусственную форму логарифмы: — 2,69537, — 4,21293, —0,54225, —1,68307, —3,53820, —5,89990. 93. Преобразовать в искусственную форму логарифмы: — 3,21729, — 1,73273, —5,42936, —0,51395, —2,43780, —4,22990. 94. Найти истинные значения логарифмов: f,33278, 3,52798, 2,95426, 4,23725, 1,39420, 5,67990. 94. Найти истинные значения логарифмов: 2,45438, 1,73977, 3,91243, 5,12912, 2,83770, 4,28990. ■ Вычислить при помощи логарифмов следующие выражения: 95. 311-25,6. 96. 758-0,53. 97. 6603:213. 98. 3,264:0,078. 99. 23,5*. 00. 0,028*. 95. 4,51-215. 96. 0,037-269. 97. 8132:338. 98. 23,65:0,94. 99. 11,8*. 100. 0,0067». 01. /12,5. 101. /23^2. 102. ,70,052. 102. /0,61. по 438-6-2.138 1ЛО 47,54-3,642 1пл 0,045-7-513 tnA U ' 25,58 ' 145,4 * 2,071-0.864 ' 14.5-0,0178 0,83-3,105 * 05. '[/34,567. 105. /71,238*. 106. /0,06432. 106. p'OJo1’’. 07. 5'/3,1866. 107. 2'/2'7892~ ,ЛО ШУ ~ //" 21 69. 1,04100. 109. 2,0830. 110. 11. /6^09S7563T 111. /0,98437*. 13. (8,53 710)1. ] , . /38YW /51\о.°з У (43) * 1 15. / 0,006/0” 1*7624. 1 16. у 0,4293 л/\ 17 1 _ . '• 0,73453-о, 1642 ■ 18. У 0,054 [/0,0003617. .9. (*)“. »iШ- 0,0045-7,5132 19 34' «• mVl S/2- 00,- 110. 27/_50. 3. (2,38 /То) ? . . /25\о,оз /39\п,о7 4- (т) У ■ 5. /.0,89394"/0^92. 6. /-[/0,3798. 7 1 0,21272-0,9213- 8. р' 0,0007 / 0,09342. —3 9. 21. 2.0719-0,864 120. 0/2,39977-», 0 14,51 017085 1 ■ 0,78-3,1057 *
Логарифмы 65 4 2 5 42. При данных предыдущей задачи найти lg 2у, lg у, lg , lg 0,07, lg 0,0014. 43. При данных задачи 41 найти lg 20, lg 200, а также lg 15, lg 150, lg 1500. 43. При данных задачи 41 найти lg 70, lg 700, а также lg 35, lg 350, lg 3500. 44. При данных задачи 41 найти lg 0,3, lg 0,003, lg 0,06, lg 0,0006. 44. При данных задачи 41 найти lg 0,2, lg 0,002, lg 0,14, lg 0,0014. Прологарифмировать следующие выражения: 45. х = 2аЬ. 45. х = ЪЬс. 46. ab Х--~ с 46. а Х=Ъс- 47. х — а3Ь2. 47. х = а*Ьс*. 48. А2 Х~Ш' 48. *=-*" 49. х = 2{а-\-Ь). 49. х = Ъ{а — Ь). 50. 3 •*—д2_ fr2- 50. д2—Ь2 х- 7 * 51. (а — Vfc Х (a + b)d' 51. аф + с) x~{b~cfd' 52. х = 5а*Ь Ус. 52. х = 2Ь |/ ас. 53. „ /Шь Х=У~7Г' 53. х-l х~~ V 2ЬЧ' сл х — 5 а У а* (а — Ь). 54. х=Ы*У аф + с)\ 55. 2 аЬ» х=—=. cVd 55. a*Yb X— . cVd 56. 56. 57. 1 57. 1 X — . • Л — _ »— • а*уь 58. X=W^' 58. х= |/i- . 59. 1 з x = azb 59. х = а~‘1Ь3. 60. т fm\W хНп) • 60. 61. x=V 2.V 6/15. 61. х— у з fol у^Ъ- 62. х=л/%. у Vtf» 62. ,8/ a3 Vb Х~~\ 62 * б Сборник алгебраических задач ч. П.
72 ftiaea XVI III. 2 Ig x — 1 = 0; ~ = 1; x2 = 10; x = j/lO (здесь нельзя брать радикал со знаком минус, так как при десятичном основании отрицательное число логарифма не имеет). IV. 10-2*— 2** = 16; 22*— 10-2* + 16 = 0; 2*=5:±: j/25 —16 =8 и 2; 2*'= 8, откуда xt = 3; 2*» = 2, откуда х„ = 1. 175. 10-^= 10 000. 177. 16* = -^. т ФЧ?Г- ,81‘ Ш'= 12S- 183. /~256 = 4х. 185. 2®*3*= 144. 187. 10*3—*>(■*—*) = 100. 189. 4^й5 = 64.2,/1+1. 191. 2*1-7,7*+1в’5 = 8 /~2. 193. 92^* = 778 688. 195. 5-e+^3l25wl =e+/l5625*+a. 197. 6* + 6*+1 = 2* + 2*+1 + 21+а. 1". (4гУ4=т(^)-‘. 201. х*=х. 203. 3-2*= 4/9. 205. х’г*=100х. 207. 10*=/5. 209. 10* = 200. £11. 23*= 100. 213. 52* = 0,1. 215. 3*— 5*+* = 3*14 — 5*+*. 216. 7*-1 + 7*-а + 7*-» = 5*-1 + 5*-* 219. Igx=l — Ig3. 221. Igx= 31g 18 — 4 Ig 12. 176. /e^/e53. 178. ^/a*=*-/?. 180. 6*^=1. 182. at*—)(«-*) =1. a® 184. 2*—2*-* = 3. 186. 5*+1 + 5*= 750. 188. °~/с^=a+f/c^ 190. 5C«^+*—=90—^»= 1. 192. V9*(*_i)~| = У~3. 194. *~V 4096 = 2 У 32768. 196. 6**+i = 33*.2*V8. 198. б1-*^*-1. 200. 5s*-1+ 22* — 5a* + 2®*+a=0. 202. x»^=(/x)*. 204. xlg* = 10. 206. ^xI»*-1 = 100. 208. 5*= 17. 210. (|J=8. *212. 10»=/2. 218. /^35^=0,00007882. 220. Igx=lg24— lg 8. 214. /1,3713 = */10. + 5-».
Логарифмы 73 223. l-Ig5 = i-(lgi + IgJ; + -llg5). 224. lg(2Ar-|)-lgA:=Ig(ji:-3). 225. lg(x-J-) = 21g-g-. 226. Ig (д: — 2) = lg д; — lg 2. 227. lg(3*2 + 7)-Ig(3jc_2)=l. 228. Ig (* +/~3) = - lg (* - ^) . 229. ц-^-i. 230. |lg(*-9) + lg/2FTT = l. 231. Ig 10 + lg (271 + 3^21) = 2. 232. log0 Iog0*=Iogem + log0«. 233. logeIogex=logeIogew — Iogen. 234. a2x + c2 = 2baw. 235. 52*—5*= 600. 236. 32*+s=3*+» + 2. 237. 0,1 lg *x + 0.9 = lg 2x. 238. 8*+1 — 82*-1 = 30. 239.{/+/=1°* 240.{'^У=7- ( lg*—lg.y=lgl,6. (Ig* — IgJ' = 5. / 14*= 63к (x»=yx; ш- {17.=ет>. 242-{,-=y. 2И.(С“0 244. °'4”'=©’: • I > * • I 1,4”-»= 1,6565. (xV*=y; (х*Гх~У»=у1; 245. { __ ’ 246. { ^ ir I yV» = x1. I yVx—Vv=Xm 247. yVy — x1. Ky* гл? = 243; /Э*1 £/64 = 36; | j/ 1024 = (j xj. 248' \ 5» j/512 = 200. § 4. Задачи на сложные проценты. 249. В какую сумму обратится вклад в 246 руб., положенный в банк на 8 лет по 5°/0? 249. В какую сумму обратится вклад в 3768 руб., положенный в банк на 20 лет по 4%? 250. Сколько нужно внести в банк, платящий 6°/0 в год, чтобы через 20 лет иметь 8000 руб.? 250. Сколько нужно внести в банк, платящий 3°/0 в год, чтобы через 12 лет иметь 6720 руб.? 251. Через сколько лет капитал в 20 728 руб. обратится в 50 000 руб., считая по 4 ~ °/0? 251. Через сколько лет вклад в 18 978 руб, обратится в 48 593 руб, считая по 7 -i- °/Q?
80 Глава XX 19. — 3<5, — 5<2. 20. — 13 <— 7, -9>-15. Разделить неравенства: 21. 35<40, 7>5. 22. -6<4, 3>2. 23 — 24 — !> — — — <" — -so. 4 ^ 9 * 7 3 " 5 ^ 3 ' 18 9 ’ * Решить неравенства: 25. jc+4>2 — Злг. 25. 3 + 5л:<7л; + 4. 26. 4 (л;— 1)>2 + 7лг. 26. 3(jc—2)<4л; —9. 27- 7-Т<4*-3- 27' Т-4»Т-4Л 28. 28. 3-^>|— *2=$. 29. (л: — 3)* + 7>(х + 4)’*. 29. (1 + xf + Зл*<(2jc- 1)* + 7. 30. ^=^+l2<^i-?-i0x. 30. 8+^£zii>i=d_§£zi3. Определить, при каких значениях х ниже написанные выражения поло¬ жительны: 31. 2jc-16. 32. 5 —Зх. 33. -х-4. 34. - 2л; + 2-1-. 5— х 3+2х ' 35. -g- + —4~. Определить, при каких значениях д: ниже написанные выражения отри¬ цательны: 36. Здг+15. 37. 7-14л:. 38. 5--|л;. 39. х~2 ' к Решить совокупные неравенства: 41. 2л; > 4л: + 6 и 4л; + 3 < 2л: + 1* 42. Зх + 7> 7х—9 и х—3]> —Зл:+ 1. 43. 5л; — 3>1+л; и — — Зх<^~х— 5. 44. 4лс -f- 7 ^ 2л: -|-13 и Зх — 18 2л: -1-1. 45. 6л; — 7^> 5х — 1 и Зл; + 6 8л: — 4. 46. 2 (х—3)— 1 <5 и ®5_7>^. 47. Зх + 2>х — 2, х + 15]>6 — 2х и х— 14 <^5х -)- 14.
Соединения 77 20. Сколько нужно взять предметов, чтобы число сочетаний из них по 3 относилось к числу сочетаний по 5 как 2:3? 21. Число сочетаний из л элементов по 3 в 5 раз меньше числа соче¬ таний из и + 2 элементов по 4. Найти л. 21. Число размещений из л элементов по 5 в 18 раз больше числа размещений из л — 2 элементов по 4. Найти л. 22. Число сочетаний из 2 элементов по л + 1 относится к числу соче¬ таний из 2л+1 элементов по л—1 как 3:5. Найти л. 22. Число сочетаний из 2л элементов по л—1 относится к числу соче¬ таний из 2л — 2 элементов по л как 77:20. Найти л. 23. Показать, что непосредственное определение числа парных сочета¬ ний приводится к суммированию разностной прогрессии. - 23. Показать, что непосредственное определение числа тройных сочетаний приводится к суммированию ряда парных произведений. 24. Между перестановками цифр числа 12 345 сколько таких, которые начинаются цифрой 1? числом 12? числом 123? 25. Между сочетаниями из 10 букв а, Ъ, с... по 4 сколько таких, которые содержат букву с? буквы а и Ы 26. Между размещениями из 12 букв а, Ьл с... по 5 сколько таких, которые содержат букву а? буквы о и Й 27. Между сочетаниями из h букв по к сколько таких, из которых каждое содержит л определенных букв? 28. Между размещениями из h букв по k сколько таких, из которых каждое содержит л определенных букв? 29. При каких и скольких значениях k существует неравенство с^сс*? 30. Показать, что при четном л в ряде чисел сочетаний CJ, СЦ... dP имеется одно среднее, наибольшее из всех число. t ГЛАВА XVIII. БИНОМ НЬЮТОНА. Формула бинома Ньютона такова: (х + а”) = хК + у ахп~х я ^ а2*"-5 + ... + + —n1~1^Qw~ajc2 + у ап~Кх + а". Выражение общего члена бинома Ньютона: Найти сокращенным путем произведения двучленов: 1. (л: + 1)(л; + 2)(д; + 3)(д; + 4). 1. (х— 1)(х — 2)(х — 4)(лг — 5). 2. (х—1)(л;+3)(д; — 4)(л: + 5). 2. (х + 2)(л;-3)(х + 4)(х-6). 3. (х+ 1)(х+2)(л;+3)(д: + 4)(л: + 5). 3. (х—1)(jc — 2)(jc—3)(л: — 4)(д;—5). 4. (л: — 2) (д; + 3) (х — 4) (л: + 5) (х — 6). 4. (л: + 2) (х — 3) (л: — 4) (л: + 5) (х — 6).
% Глава XVII 5. Составить сочетания всех видов из пяти элементов. 6. Составить посредством сочетаний размещения всех видов из трех эле¬ ментов. 6. Составить посредством сочетаний размещения всех видов из четырех элементов. 7. Выразить арифметические числа: Л®, Я„, С\. 7. Выразить арифметические числа: Л|, Рв, С,0. 8. Выразить арифметические числа: Р8, Л,7.,, С/,. 8. Выразить арифметические числа: Р „ Л,95, Cj78. 9. Выразить число размещений из я+1 элементов по ft — 1 в каждом размещении. 9. Выразить число размещений из л — 2 элементов по k + 1 в каждом размещении. 10. Выразить число размещений из те + я элементов по те — я+1 в каждом размещении. 10. Выразить число размещений из те—я элементов по те — 2л—1 в каждом размещении. 11. Проверить равенства: = и С,7 =С,5г. 11. Проверить равенства: С8 = С| и = С185. 12. Проверить равенства: с\ + С® = С* и С160-1-С150 = С1в|. 12. Проверить равенства: с\ + С* — и С1в, + С1\ = С1в8. 13. Выразить число сочетаний из л+ 2 элементов по ft—1 в каждом сочетании. 13. Выразить число сочетаний из л—1 элементов по ft+ 2 в каждом сочетании. 14. Выразить число сочетаний из те — л элементов по л+1 в каждом сочетании. 14. Выразить число сочетаний из те + л элементов по л — 2 в каждом сочетании. 15. Сколькими способами можно рассадить за столом четырех человек? 15. Сколькими способами можно рассадить за столом пять человек? 16. Сколькими способами можно составить четырехцветные ленты из семи лент различных цветов? 16. Сколько различных трехзначных чисел можно написать посредством девяти цифр? 17. Сколькими способами можно выбрать четырех лиц на четыре раз¬ личные должности из девяти кандидатов на эти должности? 17. Сколькими способами можно выбрать четырех лиц на четыре одина¬ ковые должности из девяти кандидатов на эти должности? 18. Сколько прямых линий можно провести между десятью точ¬ ками, расположенными так, что никакие три из них не лежат на одной прямой? 18. Сколько окружностей можно провести между десятью точками, расположенными так, что никакие четыре из них не лежат на одной окружности? , 19. Из скольких предметов можно составить 210 размещений по два предмета в каждом? 19. Из скольких предметов можно составить 66 различных пар? 20. Сколько нужно взять предметов, чтобы число размещений из них по 4 было в 12 раз больше числа размещений по 2?
Соединения 20. Сколько нужно взять предметов, чтобы число сочетаний из них по 3 относилось к числу сочетаний по 5 как 2:3? 21. Число сочетаний из я элементов по 3 в 5 раз меньше числа соче¬ таний из л+ 2 элементов по 4. Найти я. 21. Число размещений из л элементов по 5 в 18 раз больше числа размещений из л — 2 элементов по 4. Найти л. 22. Число сочетаний из 2 элементов по л +1 относится к числу соче¬ таний из 2л + 1 элементов по п—1 как 3:5. Найти л. 22. Число сочетаний из 2л элементов по п—1 относится к числу соче¬ таний из 2л — 2 элементов по л как 77:20. Найти л. 23. Показать, что непосредственное определение числа парных сочета¬ ний приводится к суммированию разностной прогрессии. 23. Показать, что непосредственное определение числа тройных сочетаний приводится к суммированию ряда парных произведений. 24. Между перестановками цифр числа 12 345 сколько таких, которые начинаются цифрой 1? числом 12? числом 123? 25. Между сочетаниями из 10 букв а, Ъ, с... по 4 сколько таких, которые содержат букву а? буквы а и А? 26. Между размещениями из 12 букв а, Ь, с... по 5 сколько таких, которые содержат букву с? буквы а и А? 27. Между сочетаниями из Л букв по k сколько таких, из которых каждое содержит л определенных букв? 28. Между размещениями из А букв по А сколько таких, из которых каждое содержит л определенных букв? 29. При каких и скольких значениях k существует неравенство ci-i<c!I? 30. Показать, что при четном я в ряде чисел сочетаний С,1,, С*... С^-1 имеется одно среднее, наибольшее из всех число. ГЛАВА XVIII. БИНОМ НЬЮТОНА. Формула бинома Ньютона такова: (х + ан) = х" + ~ ах*~1 + п ^ - а2*"-5 + ... + + П(-П.~п1)ая~2х2 + 4 я"'1* + а". 1 “2 I Выражение общего члена бинома Ньютона: Найти сокращенным путем произведения двучленов: 1. (jc+ 1)(jc+2)(at + 3)(jc + 4). 1. (x—l)(x—2)(x—4)(x — 5). 2. (х—1)(д:4-3)(д:—4)(д; + 5). 2. (* + 2)(*-3)(* + 4)(х-6). 3. (jc+ 1)(л:+2)(л: + 3)(л:-1-4)(л: + 5). 3. (х—1)(х — 2)(х—3)(х—4)(х—5). 4. (х—2)(х+3)(х —4)(х+5)(х —6). 4. (х + 2) (х—3) (х — 4) (х + 5) (х — 6).
% Глава XVIll 5. Составить сочетания всех видов из пяти элементов. 6. Составить посредством сочетаний размещения всех видов из трех эле¬ ментов. 6. Составить посредством сочетаний размещения всех видов из четырех* элементов. 7. Выразить арифметические числа: A*, Pt, CJ. 7. Выразить арифметические числа: А*й, Р„, С^0. 8. Выразить арифметические числа: Р3, /З,7,, С,®х. . 8. Выразить арифметические числа: Ри, АД, С{3. 9. Выразить число размещений из /г -j- 1 элементов по k— 1 в каждом] размещении. • 9. Выразить число размещений из я — 2 элементов по k + 1 в каждом размещении. 10. Выразить число размещений из т + я элементов по яг— я+1 в каждом размещении. 10. Выразить число размещений из яг — я элементов по гя — 2я — 1 в каждом размещении. 11. Проверить равенства: С* = С® и С,7г=С,5г. 11. Проверить равенства: С| = С§ и С175 = С18Ь. 12. Проверить равенства: С\ + Cf. = С* и = С,6,. 12. Проверить равенства: С*+С* — С® и С,6, + C,5Z = С®8. 13- Выразить число сочетаний из я+2 элементов по k—1 в каждом сочетании. . 13. Выразить число сочетаний из я—1 элементов по k + 2 в каждом сочетании. 14. Выразить число сочетаний из яг — я элементов по я+1 в каждом сочетании. 14. Выразить число сочетаний из яг + я элементов по я—2 в каждом сочетании. - 15. Сколькими способами можно рассадить за столом четырех человек? 15. Сколькими способами можно рассадить за столом пять человек? 16. Сколькими способами можно составить четырехцветные ленты из семи лент различных цветов? 16. Сколько различных трехзначных чисел можно написать посредством девяти цифр? 17. Сколькими способами можно выбрать четырех лиц на четыре раз¬ личные должности из девяти кандидатов на эти должности? 17. Сколькими способами можно выбрать четырех лиц на четыре одина¬ ковые должности из девяти кандидатов на эти должности? 18. Сколько прямых линий можно провести между десятью точ¬ ками, расположенными так, что никакие три из них не лежат на одной прямой? 18. Сколько окружностей можно провести между десятью точками, расположенными так, что никакие четыре из них не лежат на одной окружности? , 19. Из скольких предметов можно составить 210 размещений по два предмета в каждом? • 19. Из скольких предметов можно составить 66 различных пар? 20. Сколько нужно взять предметов, чтобы число размещений из них по 4 было в 12 раз больше числа размещений по 2?
Логарифмы 73 223. l-Ig5 = i(lgi + IgJ: + -ilg5). 224. lg(2x — -^—lgx=Ig(x—3J. 225. lg(x — |-) = 21g~. 226. Ig(x 2) = lgx lg2. 227. lg(3x* + 7)-Ig(3x-2) = I. 228. lg (x + /~3) = - Ig (x - /~з) 229. lg't?-+-i-)= ~L 230. |lg(*-9) + Ig/2^1 =1. 231. Ig 10 + -j lg (271 + 3l/2*) = 2. 232. Iogfllogex=log0wz + loge«. 233. IogaIogox=Iogaloge/7z —Ioge/z. 234. a*x+ с*=2Ьах. 235. 52z—5r=600. 236. 32г-я=3*^ + 2. 237. 0,llg*x + 0,9 = lg2x. 238. 8*+1 — 8s*-1 = 30. ooq / 5x + 2y = 100; ngx + lgy=7; * I lg*-lgj> = lg 1,6. 4 \ lgx-lgj = 5. / I4*=63jr, ( 24,-{l7*=87£ 242‘{ x*=yx; x*—y3. 245 2M \ 1,4-*= 1,6565. ( x^^y; (x*/*~l/2=yl; (.j^s^x*. I yV*~Vs = x. i Xs* = 243; , 3y */64 = 36; 247. < y/ irio-.- r2 \e 248. S - ^f/ 1024 = (jx] . I 5*^512 = 200. § 4. Задачи на сложные проценты. 249. В какую сумму обратится вклад в 246 руб., положенный в банк на 8 лет по 5“/0? 249. В какую сумму обратится вклад в 3768 руб., положенный в банк на 20 лет по 4%? 250. Сколько нужно внести в банк, платящий 6% в год, чтобы через 20 лет иметь 8000 руб.? 250. Сколько нужно внести в банк, платящий 3% в год, чтобы через 12 лет иметь 6720 руб.? 251. Через сколько лет капитал в 20 728 руб. обратится в 50 000 руб., считая по4-^-°/о? 251. Через сколько лет вклад в 18 978 руб. обратится в 48 593 руб.. считая по 7-j? %?
80 Глава XX 19. —3<5, —5<2. 20. -13 <-7, -9>-15. Разделить неравенства: 21. 35<40, 7>5. 22. -6 <4, 3>2. 23 ——24 — 4 у » 7 з • 5 3 ' 18 9 * Решить неравенства: 25. х+4>2 — Зх. 25. 3 + 5х<7х + 4. 26. 4(х—1)>2 + 7х. 26. 3(х — 2)<4х— 9. 27. |-|-<4*_3. 27. 4-з1>1|—|*. 28. —^—+ 9<^=^-х. 28. 29. (х - 3)а + 7 > (х + 4)*. 29. (1 + х)а + Зха < (2х -1)* + 7. 30. ^=^+12 <^±---1 Ох. 30. 8+^—?>f=i_5£=l3. Определить, при каких значениях х ниже написанные выражения поло¬ жительны: 31. 2х-16. 32. 5 —Зх. 33. §х-4. 34. -2х + 2-^. ое 5—х 3+2х 35. g I 4 • Определить, при каких значениях х ниже написанные выражения отри¬ цательны: 36. Зх+15. 37. 7-14х. 38. 5-|-х. 39. + . 40. —jp— — + 2. « Решить совокупные неравенства: 41. 2х>4х + 6 и 4х+3<2х+1. 42. Зх + 7 > 7х — 9 и х — 3 > — Зх + 1. 43. 5х — 3>1 + х и -i- — Зх<-|-х —5. 44. 4х + 7>2х+13 и Зх—18<2х + 1. 45- 6х — 7^>5х—1 и Зх+6>8х — 4. 46. 2(х—3)— 1 <5 и ^-7>^. 47- Зх + 2>х — 2, х + 15>6 — 2х и х— 14<5х+14.
Решение неопределенных уравнений первой степени 81 Определить, при каких значениях а ниже написанные дроби положи¬ тельны: 2а—3 .а За —8 2 —За Зг — 7 48-<га* 48‘ Ъ-=7Г- 2а+~7' 49-2=55- Определить, при каких значениях а ниже написанные дроби отрицательны: гп 8 —Зд е1 5д + 8 °и* 7а —-2' ° Зт —7‘ 52. На основании неравенства {а — b f 0 доказать, что сумма квзл- ра'ов двух чисел всегда больше удвоенного произведения тех же чисел. 53. Доказать, что правильная дробь увеличивается от прибавления к членам ее одного и того же положительного числа. 54. Доказать, что неправильная дробь уменьшается от прибавления к членам ее одного и того же положительного числа. '55. Доказать, что среднее арифметическое двух чисел больше среднего геометрического между ними. 56. Доказать, что во всяком треугольнике полупериметр больше каждой из сторон. 57. Доказать, чго во всяком прямоугольном треугольнике высота, опу¬ щенная на гипотенузу, меньше половины гипотенузы. 58. Доказать, что во всяком прямоугольном треугольнике квадрат удвоен¬ ной высоты, опущенной на гипотенузу, меньше суммы квадрата гипотенузы с удвоенным произведением катетов. 59. Город N находится между двумя угольными районами. В первом жирный каменный уголь с теплотворной способностью р (8000 калорий), во втором тощий уголь с теплотворной способностью q (3500 калорий), q р. Стоимость тонны угля I района с погрузкой а руб. (14), доставка до N обходится т руб. (16) с тонны. Для II района расходы соответственно b руб. (7) и п руб. (12). При каком значении п выгодней получить городу N уголь из II района? 60. Пусть N руб. — стоимость сельскохозяйственного орудия, Р руб. — общая стоимость починки, п — число возможных случаев употребления ору¬ дия без ремонта, т — число возможных случаев употребления орудия при наличии ремонта. При каком условии производство ремонта себя оправдает? ГЛАВА XXI. РЕШЕНИЕ НЕОПРЕДЕЛЕННЫХ УРАВНЕНИЙ ■ ПЕРВОЙ СТЕПЕНИ. Решить следующие уравнения в целых числах способом подстановок: 1. х-Ь2_у = 7. 2. у — 5jc = 12. 3. Зх — 5у=0. 4. 5х+8_у = 0. 5. 2х + 3у=13. 6. Ъу — 7х = 21. 7. 7х+13_у = 71. 8. 14х—9у=11. Решить следующие уравнения в целых числах способом последователь¬ ных делений: 9. 2x+3jy = 7. 10. Зх — 4у=11. 11. 5х + 3у=6. 12. 7х — 4у — 3. 13. 7х + 5v= 12. 14. 5х —11у = 4. 15. 11х + 8у = 73. 16. 11 х — 7jv = —31. 6 Сборник алгебраических задач, ч. /I,
Глава XXI Могут ли быть решены в целых и положительных числах следующие уравнения: 17. 2x+6jy = 25. 18. 6л+11у =—48. 19. 8л + 7у=3. 20. 9л— 6у = 17. 21. Юл+13.у=16. 22. 13л —15j> = 45. 23. 8л + 6у=12. 24. 15jc — 10у = 25. Следующие уравнения решить в целых и положительных числах: 25. 4л: 4- lljy = 47. 26. 12л:— 7у = 45. 27. 11л+18у/= 120. 28. 15л— 49у— 11. 29. 18л —35_у = 30. 30. 45л+27у=117. 31. | + | = 37. 32. *-±.^=-20. 33. 3^ = 2^=^.. Найти наименьшие положительные числа, удовлетворяющие следующим уравнениям: 34. 17л —29j/=lC0. 35. 13л—15у = 2. 36. 52л + 64у; = 388' 37. 16л — 25у=1. 38. 41л — 36j/=187. 39. 9л.+20у = 547. Решить в целых положительных числах следующие системы уравнений: 40. 2л — 5у=5, 2у — Зл=1.** 41. 8л-5у = 6, 7z + 3_у = 13. 42. Зл+jy+д= 14, 5л + 3з>+г = 28. 43. 4л-i-v + 3г = 30, 7л+,у+6д = 51. 44. л= 5у + 3= llz+ 7. 45. л + 2у + 3г = 20, Зл + 5_у + 4г = 37. 46. 2л+14у— 7.г= 341, 10л + 4з» + 92г=473. 47. Разложить число 200 на два слагаемых, из которых одно делилось бы без остатка на 7, а другое на 13. 48. Сколькими и какими способами можно заплатить 149 руб., имея билеты по 3 руб. и по 5 руб.? 49. Найти два числа, разность которых 10, зная, что уменьшаемое кратно 8, а вычитаемое кратно 17. ’ 50. Сколькими и какими способами можтСо взвесить груз в 114 кг, имея гири в 5 и 3 кг} 51. Сколькими и какими способами можно взвесить груз в 87 кг, имея гири в 5 и 2 кг} 52. Двум артелям рабочих выдано 330 руб. Каждый рабочий первой артели получил 16 руб., а каждый рабочий второй — 9 руб. Сколько было рабочих в каждой артели? 53. Сколько можно поместить пятикопеечных и двухкопеечных монет (бронзовых) на протяжении метра, полагая, что диаметр первых равен 25 мм, а диаметр вторых 18 мм} 7 54. Дробь (-g равна разности двух дробей, из которых у одной знамена¬ тель 9, а у другой 13. Найти эти дроби. 1 55. При каком значении х дробь —— обращается в положительное чет¬ ное число?
Непрерывные дроби 83 56. Найти общий вид чисел, кратных 5, которые при делении на 8 дают в остатке 1. 57. Найти общий вид чисел, кратных 7, которые при делении на 5 дают в остатке 2. О jx 58. При каком значении х дробь ——— обращается в положительное число, делящееся на 4 с остатком 3? 59. Найти общий вид чисел, которые при делении на 3 дают в остатке 2, а при делении на 7 дают в остатке 3. 60. Найти общий вид чисел, которые при делении на 7 дают в остат¬ ке 4, а при делении на 8 дают в остатке 3. 61. Стрелок за каждый удачный выстрел получает по 8 коп., а за каж¬ дый неудачный сам платит по 27 коп. Сделав некоторое число выстрелов, меньше 120, он выручил 97 коп. Сколько было удачных выстрелов и сколько неудачных? 62. При вращении двух зацепляющихся зубчатых колес, из которых одно имеет 19 зубцов, а другое 23, первый зубец одного колеса попал в первый промежуток другого. Сколько полных оборотов должны сделать оба колеса, чтобы первый зубец попал опять в первый промежуток, сколь¬ ко, чтобы попал во второй промежуток, в третий и т. д.? 63. При вращении двух зацепляющихся зубчатых колес, из которых одно имеет 25 зубцов, а другое 36, первый зубец одного колеса попал в первый промежуток другого. Сколько полных оборотов должны сделать оба колеса, чтобы первый зубец попал опять в первый промежуток, сколь¬ ко, чтобы попал во второй промежуток, в третий и т. д.? 64. Найти трехзначное число, сумма цифр которого 16; если к этому числу прибавить 99, то получится число, обозначенное теми же цифрами в обратном порядке их. 65. Найти наименьшее из чисел, которые при делении на 3, 4, 5 дают в остатках 1, 2 и 3. 66. Найти общий вид чисел, которые, будучи кратны 5, при делении на 8, 11 и 3 дают остатки 1, 3 и 1. 67. Найти наименьшее из чисел, которые при делении на 5, 6, 7 и 8 дают остатки 3, 1, 0 и 5. ' 68- Заплатить 25 коп. монетами по 2, 3 и 5 копеек. ГЛАВА XXII. НЕПРЕРЫВНЫЕ ДРОБИ. Обратить следующие непрерывные дроби в простые: 1. (2, 1,2, 3, 2...). 2. (2, 3, 1, 1, 1, 2...). 3. (0, 2, 1, 4, 3, 2...). 4.. (а, b, а, Ь, а...). 5. (0, х, 3.v, х, 2х...). 6.(а— 1, а, я+ 1, а...). Обратить следующие простые дроби в непрерывные:
84 Глава XXIII Найти приближения к следующим непрерывным дробям и вычислить пределы ошибок этих приближений: 10 99 ,9 51 ,о 55 14 463 л. 1702 685 *~ 239' 117’ 89' 640' 3919' 126' Найти приближения к бесконечным непрерывным дробям и определить пределы их ошибок: 15. (1, 3, 5, 7, 9, 11...). 15. (2, 4, 6, 8, 10, 12...). 16. (0, 10, 100, 1000...). 16. (0, 5, 50...). Обратить следующие корни в непрерывные дроби: 17. V2. 17. 1/5Г_ 18. l/^ 18' 19. 1/201 19. |/12. 20. i/7. 20. |/13. 21. ]/аа + 1- 21. |/аа + 2. Обратить следующие дроби в иррациональные выражения: 22. (4, 8, 8, 8...). 22. (5, 10, 10, 10...). 23. (3, 1, 6, 1, 6...). 23. (3, 2, 6, 2, 6...). 24. (0, 2, 3, 2, 3, 2...). 24. (0, 1, 2, 1, 2...). 25. (с, 2, 2а, 2, 2а...). 25. (а, 1, 2а, 1, 2а...). Решить в целых числах неопределенные уравнения: 26. 8лг + 13jv = 1 - 26. 7jc + 12jv = 1 - 27. 9лг— 14jy = 3. 27. Юдг— 17у = 2. 28. 23х+16у = 2. 28. 41x+29y=l. 29. 7дг—1Гу=1. 29. 17jc — 25jy=3. 30. 49дг + 34у/ = 6. 30. 29х+17jy = 25. Разложить в непрерывные дроби и вычислить приближенно следующие логарифмы: 31. 10*= 500. 31. 10*= 800. 32. 10*= 4000. 32. 10*= 60. 33. 72*= 432. 33. 36* = 432. 34. 50*= 500. 34. 75* = 375. Разложить в непрерывные дроби и вычислить приближенно корни урав¬ нений: 35. х3 — 2х — 5 = 0. 35. х9 — х — 3 = 0. 36. д:3 + лга + х — 1 = 0. 36. х3 + х% + х — 2 = 0. ГЛАВА XXIII. ИССЛЕДОВАНИЕ УРАВНЕНИЙ. § 1. Исследование уравнений 1-й степени с одним неизвестным. Определить, при каких значениях нижеследующие уравнения имеют по¬ ложительные решения: I. 5(дг —3) = 3(3х —2а). 2. 3(х+1) = 4 + ах.
Исследование уравнений 85 Определить, при каких значениях нижеследующие уравнения имеют отрицательные решения: 2 я о 5. 7 — а.—-—г. 6. лг—1* 4х— а ах—5' Нижеследующие уравнения, имеющие отрицательные решения, изменить так, чтобы решения их сделались положительными: 7. Ах — 75 = 6(* — 10) + 85. 8. 13* — 22 = 17(х~2) + 28. 9. 5(3 —7*) + 4(3* —7) = 35 + х. 10. 6(*— 1)— 12дг=12(дг+3) — 2(* + 5). Исследовать, пои каких значениях буквенных количеств, входящих в нижеследующие уравнения, эти уравнения имеют положительные, отрица¬ тельные, нулевые, бесконечные и неопределенные решения: .11. -а-=т . 12. 3ax + b = b(a + x). а—х п » \ | / 13. ах + т = Ь{х + п). 14. ^гт1£ = j - 15. Две партии рабочих получили вместе 120 руб.; каждый рабочий первой партии получил 7 руб., а каждый рабочий второй — 5 руб.; во вто¬ рой партии было тремя рабочими больше, чем в первой. Сколько было рабочих в каждой партии? 16. Определить двузначное число, в котором число десятков вдвоу меньше числа простых единиц, а разность между числами единиц и десят¬ ков составляет 6. Решить и исследовать следующие общие задачи, приводящие к буквен¬ ным уравнениям: 17. В одном резервуаре налито а ведер, в другом b ведер воды. Каж¬ дый час прибавляется в первый по т ведер, а во второй по п ведер. Через сколько часов количества ведер в обоих резервуарах сравняются? 18. Отцу а лет, сыну b лет. Через сколько лет отец будет в k раз старше сына? 19. Какое число нужно вычесть из чисел а и b для того, чтобы от¬ ношение разностей оказалось равным А? 20. Переднее колесо повозки имеет в окружности а метров, заднее Ь метров. Как велик путь, на котором переднее колесо сделает одним обо¬ ротом больше заднего? 21. Какое число нужно приложить к числителю и знаменателю дроби ~, чтобы она обратилась в дробь ~ ? 22. В а литрах воды растворено Ь граммов соли; сколько нужно при¬ бавить воды, чтобы на каждый литр приходилось т. граммов соли? 23. Разложить число а на две части так, чтобы сумма произведений первой части на т и второй на п была равна сумме произведений первой части на р и второй, на q. 24. В треугольнике ABC даны стороны: АВ — с, АС = Ь и ВС = а. Проведя равноделящую внешнего угла при вершине С, отметить точку пере¬ сечения этой равноделящей с продолжением стороны АВ. Определить рас¬ стояние AD.
86 Глава XXJVJ § 2. Исследование системы уравнений 1-й степени j с двумя неизвестными. 25. Определить, при каких значениях а система уравнений х+у = а\ и Зх+2_у=10 дает положительные решения. 26. Определить, при каких значениях а система уравнений 4х— Зу=( и — 5х 4- ау~ 8 дает отрицательные решения. 27. Определить значение а, при котором система уравнений Зх—-7у=15] и 46х + ау = б0 не имеет решений. 28. Определить значение а, при котором система уравнений 2х + 5_у = 71 и 7х — оу ~ 9 не имеет решений. 29. Определить значения а и Ь, при которых система уравнений ах—by = 151 и 4х + Ьу = 2 имеет бесчисленное множество решений. 30. Определить значения а и Ь, при которых система уравнений ах—у — Ь ' и 4х +3^=10 имеет бесчисленное множество решений. § 3. Исследование уравнений 2-й степени. В нижеследующих задачах определить условия, при которых корни уравнений будут действительными и положительными, а также подыскать для корней некоторые соизмеримые целые значения, соответствующие частным предположениям. 31. Найти два числа, сумма которых а, а произведение Ь. 32. В данный квадрат, сторона которого а, вписать другой квадрат, сторона которого Ь. 33. По данной гипотенузе а построить прямоугольный треугольник, равновеликий квадрату, сторона которого Ь. 34. Дан круг радиуса R и вне его точка на расстоянии d от центра. Провести через эту точку секущую к кругу так, чтобы ее внутренний отрезок равнялся радиусу круга. 35. Вписать в круг радиуса R прямоугольник, площадь которого была бы равна площади квадрата со стороною к- В нижеследующих уравнениях 2-й степени с двумя неизвестными требуется определить те действительные значения переменного х, при ко¬ торых переменное у также действительно: 36. х* + у®—2ху=0. 37. 2х®—2ху +Уг + 2х—4у + 1 = 0. ГЛАВА XXIV. ПРЕДЕЛЫ. Найти пределы: t 1 1 • 5 о I* 6х—*1 о 1* ^—Здг4 • — ,1ГП*->0 5Х + 3 ~ * *->2 JC + 4 ■ л г- ■*2—1 е .• ** + 3*2 с в2 — 4 4. lirrw j • 5. 1нпв^0 xz 4-^9 + 2*2 ' о-»2 в —2*
Исследование уравнений 81 7. „т ,^^+4 дф-о» <=-*■1 *■* + * — 2 *-♦* д2 — 9 *» *-«*2—а2 -п Злг+1 и | 5лгг—I lo . 2jcs + I 10. tlfflwo3jcf 2 • imjf-^co 4д:2^-3 • *2. ,]тЛ_»0О xi + 3 ' <п яд: 4-А 11 .. адг*+А 13- 1тх-*со сх-\- <1' ' 1т ®-*оо сд-з -f- rf * Найти пределы сумм членов бесконечно убывающих прогрессий: 15. 2 + 1+2Г + -4-+... 16. 1+-^-Ьд + ... 17. 2 — 1+1-1+... 18. 2 + 1/2+... 19.0,3333... 20. 2,3(4). Если Л приближается к нулю, то какие из следующих величин будут бесконечно малыми? 21. 12h-tVh 0,0001Л. 22. л3. 23. 2Л; 10А; lg Л. 24. sin Л; cos A; tg A; ctg А. К каким пределам стремятся следующие выражения при х->0? „ , ддс+й яд:2 + b ах2 + Ъ 25. Ъх', ax + b, сх+аcxz±d' £х< + d' 26. ах—\\ ах—2; lg*; lg(5 — *);
1. 4. 5. 6. 59. &**{/**. с2 \ Ь 63.V375. 67. Уб25й. 71. V от*п. 75. Уот* —1. 2. — в*. 6. 6. ОТВЕТЫ. Глава IX. з. 7. ю. 14. 18. 22. 26.- 30. -Зя«Ь. 1_ а1 * 5№ 2 а"' 100 10а’>;/" 4с”~* 34. -3 V 4. 38. - 2 V 7. 42. 7*bVb. 46. 4x?yVy. 6* ' Va— b 50. 54. 57. 2я*ЬсР 7/26"Л 60. Iх 12. 64. Vi 60. 68. Vmbi. ' a 4' и I 78. ™/3m+,amn+,bm+l. 82. bVa*. 86. УвоЙЛ 7Э- —' 83. j/ab*. 87 V ibW 4. a*. 11. 15. 19. 23. 27. 31. №• s' _1_ 3' —2. 5 A" 2 3a2b 26V ’ 16b2 (я-I-b)* a* 2V2 8. 12. a* 63' 3 2 ' i6. 2a’"6> 20. 24. 3 28b«rf «*c3 28. 4я3Ь462. 32. 5V3. 2 V 3. 36. 5V 2 18V 5. 40. 2V3. xyVxyZ. 44. abVab2. 12c°-dV 5. 48. a2V a. ЗУбг 59 10a2 xy2z~ . 65. 69. V180. VSa2. VanK n 62. V54. 66. V2x>. 70. I 9a3x. 73. 77. V 24a‘bc«. 74. 1/5. 80. 84. У a2. ьУзЖ ,VE I 9b*a2 * 81. nbV ab*j 85. Уз^Ь2. 89. a2 Vbc 63 •
Глава IX 89 90. ’Ve» и 1Vая. 91. 6/4а* и Vali*. 92. 1?/Тба86* и 1V 27flS63. 93. и ^Z.100^068. m*n /onJn *n*n /от .,m/,>>»' 94. у и |/—рщ—. 95. */аЩ 2Уа» и 2Vc9. 95. зп/Ет И 97. y^pj. у^* и j/^”- - Ш РШ? - У(РJ № урну. У(т^)' » |/Ш"- ■1да G. « 103. а I а?—Ь\ 104. а4/о=6. 1С1 *ЬУбаМш ш 3са т+п ■ ,05. lE^.. ш Affr-.. 107. и «Vn'■’,+, а — 6 108. а(а-Щ±^ш 109> и 2/3. 110. 3/7 и 2У~7. 111. 3 я/2 и 2/2. с* 112. 2V 0 и 3*/5. 113. 3/2, 8/2 и 4/2. 114. 3V2, 2я/ 2 и 6/2. 115. Z-/^ и 2/3. О 11R У10 /!и fl, /5 /5 11в 23/9 3/9 ~5~ и IT' 11Х М _и -5"-_ 11&— И 7Г' /8 У~5 Ую Ую _ _ 119. и -Цр . 120. —^ и -jj . 121. a/ab и ФЬ/аЬ. ■а о.з>/1? . М^. ,м. >2Е1 „ у а Ф /* (1 — аб) rf/6(l —аб) 124. —— v и 5—-—-. 6 с 125. (а—6)Уа=6, ~2 и а/77=6. 126.^/7=37, и У У 127. 2а /(,2 — 262, Ь3/Ф — 2Ь* и /а2 — 262 . аЪ y'J&y я__ (2л?— 128. и 5 j?- 129. 2 (4 / 2 + УЗ). 130. 4 (2*/7 — 2 / 3). 131. 2(26/7- с У ft). 132. 3cVd — 2а\~Ф. 133. —/2. 134. 129/5 135. 22s/5. _ 5^5 + 131^ - гуе-тУЗ-иТТ 3 4 Го l/~0 О l/*9 138. 6 . 139. (а + 6—3)/7 140. (5а2 + За — 6)Vа. 141. 7аЬ\'Ъа, 142. — 4а!с/ЗЛ 143. 2у У фуг, 144, —2nVm—n,
Ответы 302. — 2х Vx\ 303. а. 304. . 305. а + у'а2~^. 2х х 1 2 _» п 1_ 1 325. а». 326. л-ту я. 327. с 2. 323. а з *15. 2 329. (в — ху. 330. Va\ 331. Vа~з. 332 /(a + ft)2. 333. ЗУ'д 8/(a —fcjC 334. ,-L. 335. Va^-b. /4а 336. 337. 338-4-- 339. я2 »^л. I'm* 5^х2«-з j// 348. 5. 349. — 52. 35Э. аЬ67аЩй. 351. р' 352. а + Vab + Ь. 353. а + Vab + Ъ. 354. a"— + ^. 355. Va2 — 2 Vab + 4 Vb2. 356. ‘Va — / ft + VT. 366. /. 367. —368. 1. 369. 1. 370. L 371. —I. 372. — 1. 373. — i. 374. i. 375. 2L 376. 91. 377. at. 37a № 379. y. 380. 381. / Va. 382. 3r l/*. 383. i VW+Ж 384. (x—y)i. 385.5L 386. 2 (5 VT— 3)i. 387.4+17*. 388. 5a — 2Ы. 389. — 12. 390. — Vab. 391. — x. 392.(a — b)i. 39a 1 —46i 394. \QQ — \3iY7. 395. a + 3fc + 2iVab. 396. —31. 397,—iVa. 398. У a. 399. a + bi. 400. * ^ ^ i. x2+y* x2-\-y2 1Q1 ЛП1/9 _ 401. 1 - П/& 402. - ^ i. 403. 3-5iV2. 4G6. 7 —fiif/z 407. . 411. n»—3ci> —(3o»ft —*3)/. 419. 1/22 — / l^ 420. 4 (1/26 + 1/ЗД. 421. . * V2 V2 423. Ц- (j-71/2+1+1/1/2— 1-i). Глава XI. 1.0и— —. 2. 0 и a + ft. a 0 и —-—. 4.0и4?-. а /и+ я 2 Б. it 3aft. & 0 и 2а. 7. it l/aft. 8. it л. I 9. 10. It l/(a2— aft + ft2). 11.^1/402-^112. 12. it a. 13. За и а. 14. — 7а3 и 5а3. 15. а it ft. 16. а—5ft И 3ft—tf. 17. 2а и —у. 18. — у и —у
Глава XI ia 1 *18 В 1 81» 1 X -8|3 4 а 5 Ь ‘ 21. т п — и п т . 22. а b ~г и — b а . 23. а t -и-1. а Ь 25. а Ь . 26. 2 а 5а а Ч" b а — Ъ' 0 И а 3 и ~ 7 27. За — b itz Ува2- — lOib + b2 23. cat 2Ь. 2 ’ 29. -|-(3rt:V/3). 30. а 5а Т И ~~6 . 31. — а и — - Ь. 32. 1 и 1. 33 аЬ 34. 2с с anb a-\-b U а+Ь’ 35. а и Ъ. 36. а и Ь. Ч 37. (а2 + &)± У а* — 4аЧ +10сРЬ2 — 4аЬ* + Ь*. 1\а — о)*5 38. ~ (а + Ь + с ± У а2 + Ь1 -}- с2 — ab — ас — be). U а± Уа-—4Ь(а—с) яп 5а+ЗЬ _ Зя + 56 39. 2 ' 8 “ 8 ' й(с + 1) „ аЬ 4- ас + be rt Va2b2 -+- а2& 4- b-c2 — a2bc — ab2c — abc1 42- F+TTc 1 2' 43. а" и а~п. 44. а3" и —а". 45. |/о и — V2a. 74. (х— a — Yb)(x+ V b). 75. (х УаЬ—а—b)(x У ab—a-\-b). 95. qx2 + px + 1 = 0. 96. ax- mbx -j- rrfic — 0. 97. 4 v2—p2 + 4q = 0. 98. a2x2 + a (b — c)x — be .= 0. 99. b(3ac~:^), 100. p® — q. 101. ±20. Qr 103. г?<27 и с >27. 10й —16. 107. 4. 108. трг— (яг + 1)г0 = О. 110. ± Уpq и „ Ъ+ УЬ2 +- 4 ас — Ь + \'b2 + 4 ас 111. 2 И 2а ' ик,у^ . 4|/ Р2 + д- ,,, 100(Ь 4- с) — а + У[№(Ь+с) — а\2+ШлЬ 115 . . 2я l/g" j з l/h~ 116. *—=—а и :——а. 117. Отрезок надо разделить пополам. р У п- — 45 118. Стороны прямоугольника таковы: ^1 получится квадрат . „г. „„ 2 У 5л2—m2-t-4m 2 Г5л2— т2 — т если р2 = 45. 119. —— и = . JL> О 120 с±Ус2-4Н2 / с\ 121. —~—. 122. - 120. 2 Уп-1 yvf + vf
96 , ОтзетьЛ a b а Ь 26. — ■; — ■■ - и — - ; —. I/а+ft I/а+ft I/а + ft Va + ft 27. (а; а), (-а; - а), Ц(1+||/3); |(l -И'З)] и g(l-*V3); §(l+i »/§)]. 28. а + 1; а —1 и 29. (а; а + ft), (a; —ft), (ft + Va2 + *2; *) и — 1—а; 1—а. (Ь — Va? + Ь* Ь). 30. (—а; 2о — 2), (—а; —2а), (2а; а) и (—а; —2й). 31. 7; 5 и 5; 7. 32. (3; 2), (3; —2), (—3; 2) и (—3; —2). 33. 7; 5 и 5; 7. 34. 6; 2. \ 35. 20; 4 и — 20; — 4. 36. 2; 2 и 2; 2. 37. (4; 3), (—4; —3), (3; 4) и (—3; —4). 38. 7; 6 и —6; —7. 41. (3; 2), (—3; 2), (V^2; —9) и (—V^2; —9). 42. 4; 3 и —3; —4. 43. (3; 1), (—3; —1), (г; 3i) и (—г; 31). 44. 8; 4 и 4; 8. 45. (3; 1), (-3;-1), Г-|; 2) и -2). 46. (2; 1), (—2; — I), (1; 2t) У2 У У 2 / И 2/). 50. (9; 4), (4; 9), (4+ VYE-, 4 — 1/13) и (4 — 1/15; 4+ 1/15). г. /З + О/3 3 — 0/3^ /3 — iV~3 3 + iV~2\ rn /n „ 51.(0; 0), ^^ > 2 / и \ * 2 ) № °)» (^ и (-1; -1). _ _ _ _ 63. ft 2), ft 3,, JA±”3«i). 64. ft-i). (i^3Z: ^+12) . (3=|ЙГ; :=ti'>2Z). 65. ft 1), (1; 3), (2 + 2 j/y; 2-2 j/y) и (г-гу'у; 2 + 2 j/y )• re о 2 \ /1 + 1/41 — 9+l/4l\ /1 — V41 —9—V4l\ (2; 1). (3 ; - T). ; ir—) ■ —Щ— > 57. (4; 2), (2; 1), (4; 2) и (2; 1). _ _ 58. (6; 3), (-3; —12 + 3 ^39 ) и (“^2?; 12-3V39j. 59. 5; 3. 60. 333; 111, 115 и 329. 61. it 3; it 4; it 2. 62 it 8; it 6; it 9. 63. it 1; it 4; it 6. 65. it 5; it 2; it 7. 66. 13; 5; 2 и 2; 5; 13. 67. ft 3 5), ft 4; 5,. tlii®. 7) . (6-/£S 5+П® 68. 2; 4; 1 и 2; 1; 4. 69. 5; 12; 13 и 12 5; 13. 70. (1; 1; 1), (1; 1; 1), (J-^-* —.; -4; 7~^--) 71. (4; 6; 3), (4; 3; 6), (9; 2+ i 1/14; 2 — iVlA) и (9; 2—1 »/l4; 2 + 1 Vli). 72 it 1; it 5; it 2. „111 1 1 1 1 1 1 1 1 1 73 • — • — и ■ — • 7Д • • и — • - _ ■ — 2’ 6* 44*3’ 54' 3' 28* 5’* 2 75. (1; 2; 3), (1; 3; 2), (2; 1; 3), (2; 3; I), (3; 1; 2) и (3; 2; 1). 76. (5; —2; —3), (5; — 3; i— 2), ( — 2; 5; —3), ( — 2; —3; 5), ( — 3; 5; —2) и (—3; —2 5). 77. 2; 5; 4; 3 it —7; 4; 5; 12,
Глава XV &/ 7а (3; 5; 4; 7), (17; 5; 4; —7), (10+ Г'йЗ; —4; —5; — Г'М) и (10— V58; —4; —5; Iх58). 79. 10; 6; 5; 3 и 3; 5; 6; 10. 80. 3; 2; 6; 1 и 2; 3; 1; 6. 81. -^(а2 + 2Ь±аУа^-\-4Ь): у (в2 + 2b +а Уа*+4Ь). ■. ; ^-г~-—. 83. ± й; ziz 2с; ± За. 84. zt —; zfc ——; zt - —. /й2+ b2 /й2+й2 ЗЬс Узас УЗаЪ Ка2+ i>2 йл Ьп 85. ± х—; — ■ — ! Уа^+Ь2 У й2+ Ь2 яв л /(a + g — ^(g-t-fe-^c). ^ , /(fr + c — й)(й + 6 — с). **■ — У 2(Ь + с — а) ’—у 2(й+й—£) ^ I /(£ + й — й)(й + й — 1>) “Г 2 (й + Ь — й) 87. *£±1>. g(g-±fc) ^ ±1/£ 2 Гдбй 2^2fcc’ 2УаЬс ' Г 2 Г ' ^ — Ь + с. 2 i> + с — й 2 89. й + 6 z£ Г2й1>; — Ь + У2аЬ\ — a =fr Г'2лй. 90. т + л; m — л; т. 91. 19 и 11. 93. 8 и 5. 95. 6 и 8 или ■—9 и ■—7. 97. 12 и 4. 98 13 и 9. 100. ~ или -|. 101. 35 или 53. 102. 20, 18, 16. 104. 452. 105. 9, 6, 4. 106. 864. 107. 1тУ2^+*)+.1 и */2(2 —■ 109- 14и8- HI*36и 12 ■ч /7^ -X- 4.0 ш/ п2 4 с 112- 5, 12, 13. lia 6, 8, 10. 115. 5, 6, 8. 117. , t——. ш 4p±d_—УЗр2-\-сР и y8fji+(p_2P' |2l, 120сек., 20 йл. 123. 45 и 25. 129l Ум. ,ОГ1 й . d— й 10„ й(»2 + й2)—2bcvzt (о — й) 1/й3(®+й)2—АаЬсУ.^ 1о2. ■ - -*■! — . 1о4. - ■ — — * У2а — й У2а — й 2с а (у — й) zfc: У а2 (у + д)2 — Aabcv 2 а Глава XV. 1. 44; 345. 2. —37; —340. a 1065. л «(« + !). 2 Б. 5050. й 10100. 7. 2л—1; п\ й(л + 1) — 8- 2 *(л-1)я 9. d—a 10. rf = —5. 11. 55; 403. 15. 5; 18. 19. 140; Ю. 23. 10; 265. 27. 10; 47. 12. 26; 155. 1& — 1; 20. 2а 45; 3. 24. 26; 604,5. 28 52; 143. 13. 2; 1661. 17. 4; 528. 21. 9; 2. 25. 7; 61. 2а 10; 2. 14. 56; 680. 18 —2; 330. 22. 0; 7. 26. —9; a
100 Ответы 92. 94. 95. 99. 103. 107. 115. 119. 123. 127. 131. 135. 139. 141. 145. 149. 153. 157. 161. 165. 169. 3774,75; 1456,8; 253,272; 12,5365; 0,148293; 0,000177828. — 0,66722; —2,47207; —1,04574; 7961,6. 552,25. 36,6592. 5,55562. 0,050187. 0,77738. 1,24203. 79570. 248,4. 0,9937. 96. 401,74. 100. 0,000021952. 104. 0,18894. 108. 0,137762. lib 0,79668. 120. 0,85856. 124. 0,90084. 128. 21,55. 132. 0,0025531. 136. 0,88396. 13,715 сл, 41Д45 см. 6,0015 см. 142. 2,9961. 0,7937 м. 9° 54', 538,5. 512 л. 1,3631. 1,199. 1,596. 173. 1,16327. 177. — тг 181. 4- 185. 2. 189. 193. 197. 201. 205. 20& 232. 235. 238. 35. 1000. 0. 1. 100 и ОД. 1,76. а—. 2. 2 -з И 14а 11,ИЗсл. 150. 64,4 кг. 154. 30,62. 158. 21 ,99 л. 162. 0,814. 16а —25,3944. 170. —0,88852. 174. 2974,75. 17а 7. 18а 4 и — 1. isa з. 190. 3; 1 и — 2. 194. 3 и — 5. 19а 1. 202. 0; 1 и 4. 206. 1000 и 0,01. 209. 2,3. 233. У т. 23а —2. I.9C9. 239. 16 и 10. 31494,3; 0,068995; 0,316664; 0,0099782; — 3,76275; —2,60580; —4,32010. 241. 1,6624 и 1,2745. 24а 244. 247. 27 9 и т- и 0,75. 5. 251. 255. 259. 261. 2,25 3 и 20. Около 20 л. 11 603,3 руб, 6 а. 4. 2. 97. 31. 101. 3,5355. 105. 1,4252. 109. 50,4662. 117. 93,832. 121. 0,018886. 125. 0,8233. 129. 8094,66. 133. а13762. 137. 0,58662. 140. 4,4247; 9а 41,846. 102. 0,37325. 106. 0,737а 110. 1,04712. 118. 0,1567. 122. 0,146143. 126. 3,18957. 130. 2,8946. 134. 10,4864. 138. 0,537275. 7,3745; 11,7992. 143. 696,55 см3. 147. 16,453 см. 151. 6-104 155. 2,04 дм. 159. 99,45 см. 163. 0,46763. 167. 1,33496. 171. 0,093428. 17а —4. 179. |. 183. ± 2. 187. 2 и 5. 191. 5,2 и 2,5. 195. 3. 199. 2. 20а 2 и —1,585. 144. 3,275. 148. 1,3184 см. 15а 0,272. 156. 7,871. 160. 74,87. 164. 73,207. 168. 3,42838. 17а 0,85119. 176. — у. 180. 184. 2. 188. а-\-Ь ' 192. 1,5 и —0,5. 19а 4. 200. 1. 204. 10 и ОД. 207. 5 — ^0,836. 2ia — 5,13. 211.1,72. 234. log a(b + Vb* — c*). 237. 1000J 0,001; 1; 0,1. 240. 1000000 и 10. 243. 4 и 2 или 24а 1 и 1 или 249. 363 р. 47 к. 253. 7,18%. 257. Около 23 л. 260. С незначительным повышением в копейках. 26а Приблизительно на 750л3. 8 245. 2 и 248. 3 и 252. 4%. < 256. 4543 р. 12 к. 9 и —3. 16 и 4. 250. 2493 р. 94 к. 254. 5.5%. 25а 19 675,26 руб.
Главы XVII—XVIII—XIX—XX t01 263. Приблизительно 3,3%. 264. Приблизительно 31 900 pyo. 265. Приблизительно 28* 104. 266. 8,85%. 267. 79 948 мИ. 268. 20 454 л3;’ 269. 17 л. Глава XVII. 7. 21ft 120; 15. 8. 40 320; 8 648640; 54 264. 9. (л + 1) л (л — 1).. .(л — k + 3) 10. (от + л)(от+л-1)(от + л-2)...2л. 13. * +4-}. 1 -1-6—(к— 1) . (от —■ п){т —-л —■ 1) (от—л—2)...(от —2л) 14 1.2-3...(л + 1) • 15. 24. 16. 840. 17. 3024. 18. 45. 19. 15. 20. 6. 21. 14 или 3. 22. 7. 24. 24; б; 2. 25. 84; 28. 26. 7920; 720. 27. Ck~l 28. Aknzl 29.к<^±±; пЛ± или А • Глава XVIII. 1. х* + Юх3 + 35л:2 + 50л: + 24. 2. х» + Зх3 — 21х2 — 43л: + 63. 3. х® + 15л:1 + 85л:3 + 225л:2 + 274л: + 120. 4. х® — 4x1 — 37л:3 + 124л:2 + 276л: — 720. 13. 126aW. 14. — 3432+R 15. C^a^u и QJa11*». 1Ь и 17. 84л1. 1& aS \ Глава XIX. ю. 0;1±г^13. 11. 1; 1 гь/^З. 12. 1; 2; —3. 13. —3; -1 "д—. 14. 1; —2; it / V2. 15. (х +1) (х — 2) (л: — 3). 16. (х + l)(v + 2)(х — 2)(л: — 4). 17. (х + 1)(х — 1)2(х — 2). Глава XX. 25. х>- 1 " 2 ' 26. х < — 2. 27. *>!• 28. х> 56. 29.,<_4 30. х<- -4- 31. х>8. 32. 33. а: > 10 д-. • 34. х<2. 35. х>- ,2 "з • 36. х< —5. 37. х'>-2 ‘ за *>7j. СО to н А Сл| 40. х<-*-. 41. х<—3. 42. 1 <х<4. 43. х>~. 44. 3<х<19. о Z 45. Несовместны. 46. Несовместны. 47. х > — 2. 48. а<~ или л>-|-. 49. —Зу<л<J-. 60, или а^>2у* —
СОДЕРЖАНИЕ. I Глава IX. Иррациональные выражения (1—22). I § 1. Общие сведения о корнях. Извлечение корня из одночлена (3). § 2. Вывод ] множителя из-под радикала и введение множителя под радикал (5). § 3. Сокра- ] щение показателей корней и приведение радикалов к общему показателю (6). I § 4. Приведение корней к нормальному виду (7). § 5. Подобие корней (8). § 6. Сло-1 жение и вычитание корней (8). § 7. Умножение и деление корней (10). § 8. Воз-1 ведение корней в степень и извлечение из них корня (13). § 9. Уничтожение иррациональности в знаменателе дроби (15). § 10. Квадратный корень из двучлена J вида A il'B (16). § 11. Задачи на все действия над радикалами (17). § 12. Степени! и корни с дробными показателями (18). § 13. Мнимые числа (20). 1 Глава X. Функции и их графики (23—25). Глава XI. Квадратные уравнения (26—33). § 1. Решение буквенных квадратных уравнений (26). § 2. Свойства корней квадрат¬ ного уравнения (27). § 3. Составление буквенных квадратных уравнений (30). § 4. График квадратной функции. Графическое решение квадратного уравнения (31). Глава XII. Уравнения высших степеней (34—37). § 1. Биквадратное уравнение (34). § 2. Двучленное уравнение (35). § 3. Трехчлен¬ ное уравнение (35|. § 4. Уравнения, левая часть которых разлагается на множи¬ тели (35). § 5. Возвратное или симметричное уравнение (36). Глава XIII. Иррациональные уравнения (38—39). Глава XIV. Системы уравнений степени выше первой (40—50). § 1. Решение системы уравнений (40). § 2. Составление систем уравнений (48). § 3. Графическое решение систем уравнений с двумя неизвестными 2-й степени (51). Глава XV. Прогрессии (51—60). § 1. Арифметическая, или разностная, прогрессия (51). § 2. Геометрическая, или кратная, прогрессия (56). Глава XVI. Логарифмы (61—74). § 1. Общие свойства логарифмов (61). §2. Десятичные логарифмы (67). § 3. Пока¬ зательные и логарифмические уравнения (71). § 4. Задачи на сложные проценты (73). Глава XVII. Соединения (76—76). Глава XVIII. Бином Ньютона (77—78). Глава XIX. Делимость многочленов (78—79). Глава XX. Неравенства (79—80). Глава XXI. Решение неопределенных уравнений 1-й степени (81—82). Глава XXIL Непрерывные дроби (83—84). Глава XXIII. Исследование уравнений (84—86). § 1. Исследование уравнений 1-й степени с одним неизвестным (84—85). § 2. Исследование системы уравнений 1-й степени с двумя неизвестными (86). § 3. Исследование уравнений 2- й степени (87). Глава XXIV. Пределы (86—87). Ответы (88)
ЗАМЕЧЕННЫЕ ОПЕЧАТКИ Стр. Строка 7 1 снизу 14 И сверху 15 1 снизу 21 5 сверху 35 3 сверху Глава А® задача К 23 25 26 52 53 106 126 130 150 153 В тексте Напечатано а + Ь 1—аУЬ 6)s 'Щ V. а-*ь* ! У~а—УТ а + Ь pt=i=—1 х=\Ya7* В ответах Напечатано 3аЧ 263с» 2с" вЛ"-»/“2-1 36V 10атр*/я г 9У« 10о* I» у^в*+и ' eV»-'*+2 Xs V X — у 4(2 VT—2УТ) 2УТ • ••4*78 У 3—*. • Должно быть а \ (а — б)2 / Va-362 V в-5*3 / У^а—УТ УТ+ УТ i2»=/2 = — 1 д: = !/в\* Должно быть Said 26»с* 2с" й»6п-з/2""Т ч 36-V- .10e"W" 9^да •»4-» cncm-“+i х2 У лс — у У 4(2 V7—в/3) 2/Т •« •4-8 У 3 —•• t Сборник алгебраических задач, ч, II.